You are on page 1of 137

ELEv ENTH EDITION

LANGE q &A ™

PSYCHIATRY
Sean M. Blitzstein, MD
Staff Psychiatrist, Jesse Brown VA
Medical Center
Director, Psychiatry Clerkship
Clinical Associate Professor of
Psychiatry
University of Illinois at
Chicago
Chicago, Illinois

New York Chicago San Francisco Athens London


Madrid Mexico City Milan
New Delhi Singapore Sydney
Toronto
Copyright © 2017 by McGraw-Hill Education. All rights reserved. Except as permitted
under the United States Copyright Act of 1976,
no part of this publication may be reproduced or distributed in any form or by any
means, or stored in a database or retrieval system,
without the prior written permission of the publisher.

ISBN: 978-1-25-964395-8
MHID: 1-25-964395-6.
The material in this eBook also appears in the print version of this title: ISBN:
978-1-25-964394-1,
MHID: 1-25-964394-8.

eBook conversion by codeMantra


Version 1.0

All trademarks are trademarks of their respective owners. Rather than put a
trademark symbol after every occurrence of a trademarked
name, we use names in an editorial fashion only, and to the bene t of the trademark
owner, with no intention of infringement of the
trademark. Where such designations appear in this book, they have been printed with
initial caps.

McGraw-Hill Education eBooks are available at special quantity discounts to use as


premiums and sales promotions or for use in
corpo rate training programs. To contact a representative, please visit the Contact
Us page at www.mhprofessional.com.

Notice

Medicine is an ever-changing science. As new research and clinical experience


broaden our knowledge, changes in treatment and drug
therapy are required. The authors and the publisher of this work have checked with
sources believed to be reliable in their efforts to
provide information that is complete and generally in accord with the standards
accepted at the time of publication. However, in view
of the possibility of human error or changes in medical sciences, neither the
authors nor the publisher nor any other party who has been
involved in the preparation or publication of this work warrants that the
information contained herein is in every respect accurate or
com
plete, and they disclaim all responsibility for any errors or omissions or for the
results obtained from use of the information contained
in this work. Readers are encouraged to con rm the information contained herein
with other sources. For example and in particular,
readers are advised to check the product information sheet included in the package
of each drug they plan to administer to be certain
that the information contained in this work is accurate and that changes have not
been made in the recommended dose or in the
contraindications for administration. This recommendation is of particular
importance in connection with new or infrequently used
drugs.

TERMS OF USE

This is a copyrighted work and McGraw-Hill Education and its licensors reserve all
rights in and to the work. Use of this work is
subject to these terms. Except as permitted under the Copyright Act of 1976 and the
right to store and retrieve one copy of the work,
you may not decompile, disassemble, reverse engineer, reproduce, modify, create
derivative works based upon, transmit, distribute,
disseminate, sell, publish or sublicense the work or any part of it without McGraw-
Hill Education’s prior consent. You may use the
work for your own noncommercial and personal use; any other use of the work is
strictly prohibited. Your right to use the work may
be terminated if you fail to comply with these terms.
THE WORK IS PROVIDED “AS IS.” McGRAW-HILL EDUCATION AND ITS LICENSORS MAKE NO
GUARANTEES OR
WAR RANTIES AS TO THE ACCURACY, ADEQUACY OR COMPLETENESS OF OR RESULTS TO BE
OBTAINED FROM
USING THE WORK, INCLUDING ANY INFORMATION THAT CAN BE ACCESSED THROUGH THE WORK VIA
HYPERLINK OR OTHERWISE, AND EXPRESSLY DISCLAIM ANY WARRANTY, EXPRESS OR IMPLIED,
INCLUDING
BUT NOT LIMITED TO IMPLIED WARRANTIES OF MERCHANTABILITY OR FITNESS FOR A
PARTICULAR PURPOSE.
McGraw-Hill Education and its licensors do not warrant or guarantee that the
functions contained in the work will meet your
requirements or that its opera tion will be uninterrupted or error free. Neither
McGraw-Hill Education nor its licensors shall be liable to
you or anyone else for any inaccuracy, error or omission, regardless of cause, in
the work or for any damages resulting therefrom.
McGraw-Hill Education has no responsibility for the content of any information
accessed through the work. Under no circumstances
shall McGraw-Hill Education and/ or its licensors be liable for any indirect,
incidental, special, punitive, consequential or similar
damages that result from the use of or inability to use the work, even if any of
them has been advised of the possibility of such
damages. This limitation of liability shall apply to any claim or cause whatsoever
whether such claim or cause arises in contract, tort
or otherwise.

Contents
Contributors ......................................................................
...................................................................................
...v

Preface ...........................................................................
...................................................................................
... vii

Acknowledgments ...................................................................
........................................................................ viii

1. Child and Adolescent


Psychiatry ........................................................................
......................................... 1

Questions .........................................................................
..................................................................................
1
Answers and
Explanations ......................................................................
..................................................... 17

2. Adult
Psychopathology ...................................................................
............................................................ 33

Questions .........................................................................
................................................................................ 33
Answers and
Explanations ......................................................................
..................................................... 58

3. Somatic Treatment and


Psychopharmacology ................................................................
........................ 81

Questions .........................................................................
................................................................................ 81
Answers and
Explanations ......................................................................
................................................... 103

4. Psychological Treatment and


Management ........................................................................
.................. 123

Questions .........................................................................
.............................................................................. 123
Answers and
Explanations ......................................................................
................................................... 132

5. Legal and Ethical Issues in Psychiatry and


Medicine .........................................................................
141
Questions .........................................................................
.............................................................................. 141
Answers and
Explanations ......................................................................
................................................... 154

6. Differential Diagnosis and


Management ........................................................................
....................... 165

Questions .........................................................................
.............................................................................. 165
Answers and
Explanations ......................................................................
................................................... 185

7. Practice Test
1 .................................................................................
............................................................. 205

Questions .........................................................................
.............................................................................. 205
Answers and
Explanations ......................................................................
................................................... 225

iii
iv Contents

8. Practice Test
2 .................................................................................
.............................................................. 247

Questions .........................................................................
.............................................................................. 247
Answers and
Explanations ......................................................................
................................................... 268

Bibliography ......................................................................
................................................................................
287
Index .............................................................................
...................................................................................
.... 291

Shetal M. Amin, MD Marika Inga


Wrzosek, MD

Contributors
Legacy Community Health Services
Department of Psychiatry
Clinical Assistant Professor
University of Illinois at Chicago
Department of Psychiatry and Behavioral
Chicago, Illinois
Sciences
Director, Medical Student Education Assistant
Baylor College of Medicine
Professor of Clinical Psychiatry Department
Houston, Texas
of Psychiatry and Behavioral Medicine

Medical College of Wisconsin


Jennifer Sprague, MD
Milwaukee, Wisconsin
Psychiatry Resident
Department of Psychiatry
Alexander K. Yuen, MD
University of Illinois at Chicago
Child and Adolescent Psychiatry Chief Fellow
Chicago, Illinois
Institute of Juvenile Research

Department of Psychiatry
Kelley A. Volpe, MD
University of Illinois at Chicago
Child and Adolescent Psychiatry Fellow
Chicago, Illinois
Institute for Juvenile Research
v
This page intentionally left blank

Preface

Welcome to Lange Q&A: Psychiatry, 11th edition. It is my sincere pleasure to


provide you with this edition in support
of your ongoing education in psychiatry. Psychiatric research and practice are at
the forefront of medicine,
incorporating pharmacology, psychology, sociology, and neurosciences; the result is
a genuine biopsychosocial
approach to patient care. Such a sizeable integration yields a broad field in which
to study and test. Because of this, the
11th edition covers the gamut of topics, from adult and child psychiatry, to
psychopharmacology and ethics. All major
diagnoses are covered, with the emphasis being on differential diagnosis and
treatment (whether psychological or
somatic). In fact, this edition consists of eight chapters with 810 questions, and
it has been updated to reflect the
American Psychiatric Association’s Diagnostic and Statistical Manual of Mental
Disorders, Fifth Edition (DSM-5). As
this book is geared toward medical students studying for their Psychiatry clerkship
examinations, as well as for their
United States Medical Licensing Examination (USMLE) Step 2 CK examination, each and
every question is clinically
focused comprising of a patient-centered vignette. Consistent with the USMLE
format, the answers are either
multiple-choice or extended matching items. In order to promote self-assessment and
formative learning, all answers
include detailed explanations regarding why the incorrect choices are flawed. A
list of references are also included for
those students who wish to further research a particular issue.
Finally, a few suggestions on how best to use this book. This book should be
used as a supplement to your primary
learning. It neither replaces reading a basic text on Psychiatry, nor seeing live
patients during a rotation. Chapters 1
through 6 can be used to reinforce or complement certain areas. The final two
chapters (Chapters 7 and 8) are practice
tests, each consisting of greater than 115 randomly ordered questions that simulate
an actual board examination.

Sean M. Blitzstein, MD
vii

Acknowledgments

Thank you to my patient, supportive, and organized editors at McGraw-Hill.


I also sincerely thank my contributors, and my prior mentors and mentees,
who have all given me so much. Finally, to Jackie and Anika, who are my
home.

Sean M. Blitzstein, MD
viii
CHAPTER 1

Child and Adoles cent


Ps ychiatry
Questions
A 15-year-old girl with a
history of major depressive
disorder (MDD) comes to
your primary care office for a
routine visit. When you
walk into the exam ining room,
you notice that she is
withdrawn with her head bent
DIRECTIONS (Questions 1 through 75): For each of down. She displays poor
eye con tact and barely speaks
the multiple-choice questions in this section, select during your interview.
She admits that she uses
the lettered answer that is the one best response in marijuana approximately
twice per week and cuts on her
each case. arm when stressed. Her
mother reveals that she is
particularly worried as
the girl’s paternal uncle
committed suicide 10
years ago. You suspect that she is
Questions 1 and 2 having a recurrence of
depressive symptoms and are
concerned about her risk for suicide. (C)
Prescribe antidepressant medications with

a follow-up appointment in
1. Which of the following factors most increases this 2
weeks.
patient’s risk of committing suicide? (D)
Refer her to a social worker for
(A) Cutting behavior
psychotherapy.
(B) Gender (E)
Tell the mother you will follow up with her
(C) Presence of depression
at your next routine visit.
(D) Relative who committed suicide
Questions 3
and 4
(E) Substance use
A mother
brings her 7-year-old son to you because she is
2. Further history is obtained, and the patient reveals worried that
he sits up in bed in the middle of the night
that she has, in fact, been feeling more depressed and screams.
She says that at those times he is
recently, with difficulty sleeping, low appetite, inconsolable
but eventually falls back to sleep.
fatigue, and problems con
centrating. While she had suicidal ideation 3. Which of
the following is the most likely diagnosis?
during her previous major depressive epi sode, she
(A)
Intermittent explosive disorder
denies it at this visit. At this time, she also denies
self-injurious urges. Which of the following (B)
Narcolepsy
would be the most appropriate plan for treatment? (C)
Nightmare disorder
(D)
Nonrapid eye movement (NREM) sleep
(A) Admit her to the hospital given her

arousal disorder, sleep terror type


history and potential risk.
(E)
NREM sleep arousal disorder,
(B) Ask your psychiatric colleague to assess her

sleepwalking type
at their next available intake.

1
2 1: Child and Adolescent Psychiatry
5. Which of the
following medications would be the
4. During which stage of sleep do these episodes most most
appropriate to treat this patient?
likely occur?
(A)
Carbamazepine (Tegretol)
(A) Stage 1 (B)
Imipramine (Tofranil)
(B) Stage 2 (C)
Lithium
(C) Stages 3 to 4 (D)
Olanzapine (Zyprexa)
(D) Rapid eye movement (REM) stage (E)
Sertraline (Zoloft)
(E) Any stage
6. According to
the Diagnostic and Statisti cal Manual
Questions 5 and 6 of Mental
Disorders, Fifth Edition (DSM-5), which
of the
following symptoms in this patient
A 17-year-old girl with a history of asthma pres ents for a
differentiate her disorder from that in an adult?
physical examination prior to entering college. You note
that she appears angry. Upon further questioning, you (A)
Anhedonia
learn that she has felt irri table for the past 6 months (B)
Decreased concentration
since breaking up with her boyfriend of 2 years. She (C)
Insomnia
says she feels tired all the time and comes home from (D)
Irritable mood
school every day, lies on the couch, and watches
(E) Weight
loss
YouTube videos. Her grades have dropped because she
cannot concen trate. Despite her fatigue, she complains
of difficulty sleeping. She has lost 12 lb over the last 6
Questions 7 and
8
months. She reports that she quit the senior celebration
com mittee, no longer “hangs out” with her friends, can Parents bring
their 6-year-old boy to the clinic at the
not imagine things will improve, and is considering not request of the
boy’s teachers. The teachers report that he
going to college. Her physical and laboratory is quiet in
class. When he does talk, he fre quently makes
examination is normal. errors with
verb tense. His parents recall that his speech
was delayed. On examination, the boy is friendly and
cooperative. His speech is clear, but he uses simple Questions 11 and 12
sentences with a limited vocabulary. Otherwise, his
physical and laboratory examination is normal. An 8-year-old boy is
referred to you by a school nurse
because he has been
complaining of stomach aches every
7. Which of the following is the most likely diagnosis? morning in school. On
interviewing the boy’s mother,
you learn that he does
not like to go to school, insists on
(A) Childhood-onset fluency disorder
coming home immediately
after school each day, and
(B) Language disorder sleeps in his parents’
bed at night. The mother denies
(C) Social (pragmatic) communication other complaints.
disorder
(D) Specific learning disability 11. Which of the
following is the most likely diagnosis?
(E) Speech sound disorder (A) Post-traumatic
stress disorder (PTSD) (B)
Reactive
attachment disorder
8. By what age would failure to speak 200 words be
most consistent with a speech delay in this (C) Separation
anxiety disorder
patient? (D) Social
anxiety disorder (social phobia) (E)
Specific phobia
(A) 1 year
(B) 2 years 12. Which of the
following would you most likely find
(C) 3 years in this patient’s
history?
(D) 4 years
(A) Low levels
of parental control
(E) 5 years
(B) Parent with
an anxiety disorder
Questions 9 and 10 (C) Parents
with a warm and accepting style (D)
Secure
attachment
A 15-year-old boy without prior psychiatric his tory is on (E) Temperament
characterized by
his school concert trip. He is brought to a local

sociability and extroversion


emergency room because of the acute onset of increased
anger, agitation, and paranoia. On inter view, he reports
Questions 13 and 14
feeling unsafe because a govern ment agency is spying
on him. A 16-year-old boy is
brought to the pediatric emer gency
department by his
grandmother who reports he is not
9. Which of the following tests would be the most acting like himself. He
reports that he feels “incredibly
important to order first? great” and does not
need a doctor because he has powers
to heal himself of all
sickness. He speaks rapidly and his
(A) Electroencephalogram (EEG)
grandmother reports he
has not slept more than 6 hours
(B) Glucose tolerance test over the past 3 days.
(C) Positron emission tomography (PET)
Questions: 4–15 3
scan of his head
(D) Thyroid function tests She reports that he
appears extremely revved up and
(E) Urine toxicology screen hyperactive.

13. Which of the


following is the most likely diagnosis?
10. All appropriate laboratory evaluations and studies
(A)
Attention-deficit/ hyperactivity disorder
come back within normal limits. Which of the
(ADHD)
following diagnoses would be most consistent
with this patient’s presentation? (B) Bipolar
disorder, most recent episode
manic
(A) Anorexia nervosa (C) Major
depressive disorder with
(B) Bipolar disorder, manic
psychotic features
(C) Borderline personality disorder (D) Panic
disorder
(D) Generalized anxiety disorder (E)
Schizophrenia
(E) Major depressive disorder
14. Ingestion of
which of the following sub stances
would most likely bring about similar symptoms? (E) “Sometimes
I see girls who make
(A) Alcohol
themselves throw up. Have you ever
done
that?”
(B) Cannabis
(C) Cocaine 17. Which of the
following laboratory abnormali ties
(D) Heroin would you most
likely find in this patient?
(E) Phencyclidine (PCP)
(A) Elevated
iron
15. A 9-year-old boy walks into your office (B) Elevated
protein
accompanied with his mother. They are arguing (C)
Hyperchloremia
about his wanting a new portable video gaming (D) Hypokalemia
system. His mother, exasperated with her son’s (E)
Hyponatremia
behavior, tells you that they were late because it
took him a long time to finally agree to get into 18. Which of the
following types of psychother apy
the car to come to the appointment with you. would likely
be the most effective for this
Alone with you in your office, he appears irritated particular
patient?
and refuses to answer your questions or look up
from his lap. After 10 minutes, he tells you about (A)
Cognitive-behavioral therapy
the “annoying kids” in his class and how they (B) Family
therapy
“made me get in trouble.” Further history from the (C) Group
therapy
mother reveals that, despite the above behavior, he (D)
Psychoanalysis
has never been violent or destructive, and he has
(E)
Psychodynamic psychotherapy
not had any legal problems. Which of the
following is the most likely diagnosis?
Questions 19
through 21
(A) Agoraphobia
An 8-year-old boy
with a history of major depressive
(B) Attention-deficit/ hyperactivity disorder disorder (MDD)
treated with fluoxetine (Prozac) is
(ADHD) brought to the
emergency department after running
(C) Conduct disorder
(D) Generalized anxiety disorder (GAD) (E)
Oppositional defiant disorder (ODD) into the street
in front of a car on the way home from
4 1: Child and Adolescent Psychiatry school. He is
physically unharmed, but refused to speak
to the emergency
room doctor. You are asked to consult
Questions 16 through 18 as you are on
your psychiatry rotation. The emergency
room doctor is
suspicious that the boy’s behavior
A 15-year-old girl who is a competitive figure skater reflected
underlying suicidal impulses.
presents with concerns about her weight. She believes
that she would be a better skater if she could lose weight 19. Which of
the following opening statements would
and feels very upset and frustrated that she has failed in be the
most valuable in facilitating the boy’s
her attempts. The girl reluctantly admits that she
discussion of the situation?
sometimes eats “a whole lot” of food at one time such as
a quart of ice cream, a large bag of potato chips, and a (A) “You
sure were lucky the car swerved at
jar of peanut butter. You also notice abrasions on the the
last minute.”
back of her right hand. (B) “You
weren’t trying to actually get hit by
the
car, were you?”
16. Which of the following opening statements would (C)
“What were you thinking when you ran into
be the most appropriate? the
street?”
(A) “Tell me about the scratches on your (D) “It
sounds like quite a day. Can you tell me
hand.”
about what happened after school

today?”
(B) “I’ve noticed the cuts on your hand. Are you
trying to hurt yourself?” (E)
“Children who try to hurt themselves are

very confused. Are you confused?”


(C) “How did the scratches happen?”
(D) “I see you have scratches on your hand. Do 20. Which of
the following symptoms of MDD would
you have a cat?” be more
likely in this patient compared to an
adolescent with MDD? inform the boy
and his mother of possible adverse
effects of
fluoxetine. Which of the following
(A) Drug use
would be the
most likely side effect?
(B) Hopelessness
(C) Hypersomnia (A)
Hypotension
(D) Psychomotor agitation (B) Liver
toxicity
(E) Weight change (C) Nausea
(D) Sedation
21. Which of the following would be this child’s most (E) Weight
gain
likely method of attempting suicide?
Questions 25 and 26
(A) Firearms
(B) Hanging A 30-month-old girl
is brought to the clinic by her
(C) Jumping from a significant height mother for a routine
visit. The mother tells you that the
girl seems to avoid
affection, often does not look her in
(D) Stabbing
the eye, has stopped
speaking in preschool, and does not
(E) Substance ingestion
really engage with
her preschool peers nor with her
5-year-old brother.
She continues to
22. An 11-year-old boy with enuresis presents to the
Questions: 16–27 5
clinic for routine follow-up. His bed wetting had not
responded to behavioral interventions, so you had
make odd,
repetitive movements with her hands, and
previously initiated treatment with intranasal
prefers to line
her toys up. She becomes acutely
desmopressin (Ddavp) after completion of a full physical
distressed when
routines are changed. The child’s
and laboratory examination. Which of the
physical
examination is unremarkable, but you note that
she does not seem
to cry when she trips over your
examination stool,
keeping her attention on aligning
following signs/ symptoms would be the most
your colored
pencils.
likely adverse effect?
(A) Headache 25. Which of the
following is the most likely diagnosis?
(B) Hypotension (A)
Autism spectrum disorder (ASD)
(C) Liver toxicity (B)
Intellectual disability (ID)
(D) Sedation (C) Rett
syndrome
(E) Tremor (D)
Selective mutism
(E) Social
(pragmatic) communication
Questions 23 and 24

disorder
A 9-year-old boy with a history of panic disorder treated
with cognitive-behavioral therapy is brought to your 26. The mother
asks you about prognosis for her
office by his mother because he has been irritable and daughter ’s
condition. Which of the following is
depressed. On physical examination, the boy appears the most
positive prognostic factor?
depressed but otherwise normal. Laboratory examination (A) Comorbid
epilepsy
is normal.
(B)
Engagement in social skills training (C)
23. What would be the likelihood of this patient having Functional
language by 5 years of age (D)
a comorbid major depressive disorder (MDD)? Intellectual
disability
(E)
Presence of catatonic symptoms
(A) 5%
(B) 15% Questions 27 and
28
(C) 25%
(D) 50% A 12-year-old boy
is referred by the court for evalu ation.
He skips school,
stays out late at night, and verbally
(E) 75%
abuses his
parents. He has run away from home on three
separate
occasions, prompting his parents to call the
24. After a thorough history and mental status
police. He has
been caught shop lifting and has been in
examination, you diagnose the boy with MDD and
numerous physical
fights with his peers.
decide to initiate treatment with fluox etine. You
(D) 33%
27. Upon further history, which of the following would (E) 75%
most likely be found in this patient?
(A) Absence of a biological father Questions 31 and 32
(B) Absence of a biological mother A 9-year-old boy is
referred to you for evaluation
(C) Mother with an anxiety disorder because of repeated
teasing at school related to his
(D) Patient being an only child inappropriate peer
interactions. The teachers report
(E) Parents who do not use corporal
punishment
6 1: Child and Adolescent Psychiatry that at any time,
without warning, the boy will make a
disruptive sound or
shout out in class. They describe him
28. Which of the following personality disorders is this as polite and neat
but restless and jumpy.
boy most likely to develop?
31. Which of the
following is the most likely diagnosis?
(A) Antisocial personality disorder
(B) Avoidant personality disorder (A) Conduct
disorder
(C) Paranoid personality disorder (B)
Oppositional defiant disorder
(D) Schizoid personality disorder (C) Panic
disorder
(E) Schizotypal personality disorder (D) Separation
anxiety disorder
(E) Tourette
disorder
Questions 29 and 30
32. Which of the
following medications would be the
A 10-year-old girl who has recently been diagnosed with most
appropriate to prescribe initially?
diabetes mellitus type I is referred to you by her
pediatrician for an evaluation. You notice that she seems (A) Bupropion
(Wellbutrin)
sad. Her parents are concerned about her being (B) Clonidine
(Catapres)
depressed. Consideration is given for diag (C)
Haloperidol (Haldol)
nosing adjustment disorder with depressed mood versus (D) Paroxetine
(Paxil)
major depressive disorder (MDD). (E)
Venlafaxine (Effexor)

29. Which of the following criteria for the diagno sis of Questions 33 and 34
adjustment disorder most distinguishes it from
MDD? A 7-year-old boy
with leukemia is referred to you
because of concerns
about his mood. His par ents report
(A) Symptoms cause marked distress or
that he fluctuates
between appearing depressed and
significant impairment in functioning.
acting angry. At
times, he plays qui etly in his room, but
(B) Symptoms develop following an at other times he
displays anger outbursts, often hitting
identifiable stressor. his 4-year-old
brother. His mother admits that she has
(C) Symptoms develop within 3 months of the decreased her
expecta tions of him, and feels that since
onset of the stressor. he is ill he should
not receive any punishments.
(D) Symptoms do not persist for more than 6
months following termination of the 33. Which of the
following methods would be the most
stressor. effective way
to engage his mother in a discussion
(E) Symptoms do not represent normal regarding the
role of her actions on the boy’s
bereavement. behavior?
(A)
Acknowledge her guilt and anxiety about
30. Approximately what percentage of children who are her
son’s illness and explain the
diagnosed with diabetes mellitus type I develop
importance of providing limits and
adjustment disorder following their medical
structure for his emotional well-being.
diagnosis?
(B) Empathize
with the trauma of having a sick
(A) 1% child.
(B) 5% (C) Refer
her to a parent support group. (D)
(C) 10% Sit quietly
and make no comments about her

parenting style.
(E) Tell her that treating her son like a baby is complete class
work because he is so easily dis tracted.
hurting him emotionally. When he
completes his work, it is often done carelessly
Questions: 28–38 7 and is
frequently not turned in. The teach ers also note
that he blurts
out replies and has a hard time during quiet
34. It is determined that the boy is suffering from time. The boy’s
mother reports that he has always had a
lot of energy.
Preparing to leave for school in the
morning is
challenging because of her son’s
Questions 37 and 38 disorganization
and forgetfulness. Other wise, she has no
major depressive disorder. You discuss both complaints. She
denies that her son produces any
psychopharmacologic and psychotherapeutic repetitive
movements or sounds.
treatment options, but she is concerned about his
ongoing leukemia treatment. Which of the 37. Which of
the following is the most likely diagnosis?
following approaches would be the most (A)
Attention-deficit/ hyperactivity disorder
appropriate regarding treating both his leuke
(ADHD)
mia and major depression? (B)
Bipolar disorder
(A) Treat the depression prior to the (C)
Conduct disorder
leukemia. (D)
Oppositional defiant disorder
(B) Treat the depression after the leukemia. (C) (E)
Unspecified disruptive behavior
Treat the depression concurrently with the
disorder
leukemia.
(D) Treat the leukemia first and the 38. The patient
is subsequently treated for the above
depression will resolve. condition.
He returns to an appoint ment after
(E) Treat the leukemia as treating the several
weeks, now with repetitive grimacing and
depression will not be effective in the blinking
movements, which have resulted in his
setting of a malignancy. getting
teased in school. Which of the following
classes of
medications is most likely to be
Questions 35 and 36
responsible?
(A)
Benzodiazepines
An 8-year-old boy presents to your office for a rou tine
visit. One month earlier, you diagnosed him with (B) D2
antagonists
Tourette disorder and prescribed medication. He and his (C)
Monoamine oxidase inhibitors (MAOIs) (D)
parents report that the medication has been helpful.
Selective serotonin reuptake inhibitors (SSRIs)
(E)
Stimulants
35. Which of the following disorders would this patient
be most likely to develop? Questions 39
and 40
(A) Autism spectrum disorder A 6-year-old
boy is referred to you by his school to
(B) Bipolar disorder evaluate his
difficulty with keeping up with reading and
(C) Language disorder math despite
his above average intelligence. It is
(D) Obsessive-compulsive disorder (OCD) (E) suspected that
he suffers from a learning disorder, so
further testing
and evaluation is indicated.
Separation anxiety disorder 8 1: Child and
Adolescent Psychiatry

36. Prior infection with which of the following would


39. Which of the
following findings would be required
be most likely found in the history of this patient?
for a
diagnosis of specific learning disorder with
(A) Haemophilus influenzae impairment in
reading?
(B) Influenza virus
(A) The
child has an above average IQ
(C) Parainfluenza virus
(intelligence quotient) but below
(D) Respiratory syncytial virus
average reading achievement.
(E) Streptococcus pyogenes (B)
The child has an average IQ and below

average reading achievement.


An 8-year-old boy with a family history of tic disor ders
is referred to you for an evaluation of behav ioral (C) The
child’s reading achievement is
difficulties in school. His teachers report that he is
substantially below the child’s IQ.
unable to sit still, constantly fidgets, and is unable to (D) The
child’s reading achievement is
substantially above the child’s IQ.
(A)
Hypotension
(E) The child’s reading achievement and IQ are
(B)
Insomnia
both below average.
(C) Liver
toxicity
40. What would be the approximate risk of this child (D) Tremor
having a comorbid psychiatric disorder? (E) Weight
gain
(A) 5%
44. What is the
likelihood that this patient will sig
(B) 10% nificantly
benefit from the methylphenidate?
(C) 25%
(A) 10%
(D) 50%
(B) 25%
(E) 75%
(C) 33%
Questions 41 and 42 (D) 50%
(E) 70%
An 8-year-old child is referred to you for an evalu ation
of bed-wetting. Several behavioral interven tions have
45. Despite
education and reassurance, the mother
been attempted, including eliminating fluid intake in the remains
opposed to stimulant medications but still
evening, scheduled awakenings at night to use the wishes her
son to receive pharmacologic treatment
bathroom, and a urine alarm (a bell and pad). These for his ADHD.
Which of the follow
techniques have been unsuccessful, and the child
ing
medications would be the most appropriate to
continues to urinate in bed every night.
prescribe?
41. What is the likelihood of this patient having a (A)
Aripiprazole (Abilify)
comorbid mental illness? (B)
Atomoxetine (Strattera)
(A) 5% (C)
Citalopram (Celexa)
(B) 10% (D) Mixed
amphetamine salts (Adderall) (E)
(C) 20% Valproic
acid (Depakote)
(D) 50%
Questions 46 and 47
(E) 75%
During a routine
office visit, the mother of a
42. Which of the following medications would be the 37-month-old girl
tells you that she is concerned
most appropriate to prescribe to treat this patient?
(A) Benztropine (Cogentin)
about her daughter’s
behavior. Since the birth of her son
(B) Desmopressin (Ddavp)
4 months earlier, the
mother states that her daughter has
been more irritable
and angry. The child has told her
mother that she does
not want the baby anymore and to
(C) Methylphenidate (Ritalin) take him back. The
mother is especially concerned
(D) Paroxetine (Paxil) because her daughter
tried to bite the baby the week
(E) Trazodone (Desyrel) before.

Questions 43 through 45 46. Which of the


following statements would be the
most appropriate
response to this mother?
A 10-year-old boy is referred to you because his
pediatrician suspects that he may have attention deficit/ (A) “If you
simply ignore your daughter’s
hyperactivity disorder (ADHD). After a thorough behavior,
it will pass.”
history, physical examination, and labora tory (B) “It is
understandable that your daughter is
investigation you make the diagnosis of ADHD. After angry and
experiences jealousy with the new baby
discussing the adverse effects of medications, you
joining the family.”
prescribe methylphenidate to be taken in the morning (C) “The next
time she tries to bite him, you
and at lunch on school days. should
bite her back so she knows how it
feels.”
43. Which of the following would be the most likely
(D) “Tell your
daughter she is being a very bad
side effect of methylphenidate?
girl and
you won’t love her if she
bites the baby.” discussion,
you learn that she is terrified of gaining
(E) “Tell your daughter that she needs to love weight and
believes that she is fat and needs to lose
the baby and be a wonderful big more weight.
She additionally reports that she has not
sister.” menstruated in
the past 6 months.

47. Which of the following tasks would this girl be able 50. Which of
the following laboratory abnorma lities
to perform at her current age? are you
most likely to find in this patient?

(A) Acknowledge her angry and (A)


Hypercholesterolemia
competitive feelings toward her sibling. (B)
Hyperkalemia
(B) Be able to state her age and gender. (C) (C)
Hypocarotenemia
Count to 50. (D)
Increased thyroid-stimulating hormone
(D) Recognize that water poured from one glass
(TSH)
into another of a different size is (E)
Leukocytosis
the same volume.
(E) Ride a bicycle. 51. Which of
the following complications would be the
most
likely indication for admitting this patient to
Questions 48 and 49 the
hospital?

A 4-year-old boy is referred to you for evaluation (A)


Anemia
because he has poor social relatedness. Upon inter view, (B)
Arrhythmia
he appears healthy and well-kempt. He grabs your office (C)
Bradycardia
key off your desk. At your office door, he takes the key (D)
Hypotension
and locks and unlocks your door repeatedly. Despite (E)
Lanugo
attempts to redirect and distract him, he remains 10 1: Child and
Adolescent Psychiatry
preoccupied with this task. After about 10 minutes, you
attempt to take the key away from him and he becomes
Questions 52 and
53
extremely upset, making an insistent, piercing cry.
Questions: 39–51 9An 8-month-old boy
is brought to the clinic by his
mother. She
complains that her son has been expe
48. In which of the following areas would you most riencing screaming
and crying fits when she leaves him
likely expect additional difficulties in this patient? with a babysitter.
She says that in the past he did not
object to being
left with a babysitter and asks you why
(A) Attention he becomes so upset
now, and what she can do about it.
(B) Fine motor skills
(C) Gross motor skills 52. Which of the
following statements would be the
(D) Imaginary play
most appropriate response?
(E) Potty training progress (A) “This
behavior is characteristic of

autistic children.”
49. Which of the following qualities would be most
(B) “It
is possible that your son has
associated with a more favorable prog nosis in this

separation anxiety disorder.”


child?
(C)
“This behavior suggests that you’re not
(A) Easily toilet trained
spending enough time with your son.”
(B) Interested in mechanical toys (D) “It
sounds as though your son is overly
(C) Organized in play
attached to you.”
(D) Reciprocal conversation (E)
“This behavior is normal at your son’s age
(E) Reciting songs and poems from memory
and will pass with time.”

Questions 50 and 51 53. Which of the


following diagnoses would be most
appropriate
if the boy in the previous question
A 17-year-old girl comes to your office for a routine (Question
52) were an 8-year-old boy with similar
visit. She states that she feels fine and offers no com behavior?
plaints. On physical examination, you find that her
weight is 92 lb and her height is 65 in. One year ear lier, (A)
Agoraphobia
her weight was 126 lb, and height 65 in. After further (B)
Normal behavior
(C) Obsessive-compulsive disorder (B)
Impulsivity
(D) Separation anxiety disorder (C) Low
self-esteem
(E) Social anxiety (social phobia) (D) Motoric
hyperactivity
(E)
Pressured speech
Questions 54 and 55
57. If this
child’s school is unable to manage her
A 10-year-old boy is referred to you due to extreme
behavior in
her classroom despite outpatient
difficulties in school. He has been held back a grade and
treatment
and medication, which of the fol lowing
is still not passing his classes. During the course of your
long-term
options would be the most optimal
evaluation, you learn that the boy hears voices telling
school
placement?
him that he is stupid and to leave the classroom. Afraid
to disobey, he goes to the bath (A) Home
schooling
room frequently. He also has difficulty falling asleep at (B)
Parochial school
night because the voices keep him awake. In addi tion, Questions: 52–63 11
you learn that the boy believes others can read his
thoughts. Physical and laboratory examinations are (C) School for
children with learning
normal. You suspect that the boy may be suffer ing from
schizophrenia.
Questions 61 and 62

disabilities
54. Which of the following items in his history would
(D) Residential
treatment
be most consistent with your provi sional
diagnosis? (E) Therapeutic
day school

(A) Always social and outgoing 58. Further history


and evaluation over time result in the
(B) Father with schizotypal personality diagnosis of
bipolar disorder, most recent episode
disorder manic. Which of
the following medications would
(C) Parents getting divorced be the most
appropriate treatment?
(D) Recently transferred schools (A) Bupropion
(Wellbutrin)
(E) Recently used marijuana (B)
Duloxetine (Cymbalta)
(C)
Methylphenidate (Ritalin)
55. Which of the following features would indi cate a
(D) Mixed
amphetamine salts (Adderall) (E)
poorer prognosis in this patient?
Valproic acid
(Depakote)
(A) Acute onset
(B) Affective symptoms Questions 59 and 60
(C) Good premorbid adjustment An 8-year-old boy is
brought to your office by his
(D) Onset before the age of 13 years mother for evaluation
of an upper respiratory infec tion.
(E) Well-differentiated symptoms The mother mentions
that her son has started wetting the
bed again. In
addition, she mentions that the boy’s
Questions 56 through 58 grandmother died
recently and won ders if this is
affecting him.
A 9-year-old girl with a family history of bipolar
disorder is referred to you by her school because of
59. At which of the
following ages would a child
disruptive behavior in class that has been worsen ing normally be able
to appreciate that death is
over the past 3 months. Her teachers report that her irreversible?
energy level is high, and she is markedly more
distractible. She is also sleeping more poorly and is (A) 2 years
increasingly more intrusive into her siblings’ per sonal (B) 3 years
space. Both attention-deficit/ hyperactivity dis order (C) 5 years
(ADHD) and a manic episode are considered.
(D) 7 years
(E) 12 years
56. Which of the following symptoms would be more
consistent with ADHD rather than mania?
60. Which of the
following defense mechanisms is the
(A) Distractibility boy most likely
employing when he is wetting the
bed?
multiple broken bones and ask him how
each
of these fractures happened.
(A) Acting out
(E) Tell
the mother that you notice that the boy
(B) Denial
has
had multiple broken bones and
(C) Regression
recommend that she limit the boy’s sports
(D) Repression
activities.
(E) Somatization
64. The mother of
a 6-year-old boy calls and asks you
A 12-year-old boy with Tourette disorder comes to your for ad vice.
She says that her son still sucks his
office for a routine visit. Two weeks earlier, you had thumb, and
she is concerned about this behavior.
prescribed clonidine for his illness. The boy reports that Which of the
follow -
his tics have subsided slightly since starting the
ing
suggestions for her to do is the most
clonidine, but he complains about the medicine.
appropriate?
61. Which of the following adverse effects is this boy (A) Ask
the dentist to construct a mouth
most likely experiencing?
appliance that will deter sucking.
(A) Dry mouth (B) Coat
her son’s thumb in hot pepper
sauce.
(B) Hypotension
(C) Give
him gum frequently.
(C) Nausea
(D) Ignore
the behavior.
(D) Sedation
(E)
Implement a behavioral system to
(E) Tremor
reward
stopping.
62. The parents bring in the boy’s 7-year-old brother for
Questions 65 and
66
evaluation. After further history is obtained, he is
diagnosed with attention deficit/ hyperactivity disorder A 14-year-old girl
presents to her pediatrician com
(ADHD). Which of the following classes of medications plaining that she has been
“freaking out.” The girl
would be the most appropriate choice for the brother? describes episodes of
shaking, gasping for air, and
feeling like she
is going to die. The feelings intensify for
(A) Antipsychotic
a few minutes and
resolve spontaneously. These
(B) Monoamine oxidase inhibitor (MAOI) episodes have
occurred at various times, in various sit
(C) Serotonin-specific reuptake inhibitor uations, and the
girl is worried that she is going crazy. A
(SSRI) complete history
and physical examination does not
(D) Stimulant reveal any
further relevant symptoms or signs.
(E) Tricyclic antidepressant (TCA)
65. Which of the
following is the most appropriate
63. A 6-year-old boy is brought to the emergency
pharmacologic treatment?
department by his mother, who reports that he was
(A)
Aripiprazole
playing on some steps in front of the house when
(B)
Carbamazepine
he slipped and fell. She tells you that she is
concerned that he might have broken his arm. An (C)
Duloxetine
x-ray of the boy’s arm shows a fracture of the (D)
Sertraline
ulna, as well as signs of several old fractures of (E)
Valproic acid
varying ages. Which of the following is the most
appropriate course of action? 66. Prior to
prescribing medications, which of the
following
should the pediatrician order next?
(A) Recommend calcium supplements and a
multivitamin daily. (A)
Electrocardiogram (ECG)
(B) Refer the boy to an orthopedist for (B)
Electroencephalogram (EEG)
further evaluation.
(C) Set the current broken bone in a cast and
have the boy see his pediatrician for (C)
Neurology consult
follow-up care. (D)
Pulmonary function tests
12 1: Child and Adolescent Psychiatry
(E)
Routine laboratory studies

(D) Tell the boy that you notice that he has had 67. A 5-year-
old girl diagnosed with lupus is seen by
her female pediatrician for a routine visit. After
(dysthymia)
returning home from the clinic, the girl asks her (C) Selective
mutism
friend to “play doctor.” Which of the following
(D) Separation
anxiety disorder
defense mechanisms best describes this behavior?
(E) Social
anxiety disorder (social phobia)
(A) Displacement
(B) Dissociation 71. A 2-year-old boy
is referred to you for evalu ation
(C) Identification due to the
suspicion that the child is the victim of
abuse secondary
to factitious disorder imposed by
(D) Rationalization
another. Which
of the following family members
(E) Reaction formation is the most
likely perpetrator fabricating the
illness?
Questions 68 and 69
(A) Father
A 10-year-old girl with a history of asthma is brought to
(B) Brother
the clinic after a recent increase in her asthma
symptoms. During the visit, you learn that she is being (C) Mother
physically beaten by her mother’s boy (D) Sister
friend on a regular basis. (E) Uncle

68. Under which of the following circumstances does 72. A frustrated mother
brings her 14-year-old son to a
the law require mandatory reporting by a child psychiatrist after
he is expelled from three
physician of suspected child abuse? high schools in 1 year.
She reports the boy has
tried twice to set his
school on fire, has slashed
(A) In all cases school bus tires, and
has broken into the
(B) Only in cases in which the child shows principal’s office to
steal athletic trophies. In
behavioral manifestations of abuse. addition, he has been
suspended numer
(C) Only when consent of a parent or ous times for getting
into fights with other
guardian is obtained. students. She shudders
and tearfully relates that
(D) Only when the physician believes it is in a she recently caught him
singeing one of the family
child’s best interest. cats with a cigarette
butt. Which of the following
(E) Only when the physician has examined all personality disorders is
this boy most at risk of
children in the family. developing in the
future?
(A) Antisocial
69. Which of the following manifestations would be the
(B) Borderline
most likely outcome of the abuse?
(C) Histrionic
(A) Aggression (D) Obsessive-
compulsive
(B) Dissociative disorder (E)
Schizotypal
(C) Generalized anxiety disorder
Questions: 64–75 13

(D) Major depressive disorder


(E) Post-traumatic stress disorder 73. An 8-year-old
boy is brought in by his mother who
complains that
she cannot get her son to listen to
her. She is
frustrated because he fre quently
70. A 4-year-old boy is referred to you because he will ignores
her requests and instructions.
not speak in preschool. Over the course of about 2 Consideration
is given toward the diagnosis of
months, he gradually stopped talking. His mother oppositional
defiant disorder (ODD). Which of
reports that he initially objected to going to the following
features would best support the
preschool, but now no longer complains. She diagnosis in
this child?
states that at times her son is quiet and stays in his (A)
Aggression to people
room, but that she has not otherwise noticed a
(B)
Depressive symptoms
significant change in his speech or behavior.
Which of the following is the most likely (C)
Disobedience toward teachers
diagnosis? (D) Lack of
participation in tasks requiring

attention
(A) Major depressive disorder (MDD)
(E)
Violation of rules
(B) Persistent depressive disorder
74. A 13-year-old girl is seen by her psychiatrist 1 year 76. A 14-year-old boy
who has limited attainment of
after an automobile accident. She dem onstrates conceptual
skills, has spoken language lim ited to
intact language ability and complex motor skills. single words or
phrases, requires sup port for all
She has no identifiable abnor malities in the activities of
daily living, requires supervision at
perception of stimuli, but she has lost the ability to all times, and
who cannot make responsible
read since the accident. Which of the following decisions
regarding the well-being of others.
deficits is she most likely demonstrating?
(A) Agnosia 77. An 8-year-old
girl who has no obvious deficits in
(B) Alexia learning, is
equally mature as her peers, is able to
perform daily
living tasks without sup port, and
(C) Anomia
has sound
judgment.
(D) Aphasia
(E) Apraxia
78. An 11-year-old
boy who has difficulties in academic
skills of
writing, reading, and math, is immature
75. A 10-year-old girl without significant medical
in social
situations, and who has difficulties with
history is brought by her father to the pedia trician for
regulating
emotions and behavior in
evaluation. Over the past school year, she has been
age-
appropriate fashion. While he is able to
having increasing difficul ties going to sleep. Although
provide his
own personal care, he requires support
she has “always had bedtime rituals,” they have
in more
complex daily living tasks, judgment, and
extended in complexity and length. Most of her time in
organization.
the evening is now spent going around the house
numerous times, locking and unlock ing the doors and
windows. While she knows the chances of a burglary are 79. A 17-year-old boy
whose conceptual skills
slim, she is extremely anxious about her safety, and she consistently
lag behind those of his peers, and,
“can’t stop” the urges to perform these behav iors. As a while he has
difficulties perceiving or
result, she only obtains 5 hours of sleep, and she has
been falling asleep in class with diminishing grades.
Which of the follow ing therapeutic interventions is
interpreting social cues and utilizes less com plex
considered the first-line treatment for this disorder? spoken
language, is able to retain basic language
14 1: Child and Adolescent Psychiatry for social
communication. He can care for his
basic
personal needs after exten sive teaching, but
(A) Cognitive-behavioral therapy (CBT) would
eventually require considerable support
(B) Family therapy from co-
workers and supervisors to manage

responsibilities in the future.


(C) Group therapy
(D) Short-term psychodynamic therapy (E)
Questions 80
through 83
Supportive therapy
Match the most
likely disorder with the appropriate
DIRECTIONS (Questions 76 through 93): The fol patient.
lowing group of numbered items are preceded by a
(A)
Anorexia nervosa
list of lettered options. For each question, select the
one lettered option that is most closely asso ciated (B) Autism
spectrum disorder
with it. Each lettered option may be used once, (C)
Bulimia nervosa
multiple times, or not at all. (D) Binge
eating disorder
(E)
Illness anxiety disorder
Questions 76 through 79
(F)
Intermittent explosive disorder
Match the severity level of intellectual disability with (G)
Obsessive-compulsive disorder
the patient’s adaptive functioning. (H) Panic
disorder
(A) No intellectual disability (I) Pica
(B) Mild intellectual disability (J)
Pyromania
(C) Moderate intellectual disability (K)
Tourette disorder
(D) Severe intellectual disability (L)
Trichotillomania
(E) Profound intellectual disability
80. A 14-year-old girl with episodes of palpita tions, Questions 88 through
93
chest pain, shortness of breath, and diaphoresis
who has a normal physical and laboratory Match the age range
with the corresponding devel
examinations. opmental milestone.
(A) Infant (0–
18 months)
81. A 10-year-old overweight girl feels a loss of control (B) Toddler
(18–36 months)
when she quickly consumes large amounts of
(C) Preschool
age (3–6 years)
food, denies heavy exercise, vom iting or laxative
use afterwards, and feels significant guilt and (D) School age
(7–12 years)
shame regarding her behavior. (E)
Adolescence (13–17 years)

88. Focus on
following the rules.
82. An 8-year-old boy with erythematous, chapped
hands, and an otherwise normal physical and
laboratory examination. 89. Establishing self as
autonomous, separate from
caregiver, by
practicing leaving and returning to
the caregiver.
83. A 13-year-old girl with a bald patch on the back of
Questions: 76–94 15
her head and an otherwise normal physical and
laboratory examination.
90.
Establishing trust in the world through
responsiveness
and empathy of a caregiver.
Questions 84 through 87
91. Preoccupation
with superheroes who repre sent
Match the most likely disorder with the appropriate idealized
caregivers as a result of con flicted
patient. feelings
toward caregivers.
(A) Attention-deficit/ hyperactivity disorder
(ADHD) 92. The
development of the ability to think about and
(B) Autism spectrum disorder
manipulate ideas abstractly.
(C) Generalized anxiety disorder
(D) Language disorder 93. The
development of the ability to apply rea soning so
(E) Selective mutism that the
child is not limited only by perceptions.
(F) Social anxiety disorder
(G) Social (pragmatic) communication DIRECTIONS
(Questions 94 through 99): For each of
disorder the multiple-
choice questions in this section, select
the lettered
answer that is the one best response in
84. A 6-year-old girl whose parents are going through a each case.
divorce will not speak while at school.
94. A 16-year-old
girl is brought to you by her mother
because of
dropping grades, apathy, and poor
85. A 9-year-old boy who frequently blurts out motivation.
You learn that she has recently started
comments in class without waiting his turn to be smoking
marijuana on a regular basis. Which of
called on. the
following patterns would most support the
diagnosis of
cannabis use disorder, severe?
86. A 7-year-old boy who performs well in school (A) Over 12
months, using more than
although seems to talk as if reciting a mono logue rather
intended and being unsuccessful at
than interacting in conversation and generally avoids

stopping, with a loss of her summer job and


other children, but does not display repetitive or legal
consequences from her use.
restricted interests.
(B) Over 12
months, having cravings to use,

spending significant time obtaining


87. An 8-year-old boy who is having difficulty in school
marijuana, and missing school in order to
and avoids interactions with his class mates and others smoke.
for fear of embarrassment.
(C) Over 12
months, continuing to use

despite recurrent arguments, being


“high” while driving, using despite lish than math.
Which of the following best
knowing it is affecting her motivation, explains why her
difficulty is not due to a learning
needing increased amounts to get the disability?
same “high,” and having ongoing
(A) Hearing
deficit
cravings when not using.
(B)
Neurocognitive disorder due to
(D) Over 18 months, smoking by herself but
traumatic
brain injury
needing to smoke more to achieve the
same effect.
(E) Over 18 months, smoking most days per (C) Normal
variant in academic attainment (D)
week, multiple times per day, and
Post-traumatic
stress disorder (PTSD) (E)
developing withdrawal symptoms once the
marijuana is stopped. Vision deficit
16 1: Child and Adolescent Psychiatry
98. A 7-year-old
girl is brought to the pediatri cian due
to weight loss.
She has been gradu ally refusing foods
95. As the school psychologist, you are asked to see a
over the past
several weeks to the point she is now
fourth grader who has been consistently acting out
barely even
drinking supplemental shakes. This has
in class. He often lies about things he has done in
resulted in sig
nificant weight loss and fatigue. The girls
class, such as trying to cheat on tests, and he
tells you she
is aware she is losing weight, that she
physically bullies younger children. You learn that
would like to fit
into the outfits she has, but that she is
at home he practices shooting his BB gun at
afraid of vomiting
if she eats the wrong thing. Physical
squirrels as well as at the family dog. He
examination
reveals a child in the 5th percentile of
expresses no concern for these creatures nor
weight, while
her baseline had previously been in the
remorse at his behav ior. Which of the following is
30th per
centile, and basic laboratory studies reveal a
the most likely diagnosis?
mild microcytic
anemia. Which of the follow ing is the
(A) Autism spectrum disorder
most likely diagnosis in this case?
(B) Bipolar disorder (A) Anorexia
nervosa
(C) Childhood onset schizophrenia (B)
Avoidant/ restrictive food intake
(D) Conduct disorder
disorder (ARFID)
(E) Oppositional defiant disorder (C) Binge
eating disorder
(D) Bulimia
nervosa
96. An 8-year-old boy is brought to the pediatri cian for
(E) Specific
phobia
concerns that he has significant tan trums. The tantrums
occur at least three times per week (but frequently more)
99. A 7-year-old
boy is brought to the pediatri cian
and include physical aggression toward inanimate
because, for the
past 6 months, he has been telling his
objects to the extent that he has destroyed property. His
mother that he is a
girl. Ini tially his parents felt this was
parents note that the family walks on egg shells to
“a phase,” but
the patient has become increasingly
prevent setting him off, and they are starting to feel
distressed and
more insistent that he hopes his breasts
“held hostage” by his chroni cally irate mood. They
will grow during
puberty. He has been put ting on his
believe this change in behavior started a year ago, but
older sister ’s
clothing, and prefers playing with her dolls
has markedly worsened in the past several weeks. Which
rather than engaging
in roughhousing with his peers. He
of the following is the most likely diagnosis?
has repeat edly
stated he dislikes his penis and wishes he
(A) Bipolar disorder never had one.
Physical examination reveals normal
(B) Disruptive mood dysregulation disorder male genitalia of
the appropriate Tan ner stage. Based on
(DMDD) this information,
which of the following is the most
(C) Generalized anxiety disorder
likely diagnosis?
(D) Major depressive disorder (MDD) (A) Body
dysmorphic disorder
(E) Oppositional defiant disorder (ODD) (B)
Delusional disorder
(C) Gender
dysphoria in children
97. A healthy 16-year-old student, who started school in (D)
Nonconformity to gender roles
the United States after fleeing a war-engulfed
(E)
Transvestic disorder
region 1 year ago, is struggling to keep up in her
classes, more so with Eng
Answers and Explanations
evaluate
her as soon as possible and a referral is

appropriate, the wait in get ting to a specialist

(psychiatrist) should not delay care. She does not

necessarily need to
be
admitted to a hospital, since she does not
appear
at imminent risk of self-harm. Cer tainly,
waiting
more than that without any intervention
1. (C) Suicide is a considerable risk in depressed would be
inadequate and inap propriate. She
adolescents, and should be specifically addressed would
likely benefit from med ication
during an interview with a patient who appears
management, so a psychiatrist would be
depressed or agitated, or has a history of a suicide
preferable to a social worker in this situa tion, but
attempt. The adolescent suicide rate has increased therapy
would also be appropriate concurrent
substantially dur
treatment.
ing the past few decades. Male gender, a prior
suicide attempt, history of psychiatric illness, 3.
(D) The Diagnostic and Statistical Manual of
family history, and substance use are all risk Mental
Disorders, Fifth Edition (DSM-5) recog nizes
factors for a completed suicide. A history of a NREM
sleep arousal disorders of two types: sleep
prior suicide attempt is the largest risk fac
walking type and sleep terror type. Both are
tor for suicide for both males and females in all
characterized by recurrent epi sodes of incomplete
age groups, and the majority of those who awakenings
from sleep, usually in the first third of the
complete suicide have attempted suicide in the major sleep
episode, with amnesia for the episodes, no
past. Greater than 90% of youth who com or little
recollection of dream imagery, and resul tant
mit suicide have comorbid psychiatric illness,
impairment or distress. Sleep terror type involves
frequently major depression. Although cut ting repeated
episodes of sudden awak ening from sleep
behavior is concerning in adolescents, it is not
accompanied by panic symp toms that begin with a
necessarily associated with the intent to kill
scream; it is associated with unresponsiveness to
oneself. More girls than boys demon strate suicidal
comfort or attempts to awaken the child. Attacks
behavior and attempts, but at least three times typically
last just a few minutes and are often more
more teenage boys complete suicide than do
distressing to the caregiver, as the patient does not
teenage girls. This is because boys more
recall them in the morning. NREM sleep arousal
frequently use guns and other violent methods to disorder,
sleep-walking type, involves the child sitting
attempt suicide. up or
leaving the bed but is not accompanied by terror
or
autonomic arousal. Nightmare disorders occur in
2. (C) Given the adolescent’s history of depres sion the latter
third of the night and during REM sleep, in
and risk factors of substance use and family history, it
con trast to sleep terrors. When awakened, the
is important to initiate appropriate treatment for her
individual quickly becomes oriented, unlike other
quickly. Given the severity of her depression in the
parasomnias where the individual is disoriented,
past, and decline now, it is reasonable to initiate anti
confused, and difficult to arouse. Intermittent
depressant treatment at this visit with close follow-up.
explosive disorder does not occur
While it would be optimal for a psychiatrist to

17
18 1: Child and Adolescent Psychiatry walking,
occur during deep sleep (stages 3–4).

Nightmare disorder occurs during REM sleep.


during sleep and involves outbursts of anger and
sometimes violent behavior. Narcolepsy is 5. (E) This
patient is suffering from a major
characterized by the triad of sleep attacks, depressive
disorder (MDD), single episode. A
cataplexy (sudden loss of muscle tone), and selective
serotonin reuptake inhibitor (SSRI) such
hypnopompic/ hypnagogic hallucinations or sleep as sertraline
is a first-line agent for MDD in
paralysis, as well as a hypocretin defi children and
adolescents. Tricyclic antide
ciency. pressants
(TCAs) such as imipramine cause more
adverse
effects than SSRIs typically do, and in
4. (C) Episodes of sleep terror, as well as of sleep overdose are
much more likely to be lethal. Mood
stabilizers such as carbamaze average number
of words a 2-year-old speaks is
pine and lithium are used for bipolar disor der and 200 words.
Therefore, the inability to speak 200
as adjuncts to the treatment of MDD refractory to words by age 3
would constitute a speech delay.
antidepressant medications alone. Antipsychotics
such as olanzapine are usually reserved for use as 9. (E) It is most
likely that the boy used illicit
adjuncts when psychosis develops or in bipolar substances
during his trip that caused him to
disorder. experience the
acute psychotic symptoms. While
it is important
to rule out other medi
6. (D) In the DSM-5, major depressive disorder cal causes,
substance use would be the most
requires a depressed mood for at least 2 weeks for likely. Seizure
disorders, head injuries, dia betes,
adults to receive the diagnosis, but in the case of and thyroid
disease often would be accompanied
children and adolescents, an irritable mood may by other
symptoms in addition to the new onset
substitute for having a depressed mood. The psychosis.
required symptoms that involve sleep, appetite,
anhedonia, concentration, and psychomotor 10. (B) While a
substance-induced state remains
functioning are the same for adults and possible due to
the large number of substances
adolescents when diagnosing major depressive that cannot be
confirmed by urine toxicology
disorder. screens, it is
important to consider that this
instance of
psychosis is a manifestation of an
7. (B) This boy most likely suffers from language underlying mood
disorder, most likely bipolar
disorder. Language disorder is characterized by disorder. In
addition to an elevated or irritable
persistent difficulties in the acquisition and use of mood, the
diagnosis of bipolar disorder neces
language due to deficits in comprehen sitates that
individuals display three of the
sion (receptive) or production (expressive). following (four
if irritable mood): grandiosity,
Typical symptoms include having a mark edly
limited vocabulary, making errors in tense, and
having difficulty recalling words or producing decreased need for
sleep, increase in talkative ness,
sentences with developmentally appropriate objective or
subjective flight of ideas,
length or complexity. In addi tion, the boy’s distractibility,
increase in goal-directed behav iors,
difficulties are interfering with and engagement in
potentially high-risk behaviors.
Mania in bipolar
disorder can often present
concurrently with
psychotic symp toms including
academic functioning. There is no indication that hallucinations,
delusions, and disorganized
this child has difficulties with speech production thinking. Anorexia
nervosa (an eating disorder)
(i.e., he can clearly articulate pho nemes of and generalized
anxiety dis orders do not present
speech), so diagnosing speech sound disorder with psychotic symp
toms. Individuals with
would be inappropriate. As there are no borderline
personality disorder may have
disturbances in the fluency and pattern of speech, psychotic-like
symptoms, especially when they are
a diagnosis of childhood-onset flu ency disorder regressed, but they
are much more likely to present
(stuttering) is inappropriate. Social (pragmatic) with depres sion,
suicidal thinking, and substance
communication disorder is characterized by use. While
psychotic features can accompany
persistent difficulties in the social use of verbal major depressive
disorder, the depressive
and nonverbal commu nication, for instance symptoms occur
first, and psychotic symp toms are
impairments in greet ings, inability to change more acute and
frequent in bipolar disorder
communication to a given context, difficulties
following rules for conversation or story-telling, 11. (C) This boy’s
behavior and symptoms are most
or difficul ties in understanding what is inferred consistent with
separation anxiety disor der,
but not overtly stated. This child is able to characterized by
developmentally inap propriate
appropri ately modulate verbal communication, but and excessive
anxiety concerning separation from
his difficulty is in the expression of verbal cues. the home or from
those to whom the individual is
There may be a comorbid learning disorder, but at attached.
Consistent with this diagnosis, the boy
this time, there is insufficient information to make does not like to go
to school, comes home
that diagnosis. immediately after
school, sleeps in his parents’ bed
at night, and has
repeated physical symptoms
8. (C) Speech delay refers to expressive language when at school. The
boy is not suffering from
development and number of words spoken. The PTSD because there
is no evidence of a traumatic
event that is persistently re-experienced or has nalizing
behavior disorder that involves a pat tern of
caused symptoms of increased arousal and hostile and
defiant behavior. Children with ODD are
avoidance of associated stimuli. For a diagnosis of often
angry, argumentative, and easily annoyed by
reactive attachment disorder, the boy would need others. In
order to con firm the diagnosis, you would
to have suffered mark need to
gather additional information regarding the
edly disturbed social relatedness, in most contexts, length of time
the behavior has been present (at least 6
beginning before the age of 5 years, and display a months
is required), as well as rule out a mood,
pattern of inhibited, withdrawn behavior toward psychotic, or
substance use disorder. Although anxiety
caregivers. A related disorder, disinhibited social
disorders can present with
engagement disorder, also stems from significant 20 1: Child and
Adolescent Psychiatry
neglect or deprivation and results in children
displaying a mark tension,
irritability, and even noncompliance when
edly overfamiliar approach and comfort with a child is
placed in a new situation (such as a
strangers. The boy’s symptoms are not con sistent doctor’s
office), this child does not appear to have
with social anxiety disorder (social phobia) anxiety in
places where escape may be difficult in
because he does not have a marked and persistent the event of a
panic attack (such as in
fear of social or performance agoraphobia),
nor excessive worry about a
Answers: 4–15 19 number of
events (such as in generalized anxiety
disorder).
While the patient’s present ing
situations, with exposure causing intense anxiety. symptoms are
not primarily involving inattention,
Finally, a diagnosis of specific phobia would
distractibility, or hyperactivity consistent with
require the display of marked and per sistent fear ADHD, there is
considerable comorbidity
cued by the presence of anticipation of a specific between ADHD
and ODD. His lack of violence or
object or situation. serious
violation of rules is not consistent with
conduct
disorder; how
12. (B) Children with parents who have a history of an ever, ODD
(especially if untreated) may lead to
anxiety disorder are at increased risk for conduct
disorder over time.
developing an anxiety disorder themselves. Other
risk factors for developing a child 16. (A) This patient
appears to be suffering from
hood anxiety disorder include parents who have an bulimia nervous,
an eating disorder character ized
anxious, overly controlling, or reject ing style, an by episodes of
bingeing and compensa tory
insecure attachment with one’s primary caregiver, behaviors (e.g.,
purging). Signs of bulimia
and an inhibited and shy temperament. nervosa include
erosion of tooth enamel caused by
acidic vomitus as
well as abrasions on the dorsum
13. (B) These symptoms are consistent with a manic of the hand due to
scraping by the upper teeth as
episode of bipolar disorder. Attention deficit/ the individual
pushes the hand to the back of the
hyperactivity disorder is associated with throat to induce
vomit ing. As it is not a secret that
hyperactivity and impulsivity but not grandiosity the abrasions are
there, but the patient has not
and delusional thinking. Major depression may mentioned them,
a direct but open-ended
have psychotic symptoms if severe, but it would statement, such
as, “Tell me about the scratches
not present with an inflated mood or increased on your hand,” is
most likely to be helpful in this
energy. Panic disorder would present with situation.
Usually, a patient appreciates a direct
recurrent panic attacks and significant anxiety. ques tion rather
than wondering if you ignored or
Although schizophrenia can present with did not notice
something obvious. Questions such
delusional symptoms, there are often other as, “Do you
scratch yourself?” or, “Do you have a
symptoms such as grossly disorganized speech cat?” are not
open-ended; they require “yes” or
and/ or behavior, or flat affect; in addition, this “no” answers and
are unlikely to yield new
patient has significant mood symptoms. information. “Are
you trying to hurt yourself?” or
“How did the
scratches happen?” may sound
14. (C) Cocaine is a stimulant and can produce both accusatory;
patients are more likely to offer
manic and psychotic symptoms. Alco hol, cannabis, information if you
seem nonjudgmental. If this
heroin, and PCP ingestion can induce a psychotic state girl does not
volunteer information that confirms
including hallucina tions and paranoia, but it would not to you that she is
not inducing vomiting, you may
classically be accompanied by manic symptoms. need to use a more
direct but reassuring statement
such as,
“Sometimes I see girls who make
15. (E) This boy likely has ODD. ODD is an exter themselves throw
up. Have you ever done that?”
17. (D) Medical complications can arise for bulimic 21. (B)
Suffocation (e.g., hanging) is the most com mon
patients who engage in compensatory method for
attempting suicide in individ uals under
15 years of
age. Substance ingestion is another
common
method. While firearms are used less
behaviors such as induced vomiting, laxative, or frequently,
they are more often lethal. Other
diuretic use. A hypokalemic-hypochlore mic methods of
suicide frequently attempted by
alkalosis (due to vomiting) is a possible serious children
include stabbing, cut ting, jumping from
finding that can contribute to a cardiac buildings,
running in front of vehicles, and gas
arrhythmia. Anemia (as opposed to increased inhalation.
Some suicide attempts may be
iron) can be seen, as well as decreased protein. mistaken for
accidents, so it is important to
The reduced chloride can lead to elevated sodium directly ask
children if they intended to hurt or kill
due to sodium resorption. Because of these themselves.
changes, it is important to monitor bulimic
patients for electrolyte and acid–base imbalances. 22. (A) Headaches and
nausea are common adverse
effects
associated with the synthetic antidiuretic
18. (A) Bulimia nervosa is most effectively treated hormone,
Ddavp. Hypotension, liver toxicity,
with cognitive-behavioral therapy. Family ther apy, sedation, and
tremor are not typically associated
group therapy, and longer-term, insight oriented with
desmopressin.
therapies (such as psychoanalysis and
psychodynamic psychotherapy) may be used as 23. (B)
Approximately 10% to 15% of children with
adjuncts to cognitive-behavioral therapy, but they
anxiety disorders develop MDD.
have not been demonstrated to be as effective in
changing the behaviors associated with bulimia 24. (C) The most common
adverse effects of fluox etine
nervosa. include
gastrointestinal symptoms (e.g., nausea,
loose
stools), insomnia, agitation, and headaches.
19. (D) When engaging a child in a clinical set ting, it is In general,
hypotension, liver tox icity, sedation,
important to choose words carefully in order to and weight
gain are not side effects associated
establish an open, trusting connec tion, especially with a with
fluoxetine.
troubled and resistant child. It is preferable to initiate
the interview with more open-ended rather than closed 25. (A) This girl’s
history and presentation are
ended questions. The questions should opti mally reflect consistent
with ASD. ASD is characterized by
empathy for the child and their situation without deficits in
social communication and social
appearing too sentimental or judgmental. Also, it is interaction,
as well as restricted, repetitive pat
important to avoid assumptions about the patient’s terns of
behavior, interests, or activities (e.g.,
feelings with out checking with the patient first. The stereotyped
or repetitive movements, inflex ible
state ment, “You sure were lucky the car swerved at the rigidity to
routines, fixated or restricted interests,
last minute” assumes the boy wanted to avoid being hit sensory input
abnormalities). Chil dren with ASD
by the car, and also is a comment that does not lead to have
difficulties with social emotional reciprocity
any elabora tion from the patient. Statements (B) and and have
difficulty engaging in prosocial behavior.
(C) are both judgmental and more likely to shut down Further, they
often have impaired eye contact and
conversation than facilitate it. Also, ask ing a child if have dif
ficulty understanding nonverbal
they are confused is a close-ended question that comes communica
tion or cues. They also have abnormal
across as condescending. social
attachments, such as appearing to have no
interest in
peers (as in this case). The DSM-5 sig
20. (D) Psychomotor agitation is more commonly seen nificantly
revised the category of ASD, so that
in children with MDD compared to ado lescents with verbal
language abilities no longer represent
MDD. Children with MDD may appear more anxious
Answers: 16–27 21
and irritable than sad and depressed. Hypersomnia,
hopelessness, weight change, and illicit drug use are a
separate criterion. While ASD is more com mon
more in males,
it does exist in females. ID may be
comorbid
with ASD, but must be separately
assessed.
ID is characterized by intellectual and
commonly seen in adolescents with MDD
functioning deficits in conceptual, social, and
compared to children with MDD. practical
domains. Rett syndrome is seen only in
females
and is characterized by normal pre natal
and perinatal development, normal head with an
increased risk of developing antisocial per
circumference at birth, and normal psychomo tor sonality
disorder. Conduct disorder is not as
development through the first 5 months of life. closely
associated with avoidant personal ity d
Between the ages of 5 and 48 months, there is isord er,
paranoid personality d isorder, schizoid
deceleration of head growth, loss of hand skills personality
d isord er, or schizotypal personality
with development of stereotyped hand movements disorder.
such as hand wringing, loss of social interaction
(which may improve later), appearance of poorly 29. (D) By definition,
symptoms of adjustment disorder
coordinated gait or trunk movements, and severely do not last
longer than 6 months after a stressor or
impaired expressive and receptive language the
termination of its con sequences. If depressive
develop ment with severe psychomotor retardation. symptoms
persist, MDD may be diagnosed. In
Consequently, children with Rett syndrome, during both
adjustment disorder and MDD, symptoms
the period of regression, may appear to meet must cause
marked distress or significant
criteria for ASD. In this case, there is no evidence impairment
in functioning, may develop following
of the typical physical examina tion abnormalities. a stressor,
may develop within 3 months of the
Of note, Rett syndrome no longer has its own onset of a
stressor, and must not represent normal
diagnostic criteria in the DSM-5 but remains bereavement.
Adjustment disorder may also be
described in the differen tial diagnoses for several given
specifiers depending on the pres
disorders. Selective mutism is characterized by a ence of
anxiety, depressed mood, and conduct
failure to speak, despite normal ability to do so, in
disturbances. Adjustment disorder following
social situ ations where speaking is expected, such bereavement
may be given provided that the
as at school; outside of this social situation, they individual
does not meet criteria for full MDD,
display normal speech and interactions. Chil dren and the
intensity of quality of the grief reaction
with social (pragmatic) communication disorder exceeds what
is expected for cultural, religious, or
display persistent difficulties in how they use age-matched
norms.
verbal and nonverbal communica tion in social
settings, but lack the restricted/ repetitive patterns 30. (D) Following
a diagnosis of diabetes mellitus type
of behaviors and interests that are present in ASD. I,
approximately 33% of children develop
symptoms of
adjustment disorder.
26. (C) ASD is diagnosed four times more in boys than
girls, and when it is present in girls, intel lectual 31. (E) Tourette
disorder is the most likely diagnosis
disability is often comorbid. The pres ence of because the
boy’s outbursts are consistent w ith
functional language by age 5 is a positive vocal tics,
and the report of
prognostic sign, and it also allows children to
engage more effectively in social skills training.
The remaining listed factors negatively affect the “restless
and jumpy” behavior is consistent w ith
prognosis of patients with ASD.
misinterpretation of motor tics. Tourette disorder
most
commonly develops in grade school-aged
27. (A) This patient’s presentation is consistent with boys, and
the involuntary tics may be
conduct disorder (CD). Factors associated
misinterpreted as purposefully dis
22 1: Child and Adolescent Psychiatry ruptive
behavior. It is not unusual for these
children
to have difficulty with social and peer
with an increased risk of a child developing CD
interactions. Of note, there is frequent
include a low IQ, low school achievement, poor
comorbidity with ADHD. This boy does not suffer
parental supervision, punitive or erratic parental from
conduct disorder or opposi
discipline, childhood physical abuse, parental tional
defiant disorder because he is polite and
conflict, the child’s biological father being absent, does not
display hostile, destructive, or angry
antisocial parents, and having a large family.
behaviors. The outbursts are not typi cal of panic
disorder,
in which there are dis crete panic attacks,
28. (A) Children and adolescents who are diag nosed w period s
of intense fear, or discomfort, with
ith cond uct disord er are at increased risk for physical
manifestations such as palpitations and
antisocial personality disorder as adults.
subjective difficulty breathing. There is no
Antisocial personality disorder is not diagnosed evidence
that this boy experiences distress and
until after the age of 18, and one of the criteria is worry
when sepa rated from an important
evidence of conduct disorder prior to the age of
attachment figure as in separation anxiety
15. Earlier onset of conduct disorder is associated disorder.
immune response.
Although there is a specific
32. (B) Clonidine has become the first-line treat ment stressor in this
case, it is unclear whether the
for Tourette disorder. It has a limited side-effect depressive
symptoms would resolve if the
profile and helps control symptoms of a leukemia goes
into remis
frequently associated comorbid disor der, sion; in
addition, given the impairment in this
attention-deficit/ hyperactivity disorder (ADHD). boy’s
functioning and that it is unclear if/ when the
Tricyclic antidepressants (TCAs) have been leukemia would
remit, not treat ing the depression
shown to be effective in the treatment of Tourette would be
inappropriate. Appropriate
disorder, but other antidepressants such as pharmacologic
treatment should be avoided only if
bupropion, paroxetine, and venlafax ine are not there are
specific contrain dications based on the
known to be effective. High-potency medical
treatment the child is receiving. In
antipsychotics such as haloperidol and pimo zide addition, the
effective ness of psychotherapy is
were traditionally the first-line agents for related to the
specific mental illness, not to the
Tourette’s, but are more likely to cause signifi cant presence or
absence of a medical illness.
side effects. Recently, the newer atypical (or
second-generation) antipsychotics such as 35. (D) It is
not unusual for children with Tourette
risperidone and olanzapine have also been used to disorder to
have comorbid psychiatric disor ders.
treat the disorder. Obsessive-
compulsive disorder is a very commonly
associated
disorder, often present ing in adolescence.
33. (A) It is not unusual for parents of a seriously ill Other anxiety
disorders, attentional disorders, and
child to want to try to protect them from any learning disorders can be
seen as well. Autism spectrum
further suffering or distress. Having an ill child is disorders are not
commonly associated with Tourette’s.
overwhelming and anxiety-provok
Answers: 28–38 23
ing. Parents may end up treating such chil dren as
if they were younger than their actual age and Mood lability
can be an associated symptom,
may be more reluctant to set appro priate limits. although the
incidence of bipolar disorder is not
When calling a parent’s attention to the potential particularly
increased. Language or sepa ration
harm of such interaction, a cli nician must be anxiety
disorders are also not increased in
tactful and empathic. By gently individuals
with Tourette disorder.

36. (E) A
subset of children with prior group A
acknowledging this mother’s distress, she will be beta-hemolytic
streptococcal infections have developed
more receptive to hearing feedback about her Tourette
disorder after the infec tion. Other disorders
interactions with her child, including that her associated with
such an infection include OCD and
reluctance to set limits and structure may Sydenham chorea.
This group of disorders are referred
inadvertently be detrimental. While it is essen to as pedi atric
autoimmune neuropsychiatric disorders
tial, empathizing alone will be insufficient to set associated with
streptococcal infections, or PANDAS.
the stage for a discussion of parent–child The other
infectious agents listed are not associated
interactions. Referring her to a parent support
with Tourette disorder.
group might be useful following your initial
discussion with her. Telling her she is treat 37. (A) This boy’s
history is typical of ADHD,
ing her son like a baby is overly harsh and more combined
presentation. He does not complete
likely to make her defensive rather that receptive. tasks, is
careless, loses necessary items, is dis
tracted and
forgetful, is energetic, impulsive, and
34. (C) A psychiatric illness in a child or adoles cent cannot
tolerate playing quietly. Symptoms are
with a medical illness should be treated present before
age 12 and are occurring in at least
aggressively using the type of treatment most two settings,
and are causing impairment and
effective for the specific psychiatric illness, while distress.
Increased energy and impulsivity can be a
keeping in mind potential interactions of symptom of
both ADHD and bipolar disorder, but
treatments. Medical illnesses can exacerbate to be an
exacerbation of bipolar disorder, those
psychiatric illness and vice versa. As such, symptoms must
be associated with energy and
effective treatment of a mental illness may impulsivity
beyond base
positively affect a medical illness if improve line, inflated
mood, grandiosity, increase in goal
ment of psychiatric symptoms enables the child to directed
activity, and a decreased need for sleep.
more fully participate in treatment of the medical This boy does
not display excessive aggression,
condition. In addition, emotional state is related to destruction of
property, deceitful
ness, theft, or serious violations of rules, as seen in
conduct disorder. His behaviors are not 44. (E)
Stimulant medications, such as methylphe
negativistic, hostile, or defiant, so he does not nidate, are
effective in diminishing symptoms of
suffer from oppositional defiant disorder. ADHD in
approximately 70% of patients.
Therefore, he does not have characteristics of a
disruptive behavior disorder. 45. (B)
Atomoxetine is a nonstimulant norepi nephrine
reuptake
inhibitor, which has been found to be helpful
38. (E) Stimulant medications are the first-line in treating
both childhood and adult ADHD. Mixed
treatment for ADHD but have been associ ated with an
amphetamine salts
increased risk of developing tics. In general, if a child
suffers from tics or has a family history of tics,
stimulant medi cations should be avoided, and an
(Adderall) are used to treat ADHD, but they are
alternate medication should be used to treat ADHD if also
stimulants and will have similar side effects
necessary. Benzodiazepines, D2 antagonists, MAOIs, to the
methylphenidate-based medi cations.
and SSRIs do not commonly exacer bate tics and are not
Aripiprazole is a second-generation (atypical)
used to treat ADHD.
antipsychotic, citalopram is an anti depressant, and
24 1: Child and Adolescent Psychiatry valproic
acid is a mood stabi lizer used in bipolar
disorder;
none are used to treat ADHD.
39. (C) To be diagnosed with a learning disor der with
impairment in reading, there must be evidence of an 46. (B) The
clinician can be helpful to parents by
individuals’ performance in a given academic area (in explaining
and normalizing the child’s behavior.
this case, read ing) falling below what is expected for However,
it is still important for the parent to
age. Psychometric testing, such as IQ testing, is one respond to
or correct this behavior. In the case of
measure to assess the disparity between expected this young
girl, jeal
achievement and actual achieve ment. A learning ousy of
her younger sibling is entirely nor mal and
disorder can also be diag nosed if the individual is using is more
difficult for her, since as a preschooler her
significant and excessive compensatory levels of effort language
skills may not be developed sufficiently
or support to sustain average achievement. to
articulate her dis tress. Once the mother accepts
this
child’s behavior as understandable and in the
40. (D) At least 50% of children with learning nor mal
range, she can respond in an empathic
disorders have a comorbid psychiatric dis order. way while
still setting limits on her aggres sive
The most common comorbid condi tions include behavior.
Ignoring the behavior will not address
ADHD, anxiety, and depressive disorders. the
aggression and is not empathic. Under no

circumstances is biting the child back helpful.


41. (C) Most children with enuresis are boys, and
Threatening to withdraw love is manipulative and
overall anywhere between 20% and 40% have a anxiety-
provoking to the child. By telling the
comorbid mental disorder. child that
she must love the baby, the mother is
not truly
empa thizing with her and may elicit
42. (B) Of the choices provided, Ddavp is the treat ment shame and
further resentment toward the baby.
of choice for enuresis; it is a variation of the
antidiuretic hormone vasopressin and is given 47. (B) A 37-
month-old girl is a young preschool age
intranasally. Benztropine is used to prevent child. At
this stage, she should be able to state her
extrapyramidal symptoms caused by antipsychotic age and
gender and be involved in or have
(neuroleptic) use. Meth ylphenidate is a stimulant recently
completed potty train
commonly used in the treatment of ADHD. Paroxetine ing.
Although she has significant language, it
is a serotonin-specific reuptake inhibitor used to treat would be
very unusual for her to use her language
depression and anxiety. Trazodone is an antidepressant
spontaneously to identify and artic ulate her
most often used for insomnia. feelings
about her younger sibling. Although a
precocious
3-year-old may be able to count to 50,
43. (B) The most common adverse effects of meth it would
not necessarily be expected of her until
ylphenidate and other stimulant medications she is 5
years. She is cur rently considered to be in
include insomnia, decreased appetite, weight loss, the
preoperational stage of cognitive development
dysphoria, and irritability. Tremor, hypo tension, and cannot
yet apply concepts of conservation of
weight gain, and liver toxicity are not common matter
(concrete operations). Her gross motor
side effects. skills and
balance are likely not developed enough
to ride a
bicycle. She should be able to ride a
tricycle, however. appears
by 7 to 8 months and generally resolves
with
time. Typically, stranger anxiety is stronger
toward
completely unknown persons than toward
48. (D) This patient’s presentation is suspicious for those
who are more familiar. Because stranger
autism spectrum disorder (ASD). ASD is anxiety
at this age is developmentally appro priate,
characterized by deficits in social communica tion this
boy cannot be diagnosed with sep aration
and social interaction as well as restricted, anxiety
disorder, and he is not overly attached to
repetitive patterns of behaviors, interests, or his
mother. Children with autism spectrum
activities. Of the choices listed, the difficulty with
disorder, in fact, often lack this developmental
imaginary play represents a deficit in marker.
understanding relationships, which more broadly
falls under the diagnostic criterion of deficits in 53. (D) This
boy would be diagnosed with sepa ration
social interaction. The language impairment anxiety
disorder. While an 8-month-old child
criterion of the DSM-IV-TR was eliminated in the would
typically display stranger anxi ety, an
DSM-5. Attentional difficul 8-year-
old would not. This behavior would have
ties, impairment in motor skill development, or normally
gone away for most children by age 3 or
delay in potty training are not necessarily features 4 years.
School age chil dren are expected to be
of ASD. able to
separate from the parent/ caregiver in order
to attend
school and other activities. Individuals
49. (D) Children with autism spectrum disor der (ASD) with
agora phobia often refuse to leave their home
have more favorable prognoses if they are able to due to
the fear of developing a panic attack. Obses
converse meaningfully with others; though no longer sive-
compulsive disorder is characterized by
part of the DSM-5 diagnostic criteria, the acquisition of
obsessions and compulsions; the focus is not
func tional language by the age of 5 years remains a related
to separation. Social anxiety disorder
positive prognostic factor. It is not unusual for children (social
phobia) involves anxiety in a social or
with ASD to be easily toilet trained, enjoy mechanical
performance situation.
toys, be rigidly organized in their play, and be capable
of memorizing or reciting poems, dialogue from a TV 54. (B)
Individuals who develop schizophrenia are more
show or movie. These latter characteristics are not nec likely to
have a parent with a psy chotic or schizotypal
essarily associated with a good prognosis. disorder.
Individuals who develop schizophrenia are
much more
likely to have a history of social withdrawal
50. (A) This girl likely has anorexia nervosa. and
introversion rather than extroversion. Recent
Hypercholesterolemia is common in anorexia stressors
such as parental divorce or moving may
nervosa. Other findings associated with the
contribute to the onset or exac erbation of some
starvation state are mild normocytic normo psychiatric
disorders but not specifically schizophrenia.
chromic anemia and leukopenia. If vomiting is Finally,
recent drug use would be more likely to support
induced, hypokalemia, hypochloremia, and a
diagnosis of substance-induced psychosis.
metabolic alkalosis may be seen. Hypercaro
tenemia, causing yellowing of the skin, may be 55. (D)
Early-onset schizophrenia occurs before age 18
seen if many carrots are eaten in an attempt to years, while
childhood onset schizo phrenia has an onset
satisfy the appetite with a low-calorie food. TSH is
before the age of 15 years. Childhood onset of
not typically altered.
schizophrenia is rare and
26 1: Child and
Adolescent Psychiatry
51. (B) While all of the choices are complications
resulting from anorexia nervosa, a cardiac arrhythmia is associated
with a poorer outcome. A better
considered a major complica tion and therefore alone prognosis
is associated with an acute onset, more
justifies an inpatient admission. If the patient has affective
symptoms, older age at onset, good
multiple minor complications (e.g., anemia, premorbid
functioning, well-differenti
bradycardia, hypotension, lanugo), strong consideration ated
symptoms, and lack of a family history of
for admission to the hospital should also be given.
schizophrenia.
Answers: 39–55 25
56. (C) Children
with ADHD often suffer from low
52. (E) Stranger anxiety occurs as part of nor mal child self-
esteem, while children with mania are more
development and is evidence of the development likely to
be euphoric. Distractibility, impulsivity,
of a secure attachment; it does not suggest that a motoric
hyperactivity, and pres
parent is inatten tive. Stranger anxiety usually sured
speech may be seen in children with both
disorders. an idea or
feeling is expelled or withheld from
consciousness.
Somatization is the pres ence of
57. (E) If a school is unable to manage a child’s physical symptoms
that are a manifes tation of
behavioral or emotional difficulties despite added emotional or
psychological distress.
support and special placement in a sepa rate
classroom, the most appropriate next step is 61. (D) Sedation is a
frequent adverse effect of
therapeutic day school placement. Therapeutic clonidine upon
initiating treatment. With con
schools are designed for students with psychi atric tinued treatment,
the sedation usually sub sides.
disorders that are severe enough that their home Dry mouth is a
less common adverse effect.
schools cannot keep the child or other students Hypotension is a
possible side effect as clonidine
safe and manage the child in a learn ing is also used as
an anti-hypertensive medication.
environment. Home schooling, a parochial school, The reduction in
blood pressure usually does not
or a school for students with learning disabilities result in
significant symp toms, however. In
are not therapeutic in nature and do not have general, nausea
and tremor are not seen.
teachers and staff who are trained to work with
students with emotional and behav ioral disorders. 62. (D) Stimulants are
the most commonly used
Residential placement involves the child residing medication in the
treatment of ADHD, and
at the school while receiving therapeutic treatment although they can
unmask tic disorders, ADHD
and attending classes, and is reserved for students treatment
guidelines state that stimu
whose needs can not be met by therapeutic lants can still be
used in individuals with tics. In
schools. fact, a recent
study found that stimulants did not
worsen tics in
those with tic disorders.
58. (E) Valproic acid is an anticonvulsant, which is Antipsychotics and
MAOIs are not used for
also a mood stabilizer used to treat bipolar ADHD. SSRIs are
infrequently used to aug
disorder. Duloxetine and bupropion are both ment treatment in
some cases and are not con
antidepressants and not appropriate for treat sidered a first-
line treatment. Although TCAs may
ing mania as they may worsen the symptoms or be as effective as
stimulants, their poten tial for
promote rapid-cycling. Methylphenidate and inducing a cardiac
arrhythmia or, less likely,
mixed amphetamine salts are stimulants used in sudden death
precludes tricyclics from being used
attention-deficit/ hyperactivity disor as first line for
ADHD.
der, and are contraindicated in mania because they
may actually exacerbate mania. 63. (D) The finding of
multiple fractures, espe cially
when they are of
different ages, is a
59. (D) Typically, children are 7 or 8 years old when
they begin to understand the irrevers ibility of death. To
understand that death is irreversible, children must red flag for physical
abuse. Even though the boy
have completed the cognitive stage of preoperational may be scared to
report what happened for fear of
thought and punishment, it is
important to try to talk with him
alone and find out as
much as possible.
Recommending extra
vitamins or limited sports
developed the ability to have concrete think ing. does not address the
question of whether the boy is
Usually, this level of cognitive maturity is safe at home. It is
not appropriate to refer the boy
achieved between the ages of 6 and 10. to an orthopedist or
his pediatrician prior to
investigating the
possibility of physical abuse.
60. (C) This boy is displaying regression, a defense
mechanism in which there is an attempt to return 64. (E) In addressing the
delicate task of helping a child
to an earlier developmental phase to avoid the stop or alter their
method of self-soothing,
tension and conflict at the present level of sensitive and
collaborative approaches are much
development (e.g., distress over the more preferable to
abrupt and harsh interven
grandmother’s death). Acting out is a defense tions. Encouraging a
child to stop thumb suck ing
mechanism in which an unconscious wish or with a more flexible
reward system that is enticing
impulse is expressed through action to avoid an for the child engages
the child in making choices
accompanying affect. Denial is a defense about their own body,
rather than impos ing a harsh
mechanism in which the awareness of a painful intervention. Both hot
pepper sauce and a mouth
aspect of reality is avoided by negating sensory appliance are punitive
in nature. However, more
data. Repression is a defense mechanism in which passive approaches
like ignoring or offering gum
are also not likely to be success ful. Because abuse and
are required to report all cases. The law
prolonged thumb sucking can result in significant does not
leave the issue to the clinician’s d
dental problems, helping a child stop thumb iscretion
or parental/ guardian consent. Once the
sucking is eventually valuable. report is
made, child pro tective services (CPS)
may screen
the inci dent “out,” if the incident does
65. (D) This teenager is likely suffering from panic not meet
the state’s d efinition of maltreatment or
disorder. Serotonin-specific reuptake inhibi tors if insuf
ficient information is provided, CPS may
(SSRIs) such as sertraline are effective in the screen the
case “in,” which prompts further
treatment of panic disorder. Antipsychot ics, such
investigation and possibly removing the child from
as aripiprazole, are not indicated for the treatment the home
until an investigation is completed.
of panic disorder. While dulox etine, a
serotonin-norepinephrine reuptake inhibitor, has an 69. (D) Although
all of the listed choices are pos sible
indication for the treatment of generalized anxiety outcomes
associated with childhood abuse, depression
disorder in children over age 7 years, it has not is the most
common one. In particular, abuse of a
been studied exten sively in panic disorder. Mood chronic
nature can have a significant effect on
stabilizers, such as carbamazepine and valproic personality
develop ment, and, in addition, predispose
acid, are not as effective in panic disorder. the child to
significant psychopathology as an adult.
28 1: Child and
Adolescent Psychiatry
66. (E) Although, the symptoms that the patient
describes are consistent with panic attacks, it is 70. (C) This boy’s
history is most consistent with
important to rule out any other medical causes that selective
mutism. He does not speak at school but
could be emulating that presen continues to
talk at home. Consis tent failure to
tation (e.g., metabolic problems such as thy roid speak in a
specific social situ ation despite
dysfunction, electrolyte abnormalities, anemia). speaking in
other situations is not characteristic of
An ECG would be helpful if she reported cardiac MDD, persistent
depres sive disorder (dysthymia),
symptoms as part of her pre sentation. This separation
anxiety disorder, or social anxiety
optional test can be done after disorder
(social phobia). Social phobia can be
Answers: 56–69 27 diagnosed along
with selective mutism.

the bloodwork is complete. An EEG and refer ral 71. (C) The mother
is the most common perpetra tor of
to a neurologist is not necessary unless one has intentionally
producing physical or psy chological
suspicions of a seizure disorder. A pulmo nary symptoms in
her child in order to assume the sick
function test is used to diagnose asthma, which is a role by proxy
(factitious dis order imposed on
much less likely etiology of this patient’s another). The
victim is usu ally a preschool child,
symptoms. and imposing
an illness on a child is considered to
be physical
abuse which requires reporting.
67. (C) Identification is the process of adopt ing other
people’s characteristics. Identifica tion with a parent is 72. (A) The case
represents a child with conduct
important in personality formation. This girl’s behavior disorder.
Individuals with conduct disorder are at
may occur as an attempt to imitate the doctor because increased
risk of later development of antisocial
she admires her, or it may represent an effort to cope personality
disorder; in fact, DSM-5
with anxiety about the doctor because she fears her. criteria for
antisocial personality disorder require
Displacement is a defense mechanism in which evidence of a
conduct disorder before the age of
emotions are shifted from one idea or object to another 15 years.
that resembles the original but evokes less distress.
73. (C)
Disobedience toward authority figures is a
Dissociation is a defense mechanism in which a
characteristic of
ODD. Aggression and violat ing rules
person’s character or sense of identity is temporarily but
are symptoms
consistent with con duct disorder. If the
drasti cally modified in order to avoid emotional dis
hostile and
negativistic behavior consistent with ODD
tress. Rationalization is a defense mechanism in which
occurs only in the
context of a mood disorder, the child
rational explanations are offered in an attempt to justify
can not
be diagnosed with ODD. Reluctance to
unacceptable attitudes, beliefs, or behaviors. Reaction

participate in tasks that require ongoing men tal


formation is a defense mechanism in which an

attention is a common symptom of ADHD.


unacceptable impulse is transformed into its opposite.
74. (B) Alexia is
the inability to read. Agnosia is the
68. (A) In all jurisdictions in the United States, cli
inability to
recognize objects despite intact senses.
nicians are mandated reporters of suspected child
Anomia is the specific inability to name objects note, the
range of IQ for severe intellectual
even though the object is recognizable and can be disability
(for merly MR) was historically 25 to 39,
described by the patient. Aphasia is more global for aver age
(normal) intelligence was historically
than alexia and is an abnormal 90 to 110, for
mild intellectual disability was
ity in either the expression or the comprehen sion historically
55 to 70, for moderate intellectual
of language. Apraxia is an inability to perform
learned motor skills despite normal strength and
coordination. disability was
40 to 54, for profound intellec tual
disability was
below 25, and borderline
75. (A) This patient’s symptoms are consistent with intellectual
functioning was considered an IQ
obsessive-compulsive disorder (OCD). between 71 and
84.

80–83. [80. (H), 81.


(D), 82. (G), 83. (L)] Palpitations,
CBT for OCD is the nonpharmacologic ther chest pain,
shortness of breath, and diaphoresis
apeutic treatment of choice. The CBT often commonly occur
during panic attacks, a com
involves exposure and response preven tion to help ponent of panic
disorder (H). Binge eating
extinguish compulsions. Family therapy can be disorder (D) is
characterized by shame and loss of
helpful in addressing family related issues control over
eating large amounts of food, without
contributing to the child’s symptoms. Group compensatory
behaviors, and without body image
therapy can be part of an OCD treatment, distortions.
Chapped hands and other
particularly CBT. Short-term psychodynamic and dermatologic
problems are often present in
supportive therapies are not considered adequate obsessive-
compulsive disor
treatments alone, although they may be helpful in
der (G) due to
excessive washing with water or
address ing related issues such as self-esteem and caustic
cleaning agents. Trichotillomania
relationships. (hair-pulling
disorder) (L) is much more com mon
in girls, with
the onset frequently starting with, or
76–79. [76. (D), 77. (A), 78. (B), 79. (C)] The DSM-5 shortly after,
puberty. The repeated pulling out of
revised the intellectual disabilities (formerly hair results in
decreased or complete loss of hair
mental retardation, MR) severity scale to be based in a specific
area. The scalp, eyebrows, and
on the degree of deficits in functional impairment; eyelashes are the
sites most commonly involved.
such adaptive functioning involved adaptive Autism spectrum
disorder (B) is characterized by
reasoning in the follow ing domains: conceptual, impairments
in social interactions and the
social, and practical. Individuals may function at development of
stereotyped or repetitive patterns
different levels between each domain. Clinical of behav
assessment (i.e., degree of functional ability) is iors, but without
significant delay in lan guage
necessary to assess adaptive functioning and skills or
cognitive function. Bulimia nervosa (C),
therefore severity of intellectual disability; seen in 1% to
1.5% of adolescent girls and young
psychomet women, is a
bingeing and purging type of eating
rics such as IQ testing should be taken into disorder;
anorexia ner vosa (A) is an eating
account, but may not accurately assess real life disorder with
significant weight loss and fear of
functioning and attainment of practical, daily weight gain.
Illness anxiety disorder (E), formerly
tasks. The deficits in adaptive func tioning must be hypochondria sis,
is the preoccupation that one has
related to the intellectual impairments and occur a serious disease
based on misinterpretation of
in the developmental period. The representative symp toms,
despite appropriate medical evalua
abilities for mild, moderate, and severe tion and
reassurance. Intermittent explosive
intellectual disability are listed in the question disorder (F) is
characterized by recurrent
stem. Individuals with profound intellectual outbursts of
behavior, manifesting as either verbal
disability gen erally are limited to conceptual skills or behavioral
aggression, that are out of
that involve the physical world, rather than sym proportion to
provocation or precipitating
bolic processes (conceptual domain), have limited stressors. Pica
(I) is the eating of nonnutri tional
understanding of symbolic com munication in substances such
as dirt or paint. Pyro mania (J)
speech or gestures but may understand some requires several
criteria, including deliberate and
simple instructions (social domain), and are purposeful fire-
setting and tension or affective
dependent on others of all aspect of daily physical arousal before
the act, which is not better
care, health, and safety (practical domain). Of accounted for by
con duct disorder, a manic
episode, or antisocial powerful
characters such as superheroes helps
Answers: 70–93 29 them cope
with their own feelings of smallness
and
inadequacy (as compared to their parents).
personality disorder. Eye tics such as blinking and This stage is
also called the Oedipal phase; at this
eye rolling are the most common initial symptoms time children
typically experience long
in Tourette disorder (K). Facial tics such as ings toward
the parent of the opposite sex and
grimacing or licking movements and vocal tics jealousy and
hostility toward the same sex par ent.
such as throat clearing or grunting are the next School age
(D) children develop the abil ity to
most common initial symptoms. Whole-body tics, think more
logically and concretely and thus can
such as body rocking or pelvic thrusting, and identify that
equal amounts of liquid in two
self-abusive tics, such as hitting, may develop differently
shaped containers hold the same
later. amount; once
they are able to grasp this concept
of
conversation of liquids, children have
84–87. [84. (E), 85. (A), 86. (G), 87. (F)] Selective mut progressed
into the stage of concrete operations,
ism (E) is rare, and children who suffer from it are where they
can consistently hold onto more than
only mute in certain situations (e.g., school). It one dimension
at a time. Ado lescents (E) further
often involves a stressful life event such as advance in
cognitive abili ties, finally developing
parents’ divorce and the children are mute by the ability
to think in an abstract fashion (reaching
choice. Blurting out in class is a symptom of the stage
Piaget described as formal operations).
impulsivity that is often seen with ADHD (A). No
language impairment is seen as part of ADHD. 94. (C) The DSM-5 rates the
severity of substance use
Autism spectrum disorders (B) have both disorders on
the basis of how many cri teria are
difficulties in social interaction as well as present, with
“mild” severity need ing two to three
restricted/ repetitive interests, while children with criteria,
“moderate” severity needing four to five
social (pragmatic) communica criteria, and
“severe” sub stance use disorder
tion disorder (G) struggle with the practical requiring
more than six criteria. Substance use
applications and nuances of both verbal and criteria
include trouble some behaviors and
nonverbal communication. Children with lan consequences
associated with the use, including a
guage disorders (D) understand social con failure to
meet work or school expectations (such
ventions, but struggle with the acquiring and using as failing
grades), putting oneself at risk (such as
language due to problems with compre hension or driving while
using) or legal problems. The
production. Children with gener alized anxiety criteria also
incorporate evidence of physical and/
disorder (C) display excessive, difficult to control or psy
chological dependence on the substance;
worry, about a multitude of events; the anxiety or this includes
tolerance, withdrawal, heavier use
worry also manifests with at least one (in than
intended, and displaying an ongoing desire or
children) physical symp tom. Social anxiety unsuccessful
attempts to cut back on substance
disorder (social phobia) (F) is characterized by use. Neither
using a substance while alone nor the
anxiety in situations where one is exposed to be frequency of
use are symptoms when diagnosing a
scrutinized, judged, or observed. substance use
disorder; however, using a
substance
most days per week and/ or using alone
indicate the
person’s use is more extensive and
88–93. [88. (D), 89. (B), 90. (A), 91. (C), 92. (E), 93.
poten tially
more concerning, which should
(D)] As children grow, they pass through different
prompt
further questioning.
stages of development that represent emo tional
and cognitive maturation. A vital task of infancy
95. (D) This child
fulfills criteria for conduct dis order,
(A) is to establish a secure sense of trust that
namely, three
of the four following
occurs in the relationship with one’s responsive
caregiver. Toddlers (B) struggle with the
increasing understanding that they are a separate
acts:
aggression toward people and animals,
being from their caregiver, and they practice

destruction of property, deceitfulness, and serious


separating and reuniting with their caregiver as a
violations
of rules. There is nothing to suggest an
way to consolidate their sense of separateness and
autism
spectrum disorder, par
autonomy. Preschool
30 1: Child and Adolescent Psychiatry ticularly
because it would have likely been
diagnosed
much earlier; more importantly, his
actions
seem willful and not repetitive or
(C) children’s identification with strong and

stereotyped. A child with bipolar disor


der may display impulsivity and aggressive opportunity in
her home coun try. Learning
behavior, however, the child maintains an overall difficulties due
to neurological (e.g., stroke,
sense of right and wrong and often experiences traumatic brain
injury) or sensory (e.g., hearing or
regret for the out-of-control behavior once vision
impairment) causes are distinct from
symptoms are improved. Also, bipolar children specific
learning disorders. It is imperative that
have significant mood symptoms that underlie sensory deficits
are screened and treated before
their behavior. Chil giving a
psychiatric diag
dren with conduct disorder may also have a nosis; in this
case, there is no indication of such a
comorbid mood disorder, but mood symp toms are deficit, which,
even if corrected, still would not
not necessarily part of the conduct disorder account for the
variance of this child’s aca demic
constellation. There is no evidence of psychotic background.
Learning disorders are not mutually
symptoms, such as hallucinations or delusions that exclusive from
PTSD, but there is no evidence of
would be characteristic of childhood re-experiencing,
arousal, or avoidance symptoms
schizophrenia. Oppositional defi ant disorder seen in PTSD.
requires 6 months of negativistic, hostile, and
defiant behavior directed mostly at authority 98. (B) This
child is exhibiting symptoms of ARFID,
figures. While it may be a premor bid condition of which is
characterized by avoid ing food or restricting
conduct disorder, it is not as severe. food intake with
out intentional efforts to lose weight
and without
body shape concerns, that result in
96. (B) This child is exhibiting chronic irritability that nutritional
deficiencies and w eight loss. Patients with
is impairing to his functioning, a hallmark of
ARFID do not have the same
DMDD. The DSM-5 classifies DMDD, a new
Answers: 94–99 31
diagnosis, in the depressive disorders seg
ment, and it is therefore important to distin guish it body image
concerns as in anorexia, and are often
from MDD. This patient does not have a aware they
are losing weight. These patients tend
predominantly depressed mood and neuro not to binge
eat as in binge eating disorder, nor
vegetative symptoms (e.g., insomnia, anergia, engage in
compensa
change in appetite, poor concentration) that would tory
mechanisms to lose weight, such as in bulimia
be consistent with MDD. Criteria For DMDD nervosa.
While this child does have a fear of
include that the symptoms have been consistently vomiting,
which could be a result of a specific
present (3 or more per week) for at least 12 phobia, the
overarching clinical concern is her
months, without a symptom-free period more than nutritional
status, worsened by the d istress of
3 weeks; symptoms must be present by age 10 eating. For
some child ren, the avoidance may be
years, and no earlier than age 6 years; symptoms based on the
food ’s sensory characteristics.
must occur in at least two of three settings and Patients who
lack an interest in eating or have a
must be severe in at least one. There is no low appetite
may also qualify for a diagnosis of
evidence of manic symp toms (e.g., decreased ARFID.
need for sleep, elevated mood, pressured speech,
flight of ideas, and grandiosity) consistent with 99. (C) This
child is struggling with gender dys phoria
bipolar disorder. He does not appear to exhibit of childhood.
The core feature of gen der dysphoria
anxiety over (formerly
called gender identity disorder) is an
experience of
incongruence between one’s experienced
gender and
one’s assigned gender. Presentations of
different areas of his life to indicate general ized exhibited
behaviors differ pre- and post-puberty, but the
anxiety disorder. While there is some overlap core features
remain. In body dysmorphic dis order, the
between DMDD and ODD (both may be main feature is
perception of a spe cific body part as
diagnosed), patients with ODD do not have the abnormal, not the
sense that one is the wrong gender. In
predominate irritable mood symptoms interspersed transvestic disor
der, cross-dressing behavior generates
with the anger outbursts in mul tiple settings. sexual
excitement in heterosexual individuals (most
often) and
does not challenge primary gender.
97. (C) This student is most likely struggling due to Insistence
on being of some other gender is not
variations in her academic environment, including considered
delusional in the absence of other psychotic
external factors such as learning in English (not symptoms.
Nonconformity to gender roles should be
her native language), likely hav distinguished
from gender dys phoria by the degree of
ing a gap with disruption of her schooling due to distress and
pervasive ness of gender-variant activities in
war, and possibly a lack of equivalent educational the latter,
driven by the desire to be a different gender.
This page intentionally left blank
CHAPTER 2

Adult Ps ychopathology
Questions
(B) 10%
(C) 20%
(D) 50%
(E)
100%
DIRECTIONS (Questions 1 through 120): For each
of the multiple-choice questions in this section select Questions 3
and 4
the lettered answer that is the one best response in
A 40-year-old
man with schizophrenia comes for his
each case.
regular
outpatient medication management appoint
ments. He
reports that over the last week his intestines
Questions 1 and 2 and heart have
been removed. He has subsequently
withdrawn and
been staying in his basement apart ment,
A 22-year-old single man is referred to you for a 1-year avoiding
friends and family members. When asked
history of strange behavior characterized by talking to about his lack
of getting out in the world, he responds,
the television, accusing local police of bugging his “What world?
There is no world!”
room, and carrying on conversations with himself. His
mother describes a 3-year history of progressive 3. Which of
the following terms best describes this
withdrawal from social activities, and reports the patient symptom?
dropped out of college and since has been living in his
room at home. Attempts to hold a job as a busboy at a (A)
Capgras syndrome
local restaurant have abruptly ended after disputes with (B)
Cotard syndrome
the employers. (C)
Folie à deux
(D)
Fregoli delusion
1. What is the prevalence of this patient’s likely illness
(E)
Major depressive disorder
in the general population?
(A) 0.1% 4. Upon
returning for a follow-up visit 15 days later, the
(B) 1% patient
now claims that “cyborg alien robots” that
(C) 2% look
identical to his parents have recently
replaced
his mother and father. Which of the
(D) 3%
following terms best describes this symptom?
(E) 5%
(A)
Capgras syndrome
2. The patient’s mother informs you that he has an (B)
Cotard syndrome
identical twin brother. What is the likely chance of (C)
Delusional disorder
the patient’s twin also having the same illness? (D)
Folie à deux
(A) 1% (E)
Fregoli delusion

33
34 2: Adult Psychopathology napping. She has
been irritable, and has been pacing in
the middle of the
night and weeping, while talking to no
5. A 32-year-old woman with a history of depres sion as one in
particular. Yesterday she began to ignore the
a teenager, now 6 days postpartum, is brought into the infant, but
today she vol unteered that the child “is the
emergency room by her hus band. She has not slept for Antichrist and
must be destroyed.” She admits to you
the past several days, even while the newborn is that she wants to
smother the infant in a humane way to
prevent the apocalypse. Which of the following within a month.
However, she has not done well
diagnoses is the most likely? in her freshman
year and for the past several
months has
continued to experi
(A) Bipolar disorder
ence worsening
social isolation and amotiva tion.
(B) Delusional disorder While she has not
used any substances since she
(C) Major depressive disorder (MDD) with last saw you, she
reluctantly admits to
psychotic features occasionally
hearing the devil communicat ing
(D) Schizoaffective disorder with her. She
tries to ignore the communi cation,
(E) Schizophrenia and has taken to
arranging her books in a certain
manner to prevent
his controlling her thoughts. On
Questions 6 through 8 her mental status
examina tion she makes poor eye
contact and her
affect is blunted. Her mother
An 18-year-old college freshman without prior psy reports that the
patient now rarely calls home,
chiatric history is brought to the emergency room after though before
she’d do so twice weekly. Which of
being found on her dormitory roof dressed only in her the following is
the most likely diagnosis?
underwear, despite freezing temperatures. Campus
police report she was flapping her hands and climbing (A) Brief
psychotic disorder
the banister on the roof, stating to an unseen other, “I (B)
Schizophreniform disorder
will do as you command—soar to my death to fulfill (C)
Schizophrenia
your prophecy!” She accused the policemen of being (D) Substance-
induced mood disorder (E)
“Satan’s horsemen” and cursed as she was being taken Substance-induced
psychotic disorder
down. Her roommates con
firm that for the past 5 weeks the patient has been acting 9. A 42-year-old woman
presents to a therapist with a
bizarrely, and her speech has been increas ingly history of
dramatic mood swings since early
disorganized. You plan to obtain collateral information adolescence,
where she will quickly become
from her family. deeply depressed
for hours to days, usually in
response to
separation from a loved one. She also
6. Which of the following features would you be most admits to “rage
attacks,” where she will break
likely to find in her history? items, scream, or
scratch herself superficially on
(A) Head trauma her arms. She
inter mittently binge drinks and has
(B) Low intelligence frequently
engaged in unprotected sexual
intercourse with
new partners. Others describe her
(C) Neglectful mother
as
(D) Physical or sexual abuse
(E) Progressive social withdrawal
“reactive” and
intense. Which of the follow ing
7. She is admitted to the psychiatric unit. Her physical defense mechanisms
does this patient most likely
examination is unremarkable, and employ?
(A) Altruism
her blood alcohol is undetectable. Her urine (B)
Intellectualization
toxicology results come back positive for can (C) Splitting
nabis, which she confirms she started smoking (D) Sublimation
about 3 months ago. Which of the following (E) Undoing
would be the most appropriate provisional
diagnosis? 10. The family of a 26-
year-old patient with
(A) Brief psychotic disorder schizophrenia brings
him in for follow-up. He was
(B) Schizophreniform disorder initially diagnosed at
age 25 after a psychotic
break that, in
retrospect, followed a protracted
(C) Schizophrenia
course of increasing
isolation and amotivation.
(D) Substance-induced mood disorder (E) Despite difficulties,
he was able to graduate from
Substance-induced psychotic disorder college and hold a
full-time job by age 23. He
reports intermittent
halluci nations, but has been
8. She was started on appropriate treatment. One year able to maintain inde
later this woman returns to your office with her pendent living and
part-time employment. On
mother for follow-up. Her symptoms remitted
examination, he is a disheveled man who there is no
evidence of thought disorder or
articulates a multitude of delusional beliefs with a
hallucinations. She has been functioning well at
sophisticated vocabulary. Which of the following work and in
other social relationships. Which of
characteristics in this patient is most strongly the
following is the most likely diagnosis?
associated with a better overall prognosis?
(A) Brief
psychotic disorder
(A) Age at presentation (B)
Delusional disorder
(B) Gender of patient (C)
Paranoid personality disorder
(C) Insidious symptom onset (D)
Schizophrenia
(D) Predominantly positive symptoms (E) (E)
Schizophreniform disorder
Premorbid functioning
Questions 14 and 15
Questions 11 and 12
A 46-year-old
divorced woman with a history of major
A 36-year-old patient with no previous psychiatric depressive disorder
is admitted to your inpatient
history is brought to the emergency room by his family. psychiatric unit
following an intentional acetaminophen
For the past month he has not been eating regularly and overdose. She has
had multiple psychiatric
has isolated himself in the apartment where he lives hospitalizations
with similar presen
alone. Two months ago he stopped fishing, and reported tations. The patient
reports a 3-week history of
feeling like a burden on his fiancée. On mental status insomnia, difficulty
concentrating, low energy,
examination, he displays psychomotor retardation and hopelessness, and a
decreased appetite. She has been
rarely blinks. He responds in the negative to any 36 2: Adult
Psychopathology
question asked. On physical examination, he appears in
no acute distress, is afebrile, marginally hypotensive, unable to work
recently because of her depression and
and mildly tachycardic. He resists your motions with has lost interest in
activities she once enjoyed. She has
strength proportional to what you exert and crudely no history of manic
episodes. Her past psy chiatric
mimics your movements. His laboratory studies are history is
significant for a prior episode of depression
unremarkable. after the birth of
her second child. She has undergone
Questions: 5–13 35 treatment with
several adequate tri als of medications,

including
augmentation with lithium, yet she has
11. Which of the following diagnoses is the most likely? continued to have
residual symptoms of depression. She
has never been
treated with psychotherapy. In the past
(A) Catatonia associated with major
she has experienced
auditory hallucinations when her
depressive disorder
depression was most
severe. Her medical history is
(B) Catatonia associated with schizophrenia (C) significant for
hypothyroidism, which is adequately
Hypothyroidism managed with
levothyroxine. She also had prior surgery
(D) Neuroleptic malignant syndrome after a leg fracture
from a suicide attempt where she
(E) Unspecified catatonia jumped out of a
window.

12. Recognizing the clinical situation in front of you, 14. Which of the
following factors would be most
you admit the patient to the psychiatric ward for influential in your
recommending electro convulsive
inability to care for self. Which of the following therapy
(ECT) for this patient?
treatments would be best started immediately? (A) Concurrent
thyroid dysfunction with
(A) Amitriptyline adequate
treatment
(B) Electroconvulsive therapy (ECT) (B) History of
associated psychotic
(C) Lithium
symptoms with prior depression
(D) Lorazepam (C) History of
peripartum depression
(E) Sertraline (D) Severe
depression that has not

responded to several medications


13. A 49-year-old bank teller with no known psy chiatric (E)
Treatment-resistant depression with
history is referred to your office by her internist
recurrent suicidal ideation
for an evaluation. For the past 2 months, she has
been increasingly convinced that a well-known 15. The indications for ECT
are discussed with the
music star is in love with her and that they have patient, as well as the
risks, benefits, and side
had an ongoing affair. She is well-groomed, and effects. She asks
appropriate questions and
consents to ECT. Further medical history is been spectacular!” His
wife has since called you to
explored prior to beginning the procedure. Which report that her
husband has been spending money on
of the following conditions would be a relative frivolous items and
overdrew the cou ple’s bank account.
contraindication to proceed? She states that her
husband now quickly becomes
agitated and angry.
During the interview in your office,
(A) Coronary artery disease, with a
the patient questions
your credentials and accuses you of
myocardial infarction 2 years ago
being more loyal to
his wife than to him. Most of the
(B) Implanted pacemaker for periodic interview is spent
interrupting the patient as you try to
arrhythmia decipher his
(C) Incidentally found frontal meningioma
measuring 10 cm in diameter
(D) Second-trimester pregnancy rapid speech. In his
rant he threatens to cut the brake
(E) Traumatic brain injury sustained in lines on his mother-in-
law’s car because he feels she has
teenage years been intruding in his
marriage.

18. Which of the


following is the next most appropriate
Questions 16 and 17 step?

A 34-year-old white male is referred by his primary care (A) Discharge the
patient home as he
physician for depression. Upon initial inter view, he refuses
admission, but see him
complains of feeling “blue.” His mental status tomorrow
examination reveals a disheveled appearance, depressed (B) Escort the
patient (with police assistance as
mood, psychomotor retardation, and sui cidal ideation needed)
to the nearest emergency
without plan. His thought processes are significant for room
thought blocking and some slow ing. Deficits with (C) Inform the
mother-in-law that she is in
remote and short-term memory are noted. Judgment and danger
insight are also impaired. Your provisional diagnosis is (D) Inform the
police that a threat has been
major depressive disorder. made
against the mother-in-law
(E) Tell the wife
to have the mother-in-law stay
16. Which type of sleep disturbance you would most
with her
expect to see in this patient?
(A) Decreased response to sedative drugs (B) 19. After addressing the
above situation, the patient is
Early morning awakening subsequently
started on valproic acid. Which of
the following
additional phar macologic
(C) Increased rapid eye movement (REM)
interventions would
be the most appropriate?
stage latency
(D) Sleeping too deeply (difficulty being (A) Check a serum
paroxetine level
awakened) (B) Cross taper
the paroxetine to
(E) Sleeping too lightly (awakened too
nortriptyline
easily) (C) Discontinue
the paroxetine
(D) Initiate
bupropion
17. Which of the following metabolic changes would be
(E) Initiate
lorazepam
most likely found in this patient?
(A) Decreased monoamine oxidase (MAO) Questions 20 and 21
activity
A new patient comes to
your office for a psychiat ric
(B) Increased catecholamine activity evaluation. He describes
many years where he
(C) Increased cortisol secretion experiences episodes of 5
to 7 days of feeling very
(D) Increased sex hormones depressed, with insomnia,
low energy, and poor
(E) Increased immune functions concentration. He denies
any suicidal ideation or prior
attempts. Upon further
history he reveals additional
Questions 18 and 19 periods where “I feel the
opposite,” with decreased
sleep, elevated energy,
feeling “on top of the world,” and
A 35-year-old man is being treated for major depres sive increased sex drive. These
times last 1 to 2 weeks, and
disorder with paroxetine. He has missed his last two he denies any paranoia,
delusions, or hallucinations.
appointments, leaving messages telling you that “I’ve Despite these recurrent
episodes, he has always been
able to function adequately, although they remain (E) LSD
distressing to him. He denies any drug or alcohol use,
and he has no significant medical problems. Questions 24
through 26
Questions: 14–23 37
A 46-year-old
man is admitted to the hospital for
20. Which of the following diagnoses is most likely? elective
cholecystectomy. On hospital day 4, he is noted
to be
afebrile, but acutely diaphoretic, tachy cardic,
(A) Bipolar I disorder hypertensive,
tremulous, and agitated. He tears out his
(B) Bipolar II disorder sutures and
insists on leaving against medical advice
(C) Cyclothymic disorder (AMA). He is
apparently hallucinat ing, judging from his
(D) Double depression insistence
that he be allowed to “squash those bugs on
the wall”
(there are none).
(E) Persistent depressive disorder
24. Which of
the following diagnoses is the most
21. The patient returns within 4 weeks, and he now
likely?
describes worsening depression for the past 3
weeks, with ongoing insomnia, poor appetite, (A)
Brief psychotic disorder
little energy, anhedonia, and poor concentra (B)
Delirium
tion. He also admits to passive suicidal ideation (C)
Delusional disorder
without plan. Which of the following would be
(D)
Functional neurological symptom
the most likely diagnosis?

disorder
(A) Bipolar I disorder (E)
Postoperative sepsis
(B) Bipolar II disorder
(C) Cyclothymic disorder 25. Which of
the following medications would be most
(D) Double depression
appropriate to treat his condition?
(E) Persistent depressive disorder (A)
Diazepam
(B)
Disulfiram
Questions 22 and 23 (C)
Lorazepam
A 26-year-old female presents to the psychiatric (D)
Phenobarbital
emergency department in an acutely distressed, nervous (E)
Phenytoin
state. She complains of terrible anxiety, and the
emergency department staff is unable to calm her down 26. If untreated, what
would be his most likely mortality
or gain an adequate history from the patient. On physical rate?
examination, she is slightly diaphoretic, tachycardic, and
(A) 5%
her pupils are mildly dilated. She is on no medications.
(B) 10%
22. Which of the following tests would be the most (C) 30%
useful in determining the etiology of her (D) 50%
symptoms? (E) 60%
(A) Blood glucose
Questions 27
and 28
(B) Catecholamine metabolites
(C) Electrocardiogram (ECG) A 62-year-old woman
presents to the nursing home
(D) Thyroid function where you work as a
consulting psychiatrist. She has a
(E) Urine toxicology history of a bilateral
temporal lobectomy for intrac table
seizures. After a few
weeks at the new facility, in
23. Which of the following substances would be most addition to her short-term
memory difficulties,
likely to appear on her urine toxicology results?
(A) Caffeine the staff
reports that she has become extremely doc ile
(B) Cannabis and
displays very little emotion. She has a large appetite
38 2: Adult Psychopathology and
compulsively puts both food and non food items in
her mouth.
She also displays sexual disinhibition, often
(C) Cocaine walking out
of her room without her pants on.
(D) Opiates
27. Which of the following clinical conditions best 31. A 23-year-old college
student has been suf fering
describes her behavior? with frequent episodes
of feeling “utter doom” for
the past 3 months.
During these periods, he also
(A) Arnold–Chiari syndrome
experiences tremulous
ness, sweating, dizziness,
(B) Klüver–Bucy syndrome and tingling in his
extremities. He reports having
(C) Möbius syndrome these attacks at least
once a week and is now
(D) Pick disease becoming fear ful of
attending classes lest he has
(E) Punch-drunk syndrome an episode. Which of
the following medications
would be the most
appropriate for immediate
28. Neuroimaging would most likely show dam age to relief of his symptoms?
which temporal lobe structure? (A) Alprazolam (Xanax)
(A) Amygdala (B)
Chlordiazepoxide (Librium)
(B) Hippocampus (C) Divalproex
sodium (Depakote)
(C) Inferior horn of the lateral ventricle (D) Fluoxetine
(Prozac)
(D) Insula (E) Phenelzine
(Nardil)
(E) Superior temporal gyri
Questions 32 and 33
Questions 29 and 30
A 36-year-old graduate
student comes to your office
A 22-year-old woman presents with fatigue for 4 because of difficulty sleeping
since breaking up with his
months. She also reports irritability and poor energy, fiancée 5 months ago. He is
spending at least 2 hours in
with dismal sleep and poor concentration. She has bed thinking about his ex-
fiancée and what he could
maintained a rigorous exercise routine, as she states it have done differently.
His con
makes her feel good to run off her boyfriend’s cook centration is worsening
and he’s having difficulties
ing. Her oral intake has sharply dropped and she has lost completing his
coursework. While he feels down, he is
a significant amount of weight, but she explains that she not suicidal, and he is
seeking support of friends in the
“hasn’t been as hungry” due to increased stress. Her post-breakup period. He is
attending most classes. He
thyroid studies are normal, her CBC reveals anemia, and describes feeling tenser
overall, espe
she is not pregnant, in fact stat cially when he is at a
bar. He is having more trouble
ing that she hasn’t had a period in several months. relaxing and has noted
the new onset of low back and
shoulder pain.
29. Upon further questioning, which of the following
qualities would you most expect to find in her 32. Which of the following
is the most likely diagnosis?
social history? (A) Acute stress
disorder
(A) Her parents likely praise her successes. (B) (B) Adjustment
disorder
She has a legal history significant for reckless (C) Generalized
anxiety disorder
driving.
Questions: 24–34 39

(C) She has aspired to be a model.


(D) She is of low socioeconomic status. (D) Major
depressive disorder
(E) She is a scholastically gifted (E) Normal
reaction
perfectionist.
33. Seven months
later, your patient’s mood has
worsened, and he
feels worthless and hope less
that he will
never find another girlfriend. He has
30. Which of the following diagnoses is the most likely?
lost a few
pounds and has not resumed dating,
(A) Anorexia nervosa preferring to
isolate himself in his studio
(B) Avoidant/ restrictive food intake apartment. He is
still feeding his dog, but he has
disorder (ARFID) had to miss
increasing amounts of work because
(C) Bulimia nervosa he just can’t
get himself to get ready for class.
Which of the
following is the most likely
(D) Exercise-induced amenorrhea
diagnosis?
(E) Obsessive-compulsive disorder (OCD)
(A) Acute
stress disorder
(B) Adjustment disorder heroin for the
last 8 months with substantial (but
(C) Generalized anxiety disorder unsuccessful)
efforts to quit for the last 4 months.
She is now
homeless and has recently been
(D) Major depressive disorder
arrested for
shoplifting. Which of the following
(E) Normal reaction pharmacological
ther apies would be the most
appropriate at
this time?
34. A 21-year-old college student is brought to your
office by her parents after completing her spring (A)
Buprenorphine
semester with uncharacteristically low grades. Her (B) Clonidine
parents report that since the spring session ended (C) Haloperidol
2 months ago their daughter has been staying in (D) Methadone
her room, but is irritable when they try to engage
(E) Naloxone
her in conversation. She is not interested in family
barbecues or being a counselor for the park
37. A 47-year-old
woman transfers care to your office.
district soccer camp. After her parents leave the
She has a long
history of recurrent depression.
examination room, your patient discloses she has
She also has a
history since her early twenties that
been feeling depressed since a month before final
is significant
for intermit
examinations, and her concen tration and sleep
tent command
auditory hallucinations and
have been “horrible.” She had suicidal thoughts
persecutory
delusions that are often pres ent even
after she got her report card back, but denies them
when her
affective symptoms are in remission.
currently. She also admits to drinking alcohol to
She holds a part
time job and lives independently.
make herself feel better, and her intake ranges
She has been
compliant with medications and has
from 3 to 4 beers or shots of liquor most days of
not been
hospitalized since a suicide attempt over
the week. Aside from likely alcohol use disorder,
7 years ago.
Which of the following is the most
which of the following diagnoses is the most
likely
diagnosis?
appropriate at this time?
(A) Acute stress disorder
(B) Adjustment disorder (A) Bipolar
disorder with psychotic features (B)
(C) Amotivation syndrome Major
depressive disorder with
(D) Major depressive disorder (MDD) psychotic
features
(E) Substance-induced mood disorder (C)
Schizoaffective disorder
40 2: Adult Psychopathology (D)
Schizophrenia
(E)
Schizophreniform disorder
35. A 37-year-old woman, who works the night shift at
a local grocery store taking inventory, reports that 38. A 42-year-old
business executive presents for his
her childhood and college years were uneventful first contact
with a mental health provider. He
but happy. She spends most of her time alone reports that
for the last 4 months he has been
when she is not at work. She does not venture out feeling
depressed. His low-energy level and poor
of her house and her social contacts are limited to motivation are
affecting his job performance and
work-related interactions with coworkers. She is the CEO of his
company advised him to “take a
an avid plant lover, and she spends most of her couple of
weeks off.” The patient reports that he
free time taking care of her indoor nursery. She started
feeling down when his wife discovered
reports that she is quite content with her life. that he was
involved in his third extramarital
Which of the following is the most likely diag affair. Since
then he has moved into a small
nosis for this patient? apartment by
himself. He is sleeping almost 12
(A) Agoraphobia hours every
night, has a poor appetite, and is
experienc ing
financial difficulty due to
(B) Autism spectrum disorder
indiscriminate
purchases. He laments the loss of
(C) Avoidant personality disorder his former
self. He reports that he used to have
(D) Schizoid personality disorder several
periods of time per year, lasting for up to
(E) Schizotypal personality disorder 1 week, when
he only needed 4 to 5 hours of
sleep, had
large amounts of energy, spoke quickly,
36. An 18-year-old, pregnant, human immuno deficiency and could
“party all night and work all day.”
virus (HIV)–positive woman pres ents for the Which of the
following diagnoses is most likely in
treatment of opiate use disorder. She reports using this case?
psychiatry clinic by her
husband. She denies any thing is
(A) Bipolar I disorder
wrong, but he is
concerned that for the past year or so
(B) Bipolar II disorder she has been less able
to recall things she reads about in
(C) Borderline personality disorder her magazines. She used
to be in charge of grocery
(D) Major depressive disorder shopping, but over the
past year or so she has been
(E) Narcissistic personality disorder leaving things off the
list, and her husband has taken
over the job since the
patient can’t seem to organize it
39. A 52-year-old woman who has been treated with anymore. She is no longer able
to keep track of the
medication for 3 years for a chronic mood couple’s finances, and there have
been several occasions
disorder reports dry mouth, trouble urinating, and when her son found her
wandering outside, a number of
occasional dizziness when she gets out of bed. houses away. She is
otherwise healthy, and takes a
Which of the following medications is she most multivitamin daily.
Her physical examination is
likely being prescribed? unremarkable. Mental
status examination reveals a
healthy appear ing woman
who is cooperative with your
(A) Divalproex sodium questions,
(B) Fluoxetine
Questions: 35–45 41
(C) Imipramine
(D) Lithium and laughs when she
cannot recall any of the three items
(E) Phenelzine you ask her to
repeat. Her attention is intact. She has
some difficulties
naming objects. She states her mood as
“good” and her affect
is euthymic and full range.
Questions 40 and 41
42. Which of the
following is the most appropriate
A 72-year-old man is brought in by his wife to your provisional
diagnosis?
geriatric psychiatry clinic. The patient’s wife is con
(A) Delirium
cerned about his progressive confusion over the last
year. She is particularly distressed that he repeat edly (B) Major
depressive disorder
asks the same questions throughout the day. Her husband (C) Major
neurocognitive disorder
has become increasingly unsteady on his feet and needs (D) Mild
neurocognitive disorder
to use a walker when they go out. She wonders if these (E) Normal
pressure hydrocephalus
symptoms may be related to the meningitis he suffered
from 3 years ago. 43. A workup is
performed. Her laboratory val ues are
unremarkable,
and a CT scan of her brain
40. Which of the following symptoms or signs would demonstrates
diffuse cortical atrophy and normal
most likely be found in this patient? ventricles.
Which of the following diagnoses is
(A) Elevated opening pressure upon lumbar most likely?
puncture (A) Major
neurocognitive disorder due to
(B) Frontal release signs
Alzheimer disease
(C) History of incontinence (B) Major
neurocognitive disorder due to
(D) Oculomotor difficulties
Lewy body disease
(E) Perseveration (C) Major
neurocognitive disorder due to

traumatic brain injury


41. Which of the following would most likely be found (D) Major
neurocognitive disorder due to
on neuroimaging with computed tomography vascular
disease
(CT) in this patient? (E)
Unspecified major neurocognitive
(A) Cerebellar atrophy disorder
(B) Dilated lateral ventricles
44. If the
patient’s CT scan revealed a preponder ance of
(C) Focal subcortical hypointensities atrophy in the
frontal and temporal regions, which of the
(D) Frontoparietal atrophy following
behavioral manifestations would be most
(E) Frontotemporal atrophy
likely?
(A) Decline
in object naming
Questions 42 through 44
(B) Memory
problems
A 69-year-old woman is brought to your geriatric (C) Poor
organization
(D) Social disinhibition repeated and
intrusive thoughts of the accident,
(E) Word finding difficulty and has
recurrent nightmares. Lately, she has
changed her
commute to avoid the train, even
45. A 48-year-old man has been drinking up to 6 beers though this adds
3 hours to her commute daily.
per night during the week and up to 12 beers a When on the
train she has an acute increase in her
night on the weekend. A year ago, he had his anxiety. She
also often
driver ’s license suspended for drunk driving. His
marriage is failing because of these difficulties.
Last month, he was diag becomes
“jumpy” whenever she hears the train
nosed with a gastric ulcer as a result of alcohol going by her
home. Which of the follow ing
42 2: Adult Psychopathology diagnoses is
the most appropriate for this patient?
(A) Acute
stress disorder
consumption. He admits to an alcohol prob lem (B)
Adjustment disorder
and has tried to stop on numerous occa sions. He
(C)
Generalized anxiety disorder (GAD) (D)
finds that he experiences insomnia if he does not
drink for more than 2 days. Which of the Posttraumatic
stress disorder (PTSD) (E)
following features of this case suggests a severe Major
depressive disorder (MDD)
versus moderate alcohol use disorder?
Questions 48 and 49
(A) High quantity of alcohol consumed on a
regular basis A 40-year-old woman
with a 20-year history of
(B) History of legal problems schizophrenia
presents to the psychiatric emergency
department after a
suicide attempt by carbon mon oxide
(C) Inability to stop drinking despite
poisoning. She has
ongoing command hal lucinations to
knowing the harmful effects
harm herself, and
has acted on them at least 10 times
(D) Marital conflict due to drinking since her initial
diagnosis. She also has persistent
(E) Number of difficulties he is delusions that she
is responsible for world disasters,
experiencing which is why she
must eliminate herself as the source of
pain and suffering
in the world. She has been tried on
46. A 26-year-old computer programmer without both typical and
atypi cal antipsychotics, though none
previous psychiatric history has been mar ried for have been effective
in fully eliminating her psychotic
4 years. His wife is expecting their first child. She symptoms. Her level
of functioning remains poor, and
reports that 3 months ago the patient became she presently lives
in a group home.
preoccupied with the idea that she became
pregnant by another man. During this time, he 48. Which of the
following is this patient’s most likely
began missing work and isolated himself in his
lifetime risk of suicide?
bedroom. His affect has progressively become
more blunted. Recently, he believes that his wife (A) 1%
is carrying a child conceived by extraterrestrial (B) 5%
forces. He urged her to have an abortion and she (C) 10%
refused. The patient denies any history of (D) 30%
significant alcohol or illicit substance use and his (E) 50%
recent medical evaluation was within normal
limits. Which of the following is the most 49. Which of the
following medications would be the
appropriate diagnosis? most
appropriate to prescribe for this patient?
(A) Brief psychotic disorder (A) Clozapine
(B) Delusional disorder (B)
Fluphenazine
(C) Major depressive disorder with (C)
Haloperidol
psychotic features
(D) Lithium
(D) Schizophreniform disorder
(E)
Ziprasidone
(E) Schizophrenia
50. A 27-year-old
internal medicine resident gen erally
47. A 27-year-old woman was involved in a train dislikes working in
an outpatient clinic. Today, however,
derailment 2 weeks ago. Since that event, she has
he is looking forward to his
felt down, has not slept well, has expe rienced
Dialectical behavioral therapy (DBT) (C)
Group
therapy
clinical work because one of his appointments is a (D)
Psychoanalysis
follow-up visit for a single, attractive 31-year-old
(E)
Psychodynamic therapy
woman who is finishing her anti biotic regimen for
treatment of pneumonia. Which of the following
Questions 54
and 55
terms bests describes this doctor ’s response to his
patient? A 34-year-old
woman presents for the treatment of her
severe,
medication-refractory, major depressive disorder.
(A) Countertransference
After reviewing
her past psychiatric his tory and
(B) Empathy interviewing
the patient, you conclude that she would be
(C) Identification appropriate for
ECT.
(D) Projection
(E) Transference 54. In
discussing the effects of ECT with the patient,
which of
the following should you tell her is the
Questions 51 through 53 most
likely side effect?

A 31-year-old woman was admitted to a psychiatric unit (A)


Amnesia
after attempting suicide by overdose. She had recently (B)
Aspiration
broken up with her boyfriend of 4 months. She also (C)
Cardiac arrhythmias
describes episodes of mood lability, marked by feelings (D)
Convulsion fractures
of depression and anger directed toward the psychiatric
(E)
Psychosis
resident who completed the rota
tion 5 days after her admission. When the resident left,
55. For the
best possible outcome, how many treatments
she reported that she was having urges to cut her wrists. is
this patient likely to require?
She has had numerous admissions for suicidal gestures
and a lifelong history of tumultu (A) 2
ous romantic relationships. As the patient nears her (B) 4
discharge date, she reports that “all the staff hates me (C) 10
except for Dr. Johnson.” Dr. Johnson, a medical student,
(D) 15
had a recent difference of opinion with the nursing staff
(E) 20
regarding the patient’s discharge.
Questions 56
and 57
51. Which of the following diagnoses is the most likely
for this patient? A 38-year-old
woman presents to your clinic telling you
(A) Borderline personality disorder that she has
had disturbing, recurrent thoughts about
harming her 7-
month-old infant. She imag ines using a
(B) Cyclothymic disorder
knife to stab
her child, but she has no desire to hurt her
(C) Histrionic personality disorder child. As a
result of having these distressing thoughts,
(D) Major depressive disorder (MDD) she has removed
all sharp objects from her kitchen.
(E) Schizoaffective disorder Because of
this, she has not been able to prepare meals
at home and has
chosen to buy fast food or take out for
52. Which of the following would be the most the family
meals. She feels very anxious regarding these,
appropriate outpatient treatment for this patient? although she
has not shared these thoughts with her
husband.
(A) Antidepressants 44 2: Adult
Psychopathology
(B) Benzodiazepines
(C) Group psychotherapy 56. Which of the
following diagnoses would be the most
(D) Individual psychotherapy
likely for this patient?
(E) Mood stabilizers
(A)
Delusional disorder
Questions: 46–55 43
(B)
Obsessive-compulsive disorder
53. In this patient, which type of psychotherapy would (C)
Obsessive-compulsive personality
be the most efficacious?
disorder
(D)
Schizophrenia
(A) Cognitive-behavioral therapy (CBT) (B)
(E) Schizotypal personality disorder
60. Which of the
following would be the most
57. Which of the following would be the most appropriate
long-term treatment of this patient?
appropriate first-line pharmacotherapy for this (A)
Antibiotic therapy
condition?
(B) Chest x-
ray every 6 months
(A) Lithium (C) CT head
every 6 months
(B) Lorazepam (D)
Indwelling Foley catheter
(C) Fluvoxamine (E)
Intravenous fluids
(D) Haloperidol
(E) Nortriptyline 61. Which of the
following is the most likely 6-month
mortality of the
diagnosis in Ques tion 58?
Questions 58 through 61 (A) 5%
A 72-year-old woman is brought to the emergency (B) 15%
department from a nursing home for poor oral intake. (C) 20%
She is afebrile, has a pulse of 95, and a blood pressure of (D) 35%
90/ 60. Mental status examination (MSE) reveals an
(E) 45%
awake and alert, but frail, malnourished and dehydrated
woman who is oriented to per
Questions 62 and 63
son only. She reports that the president is George Bush.
She is easily distracted and cannot recall any of three You are a research
psychiatrist conducting a double
items after a few minutes. She is irritable and swings at blind, placebo-
controlled trial of a new antidepres sant.
the staff when they try to insert an IV. The team starts IV You have enrolled 200
patients in the study, all of whom
fluids as blood and urine are sent to the laboratory. A meet the criteria for
uncomplicated major depression.
chest x-ray is unremarkable, as is the head CT. One hour You plan to randomize
100 patients to a placebo
later, she is calmer and reports the correct day, time, and medication and the
other 100 patients to the
place; she is less distractible. experimental
antidepressant.

58. Labs are remarkable for leukocytosis and dirty 62. Of the 100
patients taking the placebo,
urinalysis. Aside from cystitis, which of the approximately
how many patients would be
following is the most appropriate diagnosis for the expected to
improve after 6 weeks?
patient?
(A) 5
(A) Delirium (B) 10
(B) Major depressive disorder (C) 30
(C) Major neurocognitive disorder due to
Alzheimer disease
(D) Substance-induced neurocognitive (D) 50
disorder (E) 70
(E) Unspecified neurocognitive disorder
63. Of the 100
patients taking the experimental
antidepressant
(assuming this drug is as effi
59. The patient is subsequently admitted, but tries to hit cacious as
standard antidepressants), approxi
several staff as they tend to her; she also mately how many
patients would be expected to
repeatedly tries to get out of bed and demands to improve after 6
weeks?
be let go. Which of the following would be the (A) 10
most appropriate immediate pharmaco
(B) 30
logic management for this patient?
(C) 50
(A) Diphenhydramine (D) 60
(B) Haloperidol (E) 80
(C) Lorazepam
(D) Phenobarbital Questions 64 and 65
(E) Valproic acid
Your patient is a 38-
year-old divorced Catholic male
with a month long history of depressed mood, Oddly enough, she
reports, this can happen when she
anhedonia, initial insomnia, low energy, and poor becomes
particularly stressed out, for example, if she is
appetite. He admits to suicidal ideation with a plan to managing many
calls. The patient also states that this
overdose for the past several days, and he has gathered disturbance has not
improved despite her sleeping 8
pills this morning. Upon further his hours each night.
tory, he admits to similar symptoms 5 years prior, also
with a prior suicide attempt by overdosing. He drinks 1 67. Which of the
following is the most likely diagnosis?
beer weekly and denies illicit drugs. He is on no other
(A)
Circadian rhythm sleep–wake disorder (B)
medications except for a multivitamin. He is
subsequently admitted after being medically cleared.
Hypersomnolence disorder
(C)
Insomnia disorder
64. What percentage of patients with this illness (D)
Narcolepsy
eventually commit suicide? (E)
Nightmare disorder
(A) 1%
68. Which of the
following is the most appropri ate
(B) 5%

pharmacotherapy for this condition?


(C) 10%
(D) 20% (A)
Bupropion (Wellbutrin)
(E) 30% (B)
Fluoxetine
(C)
Lorazepam
65. Which of the following characteristics of this patient (D)
Methylphenidate (Ritalin)
is the number one predictor of a future completed (E)
Phenelzine
suicide? 46 2: Adult
Psychopathology

(A) Age
69. A 28-year-
old woman presents for her annual
(B) Gender
gynecology
appointment. She complains that in
(C) Previous attempt the week
before her period, she often experiences
(D) Relationship status marked anger
and irritability and argues more
(E) Religion with her
boyfriend. She also reports diminished
Questions: 56–68 45 energy and
concentration, and is sleeping more
than is
usual for her. These symptoms, in addition
66. A 32-year-old single successful Wall Street to breast
tender ness and headaches, always remit
executive tells you that on weekends he likes to visit a in the week
after her menses is finished. Which of
dominatrix. His regular, paid appoint ment with this the
following is the most likely diagnosis?
person is described as humili ating and somewhat painful
(A) Major
depressive disorder
but also very sexually arousing. While he admits that
this behavior “may be weird to some people,” he enjoys (B) No
diagnosis
it, can afford it, and it doesn’t interfere with his work or (C)
Persistent depressive disorder
other hobbies. Which of the following diagnoses is the (D)
Premenstrual dysphoric disorder
most appropriate? (PMDD)
(A) Fetishistic disorder (E)
Premenstrual syndrome (PMS)
(B) Frotteuristic disorder
70. A 37-year-old
accountant presents to the pri mary
(C) No diagnosis care clinic with
complaints of insom nia. He
(D) Sexual masochism disorder admits to feeling
“blue” for 6 weeks since getting
(E) Sexual sadism disorder passed over for
promotion. Since that time, he has
had poor sleep,
often awakening early in the
Questions 67 and 68 morning. He also
has had a decreased appetite
with a 15-lb
weight loss, poor energy, guilt over
A 28-year-old woman complains of falling asleep during “not being good
enough,” and he has been
the day. This problem has been occurring for 3 months distracted at
work. He admits to passive suicidal
and is now interfering with her work as a telephone thoughts with out
a plan. Which of the following
operator as she falls asleep two or three times a day would be the
likely course of this patient’s illness
while speaking with customers. At times, she finds if not treated?
herself falling asleep at her desk, and she is awakened
when her head hits the computer con sole in front of her. (A) 3 to 6
months
(B) 3 to 7 months (C) Lithium
(C) 6 to 13 months (D)
Serotonin-specific reuptake inhibitor
(D) 9 to 15 months (SSRI)
(E) 12 to 18 months (E) Valproic
acid

Questions 71 and 72 74. A 40-


year-old married woman is referred by her
internist to
a psychologist for further treat ment. She
A 36-year-old man is brought to the emergency presents
an 8-month history of recur rent bouts of
department in respiratory arrest. On examination, he is “terror,”
associated with chest pain, tachypnea, tremors,
bradycardic and unresponsive, with constricted pupils flushing,
nausea, and fears of impending doom. These
bilaterally. There are no other obvious inju
episodes
ries on the patient, but no one is immediately avail able
to provide collateral history. There is suspicion that the
patient’s condition may be the result of an overdose. last for
approximately 15 minutes and do not have
a particular
trigger. As a result, she has had
increasing
difficulty traveling far from her home
71. Which of the following drugs is most likely to cause due to concerns
over having further attacks in
this presentation? public. Despite
adequate treatment with sertraline,
(A) Alcohol she remains
symptomatic and in significant
distress. Which
of the following psychotherapies
(B) Cocaine
would be the
most appropriate for her condition?
(C) Heroin
(D) Inhalants (A)
Cognitive-behavioral therapy (CBT)
(B) Eye
movement desensitization and
(E) Phencyclidine (PCP)

reprocessing (EMDR)
72. Which of the following would be most impor tant to (C) Insight-
oriented therapy
administer immediately? (D)
Interpersonal psychotherapy
(A) Acetylcysteine (E) Supportive
psychotherapy
(B) Deferoxamine
Questions 75 and 76
(C) Methadone
(D) Methylene blue You are treating a 48-
year-old married female on the
inpatient medical unit
for pyelonephritis; she has
(E) Naloxone
responded well to
appropriate antibiotic ther apy and has
been afebrile for the
last 24 hours. You inform her of
73. A 30-year-old male veteran from Operation Iraqi
likely discharge if she
continues to improve. The next
Freedom/ Operation Enduring Freedom presents
morning, however, she
com plains of feeling feverish and
to the mental health clinic at the urg ing of his
achy, and having dys
uria again. The nursing staff reports
wife. While stationed in Iraq on his third
that she has a sudden
fever of 103°F. You treat the fever
deployment, his unit was hit by a roadside bomb.
with acet aminophen and
perform a physical
He saw a fellow Marine killed, while he and
examination, order
chest x-rays, draw blood, and order a
several other members of the command sustained
uri nalysis with
culture. While you are awaiting these
nonlethal injuries. Since that time, he has had
results, the nurse
informs you that she witnessed the
chronic insomnia with ongoing night
patient dipping her
thermometer into a hot cup of tea
mares of the event as well as occasional flash
before her temperature
was taken.
backs. He describes always feeling “on edge,”
avoiding crowds, and becoming easily startled
75. Which of the
following diagnoses most likely
with loud noises. He admits to regular alcohol use,
accounts
for this woman’s behavior?
especially when his symptoms are worse. He feels
detached from events with his fam ily, preferring (A) Factitious
disorder
to isolate himself. Which of the following is the (B) Functional
neurological symptom
most appropriate treatment to begin for this disorder
patient?
(C) Illness
anxiety disorder
(A) Atypical antipsychotic (D) Malingering
(B) Benzodiazepine (E) Somatic
symptom disorder
left
middle cerebral artery stroke. Which
76. Which of the following is the most likely moti vator 48 2: Adult
Psychopathology
of this patient’s behavior?
of the
following psychiatric symptoms would be
(A) Conscious desire to assume sick role
the most
likely as a result?
(B) Conscious desire to avoid work
Questions: 69–79 47 (A) Anxiety
(B)
Depression
(C) Unconscious desire to assume sick role (D) (C) Mania
Unconscious desire to avoid work (D)
Obsessions
(E) Unconscious conflict producing (E) Panic
attacks
symptoms
80. A 45-year-old
woman with no previous psy chiatric
Questions 77 and 78 history is admitted to
neurology for the treatment of an
acute multiple
sclerosis flare. She does not smoke, drink
A 32-year-old man is brought to the psychiatric alcohol, or use other illicit
drugs. She is started on appro
emergency department by the police after having been priate therapy and by the
third day her initial lower
arrested for public nudity. On mental status examination, extremity weakness has
resolved. How ever, she also
the patient cannot sit down and is only partly begins to act strangely,
and asks you to change her room
cooperative. He interrupts the inter to prevent the “nin
jas outside from creeping in and
view several times demanding to contact his law yer, stealing my soul.” She also
tells you the nurses have
“because my rights given to me by God and ordained by been poisoning her medicine.
Which of the follow ing
the Jeffersonians have been infringed.” His sister diagnoses
would be the most likely?
indicates that this patient has been treated for at least
two episodes of major depression in the past, one of (A) Adjustment
disorder
which resulted in a suicide attempt. He is currently a (B) Bipolar
disorder
computer programmer, but has been increasingly (C) Psychotic
disorder due to multiple
stressed at work. One month ago, his girlfriend broke up sclerosis
with him, and since then he has been increasingly (D)
Schizophrenia
irritable. Over the past 2 weeks he has maxed out his
(E)
Substance-induced psychotic disorder
credit card from online gambling sites, which he has
continued to visit over the past four consecutive nights.
81. A 42-year-old man
presents with a history of
Laboratory tests show a negative drug screen and
recurrent
episodes of major depression, with one
alcohol levels.
prior
hospitalization. For the past several months
he has felt
increasingly depressed, with insomnia,
77. Which of the following is the most likely diagnosis?
low appetite,
little energy, cry
(A) Adjustment disorder ing spells, and
poor concentration. He is not
(B) Bipolar disorder taking any
current medications. He states that he
(C) Brief psychotic disorder has been on
fluoxetine, paroxetine, sertra line, and
venlafaxine in
the past, and, while they’ve all
(D) Cyclothymic disorder
been
efficacious, he has stopped them due to
(E) Major depressive disorder (MDD) significant
erectile dysfunction. Which of the
following
medications would be the most
78. Which of the following would be the most appropriate
to prescribe?
appropriate pharmacologic treatment for this
patient? (A)
Aripiprazole
(B)
Bupropion
(A) Carbamazepine
(C)
Citalopram
(B) Haloperidol
(D) Lithium
(C) Lamotrigine
(E)
Sertraline
(D) Lithium
(E) Sertraline
82. A
33-year-old male with a history of frequent
79. A 68-year-old man without prior psychiatric history, fighting,
aggression, impulsivity, and sui cide attempts is
but with a history of hypertension, referred to a
research facility. He receives multiple tests
hyperlipidemia, and arthritis is admitted for a new
and studies. Upon cerebrospinal fluid (CSF) analysis, months. She
denies any problems with mood. She
decreased metabolites of which of the following neu denies any
recent stressors, changes to her routine,
rotransmitters would most likely be seen as compared to or changes to
her husband’s routine. Which of the
individuals without his problems? following is the
most likely diagnosis?
(A) Dopamine (A) Adjustment
disorder
(B) Gamma-aminobutyric acid (GABA) (B) Generalized
anxiety disorder
(C) Glutamate (C) Obsessive-
compulsive disorder
(D) Norepinephrine (D) Panic
disorder
(E) Serotonin (E) Social
anxiety disorder (social phobia)

83. A 38-year-old woman with multiple sclerosis of 86. You are a research
psychiatrist who is studying
moderate severity has had symptoms of signs and symptoms
associated with certain
depression and memory loss increasing over the psychiatric disorders, and
notice a category of
last year. On mental status examination, you patients who have sensory
gating deficits,
notice a blunted affect and decreased speed of short-term memory
difficulties, and abnor
mental processing. A magnetic reso malities in smooth-pursuit
eye movements. Which
nance imaging (MRI) examination is most likely of the following diagnoses
is most likely to be
to reveal which of the following? found in this patient
population?
(A) Global cerebral atrophy (A) Attention-
deficit/ hyperactivity disorder
(B) Multiple plaques in frontal white matter (C) (ADHD)
Normal brain (B) Major
depressive disorder (MDD)
(D) Periventricular lacunar infarcts (C) Obsessive-
compulsive disorder (OCD) (D)
(E) Ventricular enlargement Posttraumatic
stress disorder (PTSD) (E)
Schizophrenia
84. An 80-year-old widowed woman is admitted to the
hospital for “confusion.” On examina tion she is 87. A 23-year-old
single medical student comes to your
somnolent, though earlier in the morning she had office complaining
of difficulty sleep ing, excessive
been alert and aware. She is uncooperative, worry about his studies,
his relationship with his parents,
hostile, and hallucinating. Her insight and and that his girl
friend of 2 years is going to break up
memory are poor. The pri mary team wishes to with him despite being
happy with their relationship. He
know if she is “delirious or demented.” Which of feels irritable
at times, and has significant muscle
the following signs/ symptoms in this patient is the tension. Because of
these symptoms, his grades have
most specific for delirium? suffered. He does
not report a depressed mood. You
diagnose him with
generalized anxiety disorder, and
(A) Combativeness
prescribe a
combination of psychotherapy and a
(B) Fluctuating consciousness selective
serotonin reuptake inhibitor (SSRI). Three
(C) Poor memory months
later, the patient comes back to your
(D) Psychosis
Questions: 80–88 49

(E) Uncooperativeness
office
reporting that his mood has been down in
85. A happily married graduate student comes to your the dumps, and
he feels like he may never feel
clinic with complaints of insomnia. She has been better.
Recently, he has been thinking that life was
unable to fall asleep because not worth
living anymore and has passive
thoughts of
suicide. He reports ongo
ing thoughts of
overdosing on pills. Which of the
she ruminates about grades, money, her rela following would
be the most appropriate next
tionship, and her young children. She has always step?
been concerned about these, but lately her worries (A) Call his
parents and arrange a family
have gotten out of control. She is unable to relax meeting
and feels constant tension in her muscles. While
(B)
Discontinue his SSRI and have him
she denies symptoms of panic attacks, she has
return
in 1 week
noticed an increase in headaches and
gastrointestinal disturbances over the past few (C) Refer
the patient to the psychiatric

emergency department
(D) Schedule weekly outpatient visits (A)
Amitriptyline
(E) Switch the SSRI and have him follow-up with (B) Clonazepam
you in 1 month
(C) Fluoxetine
88. A 36-year-old married white female presents to the (D) Lithium
emergency room with a 2-month his tory of (E) Sertraline
depression, terminal insomnia, fatigue, decreased
appetite, anhedonia, and exces sive guilt. She feels 91. A 62-year-old
male patient with schizophrenia is
hopeless and reluctantly admits to suicidal brought into the
emergency department by the
ideation for the past week, with thoughts of police for
trespassing. Upon interview, he tells the
“taking all of my medicines.” After further physician that
over the past 3 weeks his television
questioning, she states that “I would never do it” has been giving
him coded messages. Which of
as she is a devout Catho lic who attends church the following
terms best describes this
regularly. Which of the following characteristics phenomenon?
most increases this particular patient’s risk of (A)
Hallucination
suicide?
(B) Idea of
reference
(A) Age (C) Illusion
(B) Gender (D) Thought
broadcasting
(C) Hopelessness (E) Thought
insertion
(D) Marital status
(E) Religion 92. A 45-year-old
patient tells her doctor that after
hearing that her
husband died, she could not
Questions 89 and 90 remember leaving
her office and going home. In
every other
respect, her memory is intact. Which
A 29-year-old married white woman with a past medical of the following
types of amnesia is this an
history of recurrent migraines is brought to the example of?
psychiatric emergency department by her hus band who
reports that, despite feeling depressed 2 months ago and (A) Continuous
being compliant with prescribed treatment, she now has (B) Generalized
been acting bizarre for sev eral days. On initial interview, (C) Localized
the patient states, “I feel superbly supreme, and you have
no idea what an amazing person I am!” The patient is
talking so rapidly that you cannot interrupt her. Her (D)
Retrograde
husband (E) Selective
50 2: Adult Psychopathology

Questions 93 and 94
reports that the patient has not slept in over a week, and
during the same time period, she has put a down A 75-year-old man
comes to your office complain ing of
payment for a car, has purchased a diamond tennis poor sleep since his
wife’s death 1 month ago. Since that
bracelet, and has booked an extravagant vacation. time, he has been
unable to fall asleep, and he has felt
“down.” He is
slightly more isolative now, as many of
89. Which of the following is the most likely diagnosis? the activities he
enjoyed doing were with his wife. His
appetite is
decreased, but he is still bathing and cooking.
(A) Anxiety disorder He sometimes feels
guilty that she died before him, and
(B) Bipolar and related disorder is angry with God
that he is alive with no “soul mate”
(C) Neurocognitive disorder anymore. He
sometimes hears the voice of his deceased
(D) Psychotic disorder wife encouraging him
to move on, and he denies suicidal
(E) Somatic symptom disorder ideation.

90. You suspect a medication is the cause for her current 93. Which of the
following is the most appropriate
condition, but neither she nor her hus band recall diagnosis at
this time?
the medication prescribed 2 months ago. Which of (A)
Adjustment disorder with depressed
the following medications is the most likely mood
etiology? (B)
Bereavement
(C) Major depressive disorder (C) Hypersomnia
(D) Persistent complex bereavement (D) Hypnagogic
hallucinations
disorder (E) Sleep paralysis
(E) Schizophrenia
98. A 21-year-old man
with newly diagnosed
94. Which of the following would be the most schizophrenia has
been compliant with his
appropriate treatment for this patient? medications and is
less psychotic. At his next
follow-up
appointment, he is noted to be rest
(A) Hospitalize the patient immediately (B)
less and constantly
moving. He states that he feels
Prescribe an antidepressant as if he has to be
moving all the time and is
(C) Prescribe an antipsychotic uncomfortable if he
sits still. Which of the
(D) Refer for brief supportive therapy following side
effects is he most likely
(E) Refer for cognitive-behavioral therapy experiencing?
(A) Akathisia
95. When asked about his level of education, a
(B) Akinesia
48-year-old man with a history of schizoaf fective
disorder, depressed type, spends the next 5 (C) Dystonia
minutes describing his high school building, (D) Rabbit syndrome
friends he had at the time, clubs he joined, and (E) Tardive
dyskinesia
eventually his high school gradua tion ceremony.
Questions: 89–100 51
He concludes by saying, “And that was the end of
my schooling.” Which of the following terms does 99. A 21-year-old
woman complains of recur rent
this response best demonstrate? episodes where she
has extreme anxiety, along
with palpitations,
dizziness, nausea, abdominal
(A) Circumstantiality
cramps, diarrhea,
tingling of her fingers, shortness
(B) Loosening of associations of breath, and
fearing she may die. These periods
last under 30
minutes, and they’ve been increasing
in frequency. She
is unable to predict when they
(C) Perseveration will occur, but,
as a result, she is often worried
(D) Pressured speech about hav
(E) Tangentiality ing another
attack, and she has been unwill ing to
leave her
apartment for several weeks. She drinks
96. In her psychiatrist’s office, a patient suddenly 1 to 2 drinks of
alcohol per month and denies
lowers herself to the floor, begins flailing about illicit drug use.
She has no medi cal problems, is
wildly, then flings a garbage pail against the wall, only on oral
contraceptives, and her physical and
and runs out of the office. Immedi laboratory workup
are negative. Which of the
ately afterward, she returns. She is alert and following would be
the most appropriate
oriented, yet does not remember the incident. treatment?
Which of the following types of seizures is the
(A) Buspirone
most likely?
(B) Lithium
(A) Complex partial seizure (C) Lorazepam
(B) Jacksonian seizure (D) Valproic
acid
(C) Nonepileptic seizure (E) Venlafaxine
(D) Temporal lobe seizure
(E) Tonic–clonic seizure Questions 100 and 101

97. A 26-year-old man newly diagnosed with A 46-year-old man with


a history of hypertension and
narcolepsy explains that he has episodes of falling hyperlipidemia presents
to his primary care doc tor with
down without any loss of conscious ness, complaints that “I’m
drinking too much.” He gives a
precipitated by laughter or anger. Which of the long history of binges
over 4 to 5 days at a time, where
following terms best describes this symptom? he will drink 1 to 2
pints of gin per day. He has had
several DUIs and is at
risk of losing his job. In addition,
(A) Catalepsy
his drinking has put a
strain on his marriage.
(B) Cataplexy Unfortunately he has
been unable to maintain sobriety
for any significant period of time. He is “stressed out” and enjoys watching his
favorite movies on the
because of the consequences of his drinking, and he television. His wife
describes extreme mood lability,
admits to chronic, intermittent insomnia. He has a good where he will be laughing
and then crying. He denies
appetite, however, and his energy is adequate unless he suicidal ideation. He
doesn’t drink alco hol and denies
is significantly hun drug use. He was a
prior smoker but quit 20 years ago.
gover. He feels helpless but denies suicidal ideation. His physical
examination and labs are unremarkable.

100. Which of the following medications would be most 103. As part of his
workup, an MRI of his brain is
appropriate to prescribe for this patient? ordered. What would
be the most likely finding?
(A) Acamprosate
(B) Bupropion (A) Atrophy of
brainstem and cerebellum (B)
(C) Citalopram Atrophy of
frontal and temporal lobes (C)
(D) Lithium Enlarged
ventricles
(E) Olanzapine (D) Global
atrophy
52 2: Adult Psychopathology
(E)
Periventricular white-matter lesions
101. Two weeks later this same patient is seen in the
104. Which of the
following would best describe his
emergency room after being picked up by the
clinical
course?
police for disturbing the peace and wandering. On
examination he appears confused, has nystagmus, (A) Continued
improvement
and an unsteady gait. Which of the following (B) No
improvement or worsening
should be admin (C) Rapid
decline
istered immediately?
(D) Steady
decline
(A) Glucose (E) Step-wise
deterioration
(B) Haloperidol
(C) Lorazepam 105. An 86-year-old
woman with multiple medical
problems and a
recent hip fracture is admitted to
(D) Naltrexone
the intensive
care unit. While in the unit, she
(E) Thiamine
awakens at
night and mistakes her intrave
nous (IV) pole
for a family member coming for a
102. A 36-year-old man presents to the emergency
visit. She then
becomes agitated, attempting to
department after being found without cloth ing in the
pull out her IV
and get out of the hospital bed.
street. He has multiple excoriations all over his body, is
Which of the
following approaches is the most
paranoid and agitated, and states that bugs are crawling
important in
the ultimate management of this
all over him. His vitals are as follows: T-99.1, BP-160/
patient?
93, P-105, R-20. Which of the following substances
would be most likely found in his laboratory results? (A)
Administer diphenhydramine
(A) Alcohol (B)
Administer haloperidol
(B) Cannabis (C)
Administer lorazepam
(C) Cocaine (D) Apply
soft restraints
(D) Opiates (E) Determine
and correct the underlying

condition
(E) PCP
106. A 24-
year-old graduate student in philoso phy is
Questions 103 and 104
referred by
his student health center for a psychiatric
evaluation.
Although he claims to have had similar but
A 72-year-old married male with a history of hyper
attenuated
symptoms in childhood, since beginning his
tension, borderline diabetes, hyperlipidemia, and
thesis, he describes
an acute worsening of fears that he w
coronary artery disease is brought in by his wife. Over
ill contract H
IV. While he understands the modes and
the past year he has become increasingly for getful,
risks of contrac
tion and practices safe sex, he is unable
misplacing his keys, getting lost while driv ing, and
to “get rid
of these thoughts.” As a result, he feels
starting to wander. When confronted with these
compelled to w ash
his hands many times per d ay, even
situations he becomes defensive, making up excuses for
to the point of
their becoming raw and bleeding. Despite
his behavior. His sleep is erratic, but he is eating well
his insight that
his concerns are irrational, he is not able
to stop the behaviors. A positron emission tomogra phy would
be most appropriate to prescribe in this
(PET) scan of this patient’s brain w ould case?
(A)
Amitriptyline
(B)
Clonazepam
most likely demonstrate increased activity in
which of the following structures? (C)
Diphenhydramine
(D)
Donepezil
(A) Amygdala
(E)
Haloperidol
(B) Caudate nucleus
(C) Cerebellum 109. A 36-
year-old woman presents with com plaints of
(D) Hippocampus a
depressed mood for the past month. She reports
(E) Parietal lobes poor
sleep, little appetite with weight loss, low
energy,
decreased con centration, and little libido.
Questions 107 and 108 She
admits to feel ing hopeless and suicidal,
although
she denies a specific plan or intent. She
A 70-year-old widowed man without significant medical is
subsequently begun on paroxetine 20 mg at
history is brought in by his daughter due to concerns bedtime.
Which of the following symptoms would
about his being able to adequately care for himself. He be most
likely to improve the earliest?
has been found lost in the downtown area on several
(A)
Decreased libido
occasions, attempting to walk into strangers’ houses, as
well. He is unable to cook for himself but is unable to (B)
Depressed mood
calculate the correct amount for purchases. His daughter (C)
Hopelessness
has noticed that he is dizzy when standing up and has a (D)
Poor sleep
wide-based, slow walk. Upon interview, he appears (E)
Suicidal ideation
disheveled and smells of urine. He is friendly and
cooperative overall, although easily distracted and 110. A 35-
year-old man complains to his therapist that
confused. While he knows his name, he believes the year his new
partner enjoys sexual activity only when
is 1989. When confronted about his memory difficul
inflicting pain on him. This dis turbs and frustrates
ties, he states that there are small gnomes living in his the
patient. Which of the following best describes
furniture, who play tricks on him by moving items the
behavior exhib ited by his partner?
around in his condominium. According to the daughter,
he rarely drinks alcohol, does not smoke tobacco, and (A)
Exhibitionistic disorder
has never used illicit drugs. (B)
Frotteuristic disorder
(C)
Sexual masochism disorder
107. Which of the following diagnoses is most likely? (D)
Sexual sadism disorder
(A) Major frontotemporal neurocognitive (E)
Transvestic disorder
disorder
(B) Major neurocognitive disorder due to 111. An
anxious 23-year-old Asian male univer sity
Alzheimer disease student
presents to student health ser vices

claiming that his penis is shrinking into his


(C) Major neurocognitive disorder with

abdomen. Despite reassurances from the staff and


Lewy bodies
the
physician, he remains convinced of this belief.
(D) Major vascular neurocognitive disorder (E) Which
of the following syn dromes is this patient
Substance-induced major most
likely suffering from?
neurocognitive disorder 54 2: Adult
Psychopathology

108. The above patient is appropriately diagnosed and (A)


Capgras syndrome
then placed in an intermediate care facil ity. Eight months (B) Koro
later he is brought by the daughter because the nursing
(C) Kuru
home is having increasing difficulty caring for him. He
is more confused, responding to internal stimuli, and (D)
Taijin-kyofusho
Questions: 101–111 53 (E) Zar

easily agitated, striking at nursing staff and his 112. A 21-year-


old male college student is evalu ated by
daughter. Which of the following medica tions his
college student health center after being
arrested
for masturbating outside of a sorority
window late at night. He admits to having medication he has
taken. While he denies intrusive
watched a particular female student inside the thoughts or overt
flashbacks, he continues to have severe
building over a period of several months. Which nightmares several
times per week which significantly
of the following is the most likely diagnosis? interfere in his
sleep. He remains “jumpy” at times, and
because of this
doesn’t attend functions where there are
(A) Exhibitionistic disorder
large crowds. He has
not used alcohol or drugs in 18
(B) Frotteuristic disorder years, and he denies
any suicidal ideation.
(C) Gender dysphoria
(D) Transvestic disorder 114. Which of the
following medications would be the
(E) Voyeuristic disorder most beneficial
to prescribe for augmentation?
(A) Alprazolam
113. A 45-year-old separated female is brought by her
(B) Bupropion
brother into the emergency room with the chief
complaint of “strange behavior.” She has been (C) Prazosin
living with her brother for the past 2 years, and (D) Risperidone
she stopped her medications 12 months ago. The (E) Valproic
acid
brother states that over the past 6 months she has
become increasingly paranoid and delusional, 115. In addition to
medications, this patient wishes to
believing that the neighbors are spying on her. She pursue “talk
therapy.” Which of the fol lowing
insists that the Chinese government is using her as forms of
psychotherapy would be the most
an agent to combat terrorism, and that they com appropriate to
recommend?
municate their instructions via a wireless sig
(A) Behavior
therapy
nal transmitted directly to her brain. She is often
noticed to be talking when no one else is in her (B) Cognitive
processing therapy
room. In addition, over the past month she has (C) Cognitive
therapy
been staying up most of the night, pac (D) Dialectical
behavioral therapy
ing around the house and attempting to con struct (E)
Psychodynamic psychotherapy
an anti-terrorism machine, using parts of various
household electronics. She states that it is 116. A 23-
year-old graduate student comes to the
important that she alone completes her mission, psychiatric emergency
room complaining of anxiety. She
that she is the most senior spy in the organization has never been seen
by a psychia trist before and is not
with a top-secret clear ance “that only the
taking any medications.
President and I possess.” Her brother has had a
hard time calming her down as she speaks too
quickly to follow, goes from one topic to another, Her vital signs
are notable for a heart rate of 110
and “won’t sit still.” Which of the following beats/ min. She is
also slightly diaphoretic and has
diagnoses would be the most likely for this mildly dilated
pupils. Given her pres ent state,
patient? which area of this
patient’s brain would most
likely demonstrate
increased activity?

(A) Bipolar disorder (A) Amygdala


(B) Delusional disorder (B) Basal ganglia
(C) Major depressive disorder with (C) Hippocampus
psychotic features (D) Locus
ceruleus
(D) Schizoaffective disorder (E) Thalamus
(E) Schizophrenia
117. You are asked to
give a psychiatric consulta tion on
Questions 114 and 115 a 28-year-old
woman with systemic lupus
erythematosus who
was admitted to the medical
A 63-year-old Vietnam veteran with posttraumatic stress service. After you
see her, one of your medical
disorder (PTSD) presents to the outpatient mental health colleagues tells
you that she will no longer speak
clinic. He has been prescribed numer ous medications to any of them
because she “hates all of them” and
over the years, and he has recently been taking now insists on
seeing you because you are the
citalopram 40 mg daily. He denies side effects and “best doctor in
the hospital.” Which of the
strongly believes this has been the “most effective” following terms
best describes the patient’s
behavior? has a
group of peers but is often immature, and his
friends
often prevent him from being taken
(A) Acting out
advantage
of as he is very gullible. He takes care
(B) Externalization of his
physical needs but parents ensure that he
(C) Regression has
enough grocer
(D) Splitting ies and
need to remind him to check his bank
(E) Sublimation account
balance. Which of the following is the
most
appropriate diagnosis?
118. A 42-year-old woman with recurrent episodes of (A) Mild
intellectual disability
major depression is admitted to a medi cal unit
(B)
Moderate intellectual disability
after a car accident that rendered her unconscious.
The patient regains conscious ness after 3 days and (C)
No diagnosis (normal intellectual
corroborates that she was, indeed, on an
functioning)
antidepressant, but she says she also cannot (D)
Severe intellectual disability
remember which one. She is started on paroxetine (E)
Profound intellectual disability
(Paxil) for her depression. Two days after
beginning this medication, she develops DIRECTIONS
(Questions 121 through 145): The
tachycardia, diaphoresis, and myo clonic jerks. following group
of numbered items are preceded by a
The neurotransmitter most likely associated with list of lettered
options. For each question, select the
the above reaction is synthe sized in which of the one lettered
option that is most closely associated
following central nervous system structures? with it. Each
lettered option may be used once,
multiple times,
or not at all.
(A) Caudate nucleus
56 2: Adult
Psychopathology
(B) Locus ceruleus
(C) Nucleus accumbens Questions 121
through 130: Match each scenario
(D) Raphe nucleus with its most
likely description.
(E) Substantia nigra
Questions: 112–120 55
(A) Alcohol
intoxication
(B) Alcohol
withdrawal
119. A 26-year-old man with schizophrenia is being (C)
Amphetamine intoxication
discharged from his third admission to the (D)
Amphetamine withdrawal
psychiatric unit in 3 years. This most recent stay (E)
Caffeine intoxication
was prompted by acute exacerba (F)
Caffeine withdrawal
tion of command auditory hallucinations and
(G)
Cannabis intoxication
paranoia that resulted in bizarre behaviors such as
(H)
Cannabis withdrawal
lying in the middle of the road and berating
customers that cost him his restau (I) Cocaine
intoxication
rant job. His symptoms are improved with (J) Cocaine
withdrawal
pharmacotherapy, which he is tolerating, with (K)
Hallucinogen intoxication
mild side-effects of sedation that are gradually (L)
Inhalant intoxication
improving. Which of the following time peri
(M) Opioid
intoxication
ods most accurately describes when his risk of
(N) Opioid
withdrawal
suicide is the highest?
(O)
Phencyclidine intoxication
(A) At his first psychotic break
(B) During his first hospitalization 121. A 32-year-old
single, male with injected con
(C) In the week preceding this junctiva can’t
concentrate at work, laughs readily
hospitalization at his
coworkers’ doodles, feels “relaxed,” and
(D) In the week while he is hospitalized (E) In speaks slowly
while seemingly focused on the air
in front of
him.
the weeks following hospitalization

120. You are asked to review neuropsychological 122. A 45-year-old separated


male admitted for
testing for a 19-year-old patient who is fail ing depression and suicidal
ideation is irritable, asks
classes at his local community college. His results for extra food, and
spends most of the first day
indicate an intelligence quotient (IQ) of 55. He is sleeping. The chest pain
he had on admis
sion has subsided and he has no ECG changes. (it is daytime).
He is tachycardic, hypertensive, and

tremulous.
123. After recess, a junior high school student smells
“funny,” is stumbling, feels dizzy and nauseated, Questions 131 through
136: Match each scenario
yet remains smiling and says she feels “such a with the most likely
receptor responsible from the
rush.” By the middle of her next class she has a following list.
headache but otherwise feels like she did this
(A) 5-HT2 A
receptor
morning.
(B) Alpha
receptor
(C) Beta
receptor
124. A 17-year-old high school student attends a college
fraternity party hosted by his elder brother. After (D) Dopamine
receptor
several hours, he feels more courageous and (E)
Histaminergic receptor
approaches ladies he would normally be too shy to (F) Muscarinic
receptor
engage, his words are slightly slurred, and he has
difficulties mov
ing in a straight line. He is slightly flushed and 131. A 32-year-old man
on clozapine experiences
notes mild memory problems for events ear lier in constipation.
the night.
132. A 29-year-old
woman on quetiapine experi ences

dizziness on standing.
125. A 27-year-old graduate student woke up late and
skipped breakfast. She now has a massive
headache and is irritable when she walks into her 133. A 52-year-old man on
risperidone experiences a
first morning class. She feels like falling asleep milky discharge
from his breasts.
and as if she “has the flu” by the end of the
morning. 134. A 41-year-old man on
haloperidol experiences
resolution of his
auditory hallucinations and his
delusions.
126. A veterinarian technician is brought to the
emergency department after attacking what he
thought was a cougar (it was a housecat). In the 135. A 22-year-old woman on
olanzapine experi ences
emergency room he is febrile, appears panicked, significant
sedation and weight gain.
has nystagmus, and demonstrates unexpected
strength and rage. 136. A 53-year-old man
on quetiapine experiences
improvement and
stabilization of mood.
127. A 35-year-old pilot is brought in for evaluation
because he has not slept for days, and now he is Questions 137 through 145:
Match each scenario
anxious, tachycardic, tremulous, and unable to with its most likely defense
mechanism from the
give coherent history. His pupils are dilated and following list.
his blood pressure is high, despite normal values 2
weeks ago on a flight physical. (A) Altruism
(B) Denial
128. A 54-year-old woman with arthritis complains of (C) Displacement
yawning, diarrhea, abdominal cramps, and nausea. (D) Humor
Her pupils are dilated and she has notable (E) Projection
piloerection. (F) Projective
identification
(G) Reaction
formation
129. A 23-year-old man is found unresponsive with (H) Rationalization
slowed breathing. He has multiple scars on his
(I) Splitting
arms and he has severe miosis.
137. A 37-year-old
married male is just told by his wife
130. On the third postoperative day your 63-year old that she has been
having an affair. He immediately
patient becomes agitated, demands you remove the hugs her and
tells her he loves her.
snakes from his room, and asks why it is so loud at night
Questions: 121–145 57
138. Your patient is angry with you and claims you are 142. You argue
with the boss at work, and when you
the worst doctor ever, as opposed to her former come home
you harshly groom your cat so that
clinician, who actually listened to her. she actually
wriggles away out of your arms.

139. Your borderline patient misses seven appoint ments 143. A 68-year-old
widow volunteers at the local
in a row resulting in your terminating her veteran’s
nursing home.
treatment contract.
144. After
repeatedly failing to bring your por tion of the
140. After slipping in front of your boss on a fro zen group project
to class, you accuse the group leader
puddle, you exclaim that “in a former life I was actually of forgetting
to e-mail you a reminder.
an elite figure skater!”
145. After not
matching into your chosen specialty, you
141. Your 69-year-old patient states he just has a bad say that it
was full of boring nerds any way and
cold after being diagnosed with meta static lung that you
really have way too much personality to
cancer, and states all he needs is some hot tea and be part of
them.
rest.

Answers and Explanations


closely
associated for a prolonged period of time. The
Fregoli delusion is
a variation of the delusion of doubles,
and is the belief
that familiar people assume the guise of
strangers. While
major depressive disorder can manifest
with
psychotic features, the prominence of
psychosis
with bizarre (not possible) delusions
and
without clear depressed mood makes this
1. (B) The patient’s 3- to 4-year history of bizarre diagnosis
less likely. Further, though nihilis
behavior, delusions, and decline in social func tic themes
and negativism can be observed in
tioning strongly suggest that he has schizo phrenia.
depression, the delusions are usually nonbi zarre
The prevalence of schizophrenia in the general
(potentially feasible).
population is approximately 1%. Schizophrenia is
found in all societies and geographical areas 4. (A) The belief
that people have been replaced by
around the world, and its prevalence is roughly imposters is
the hallmark of Capgras syndrome.
equal in males and females. Delusional
disorder is character ized by the
presence of
(usually) nonbizarre d elusions. See
2. (D) In twin studies, schizophrenia’s monozy gotic explanations
to Question 3 for further definitions.
concordance is 40% to 50%, suggesting that there
is a strong genetic component to the illness. The 5. (A) This
woman’s presentation is consistent with
prevalence of schizophrenia in the following psychosis with
peripartum onset, char acterized by
populations is: 10% to 15% in the nontwin sibling delusions and
hallucinations dur ing pregnancy or within
of a patient with schizophre nia, 12% in the child 4 weeks of
delivery. She further exhibits insomnia,
of one parent with schizo phrenia, and 40% in the irritability,
and mood lability. This, coupled with
child of two parents with schizophrenia. previous early
depressive episodes, strongly suggests an
episodic mood
disorder. Though postpar tum psychosis
3. (B) Cotard syndrome describes nihilistic delu sional can be due to
different etiolo gies, most cases eventually
content; in addition to lost possessions, patients may feel manifest as
bipolar disorder. However, only a small
they have lost blood, heart, intestines, as well as believe fraction of women
with bipolar disorder will manifest
that the world beyond them has been reduced to nothing with postpartum
psychosis. A less frequent underlying
ness. This psychotic/ delusional theme can be seen in etiology of
postpartum psychosis is MDD. Primary
many psychotic illnesses. Capgras syn drome is a psychotic
illnesses, such as delusional disorder,
delusion of doubles characterized by the belief that schizoaffective
disorder, and schizophrenia, are even
people have been replaced by identically appearing
more rare.
imposters. Folie à deux, or shared psychotic disorder, is
when a similar delusion is aroused in one person by the 6. (E)
Progressive social withdrawal is com monly seen
close influence of another; both individuals are usu ally
as part of the prodrome of schizo phrenia. All other mother,
and abuse— have not been proven in any
choices—head trauma, low intelligence, a neglectful
conclusive way

58

projective identification, and splitting. Splitting is


to be significantly linked to schizophrenia,
dividing external objects (individuals) into “all
although early theories held that a history of one good”
or “all bad” categories. Altruism (living
or more was a predisposing factor.
vicariously by helping others) and sub

limation (gratifying urges in socially acceptable


7. (E) While her history strongly suggests a pri mary ways)
are mature defenses, while intellectual
psychotic disorder such as brief psychotic disorder
ization (using intellectual processes to avoid
(lasting 1 day–1 month), schizo phreniform
feelings) and undoing (acts performed to undo
disorder (lasting 1–6 months), or schizophrenia,
obsessional thoughts) are considered neurotic
the presence of cannabis on her toxicology screen
defenses.
precludes such a diagnosis at this time, especially
since the onset of her cannabis use predates the 10. (E) Good
premorbid functioning portends a better
positive symptoms by a few weeks. Unless her
prognosis for this patient. Other fea tures of
symptoms persist after sobriety from cannabis is
schizophrenia that predict a better prognosis
attained, given the prominence of her psychotic include
later age at presentation, female gender,
(rather than mood) symptoms, her most acute
and rapid onset of symp toms (as opposed to
appropriate diag nosis at this time is a
insidious onset), and the presence of mood
substance-induced (spe cifically cannabis-induced)
symptoms. While predomi nantly positive
psychotic disorder. Further, if her symptom
symptoms also predict a more favorable
severity remains out of proportion, she may
prognosis, this patient has signifi cant negative
eventually meet crite ria for a comorbid primary
symptoms, as evidenced by his isolation and
psychotic disorder as well as a cannabis use
amotivation.
disorder.
11. (A) This
patient is presenting with several catatonic
8. (C) The patient meets criteria A for schizo phrenia:
features, including negativism, psychomotor
for at least 1 month she has exhibited
slowing, and echopraxia, with slight vital sign
hallucinations, delusions, and disorganized
fluctuations. Catatonia is often underdiagnosed.
speech. Further her symptoms are occur ring While
catatonia was for
outside of an acute mood episode and are not due merly
considered a subtype of schizophrenia, it is
to another medical condition. They have persisted found
more frequently in affective disor ders,
for over a year, despite sobriety from cannabis.
especially major depressive disorder. This
Since her psychotic symptoms have lasted longer
patient’s history of isolation, anhedonia,
than 6 months, her most appropriate diagnosis is
psychomotor retardation, decreased appetite, and
schizo phrenia. Brief psychotic disorder is charac guilt
(burden on fiancée) are more sug gestive of
terized by psychotic symptoms lasting 1 day to 1 major
depressive disorder. While hypothyroidism
month. Schizophreniform disorder refers to should
be ruled out, catato nia is not a common
symptoms lasting more than 1 month but less than
presentation. There is no evidence to suggest that
6 in the absence of concurrent mood disorder, he is
faking symp toms for secondary gain, as in
substance use, or another medical condition.
malingering. The lack of prior psychiatric history
or psy
chosis makes a diagnosis of catatonia associ
9. (C) The patient meets the criteria for borderline ated
with schizophrenia unlikely. There is no
personality disorder characterized by rapid mood
evidence of exposure to antipsychotics nor
swings, efforts to avoid abandonment, chronic
significant autonomic instability, so neuroleptic
feelings of emptiness, intense anger outbursts,
malignant syndrome is unlikely.
impulsivity, fluctuations between idealization and
devaluation, and recurrent self-mutilation or 12. (D) The
mainstay of treatment for catatonia is
suicidality. People with this personality disorder
pharmacotherapy with benzodiazepines or
commonly employ primitive defense mechanisms,
electroconvulsive therapy. In this case, it is
such as denial,
reasonable to attempt a trial of lorazepam
Answers: 1–12 59 60 2: Adult
Psychopathology
perfusion and
the skull is a fixed space (volume),
and assess for response; if the patient begins to the increase
in blood vol ume causes increased
become unstable or requires high doses of intracranial
pressure; a space-occupying lesion
lorazepam (>20 mg/ d), or if his blood pres sure/ reduces the
poten tial reserve the skull has to offset
pulse do not tolerate the titration of lorazepam, the increase
in pressure. Due to the transient
ECT would be the appropriate course of action. tachycardia
that occurs with seizures, a recent
Amitriptyline, sertraline, or lithium—depending myocar dial
infarction is also a relative
on whether his diagno sis is unipolar or bipolar contraindica
tion to ECT because of excess
depression—would be appropriate maintenance demands on the
myocardium. After at least
therapy once the catatonia is treated. several
months, however, cardiac risks of ECT are
the same as
those for general anesthesia.
13. (B) This woman’s presentation is most con sistent Pacemakers and
a history of traumatic brain injury
with a delusional disorder. She has no other are not con
traindications. Pregnancy is also not a
psychotic symptoms, her functioning is still good, contra
indication to ECT, and the risks are the
and the delusion is isolated to one specific belief. same as those
of general anesthesia. With recent
In addition, this patient’s delu sion is nonbizarre advances in
anesthesia, fractures are uncom mon
(i.e., could possibly occur). While delusional since
paralysis is attained for the duration of the
disorder may consist of a bizarre (could not occur) induced
seizure.
delusion, it is not as common. She does not
exhibit other psychotic symptoms, making brief 16. (B) While many
sleep disturbances have been noted
psychotic disorder unlikely; further, in brief in major
depressive disorder (as described by the
psychotic disor der, symptoms must resolve within other
choices), early morn ing awakening has been
1 month whereas hers have been occurring for at the most
consistently linked with major
least 2 months. There is no evidence she has a depression.
The decrease in REM sleep latency
para noid personality disorder. Schizophrenia and and slow wave
sleep deficits often persist, even
schizophreniform disorder are unlikely since her after
treatment.
symptoms are limited to one delusion, and she has
no hallucinations, negative symp toms, nor 17. (C) Increased
cortisol secretion in patients with
impairment in functioning. In addi tion, the time depression was
one of the earliest obser vations in
course is insufficient for these diagnoses. biological
psychiatry, and has been well borne out
in subsequent
studies. Levels of MAOs are
14. (E) This woman has a profound depression with unknown.
Catecholamines, sex hor mones, and
many lifetime episodes, including sev eral severe suicide immune
function are decreased in depression.
attempts. She has not responded to several medications
or augmen tation strategies. The persistence of suicidal 18. (B) The most
appropriate next step is to ensure the
ity in light of the treatment resistance is the most safety of the
patient and others, and escort ing the patient
pressing reason to pursue ECT. ECT can induce a rapid to the emergency
room with police assistance guarantees
response, and has excellent remission rates (up to 80%), this is preserved.
Discharge is inappropriate at this time
markedly more than for medication alone. If her thyroid as the
patient has made an active threat; while this
dys function were untreated, this would be a rea sonable
place to initiate treatment, however her thyroid
abnormality has been stabilized. Her prior history of patient is likely
in a manic episode and the
psychotic symptoms does not necessarily imply she homicidal threat
is related to the delusion of his
would be a good ECT candidate, but it is further mother-in-law,
the threat must be taken seriously.
evidence of the severity of her illness. Peripartum States differ in
the duty to warn, but at this time in
depression order to maintain
safety, admis
sion is
necessary. If on discharge the threat
remains, the
target and law enforcement may need
would spur the decision for ECT if she were to be notified
(there is a “duty to warn”). It would
currently in the postpartum state, acutely suicidal, be inappropriate
to have the wife stay with the
and/ or a threat to her infant. mother-in-law as
this would be putting her in
harm’s way.
15. (C) There is no absolute contraindication to ECT.
However, a space-occupying lesion can cause 19. (C) This patient
is suffering from a manic episode,
dangerous increases in intracranial pres sure. Since and given his
antecedent treatment with an
ECT causes a temporary increase in cerebral antidepressant,
paroxetine, it is likely the
antidepressant induced the manic switch. In with
panic-type symptoms, but again, in a healthy
addition to achieving mood stabilization through woman
with an acute onset of symptoms, the
valproate, it is critical to stop the offending agent. urine
drug screen is more likely to yield an
While SSRIs are occasion etiology.
ally used in bipolar patients with depressive
episodes, their use is not recommended as 23. (C) This
patient is manifesting signs and symptoms
monotherapy due to this potential to induce of acute
stimulant (cocaine) intoxi cation,
mania. Paroxetine levels are not used clinically. including
tachycardia, diaphoresis, and pupillary
Nortriptyline would be less appropriate than dilation.
Caffeine is not rou tinely tested on urine
paroxetine, as tricyclic antidepressants are more toxicology
screens and would not cause such a
likely than SSRIs to induce mania. Bupro severe
reaction. Can nabis intoxication may
pion is also an antidepressant and would not be present
with anxiety and paranoia, as well as
appropriate at this time. Lorazepam may help with
conjunctival injection, but it does not result in
agitation, but the more important step is to quickly dilation
of the pupils. Intoxication with heroin and
stop the offending agent. other
opiates will usually present with sedation,

respiratory depression, and constricted (pinpoint)


20. (C) This patient gives a history consistent with pupils.
LSD will often cause prominent
cyclothymic disorder, characterized by episodes of
hallucinations during intoxication.
hypomania and depressive epi sodes not meeting criteria
for MDD. Treat ment is generally the same as for bipolar 24. (B)
This patient has acute mental status changes,
disorder. For a diagnosis of bipolar I, only a manic
associated with vital sign abnor malities and
episode is necessary, though most patients will suffer
hallucinations. This presentation is suspicious for acute
from depressive episodes as well. Bipolar II disorder is delirium,
specifically delirium tremens (DTs) due to
characterized by major depressive episodes and hypo alcohol with
drawal. Untreated, DTs can be lethal. The
manic episodes. Persistent depressive disor der 62 2: Adult
Psychopathology
(dysthymia) is a chronic depression that may also meet
severity for a major depres sive episode; symptoms likelihood
for onset is highest in the third to fifth
persist for 2 years, with no more than 2 months of day after
the last drink. While postopera tive sepsis
symptom free periods. “Double depression” is an older is another
cause of delirium, lack of fever makes
term referring to individuals with persistent depressive this less
likely than alcohol with drawal. Brief
disorder developing an MDD, as well. According to the psychotic
disorder, functional neurological
DSM-5, the diagnosis symptom
disorder (conversion disorder), and
Answers: 13–24 61 delusional
disorder do not fit the acute onset of
symptoms
with alterations in the level of
would be persistent depressive disorder with
consciousness and vital sign abnormalities. In
intermittent depressive episodes. addition,
functional neuro logical symptom
disorder
does not manifest with hallucinations.
21. (B) This patient is now experiencing a major
depressive episode, which, coupled with his prior 25. (C) Benzodiazepines
are the mainstay of treatment
hypomanic episodes, would be consis tent with the for delirium tremens
(DTs). Of the choices, only
diagnosis of bipolar II disorder. See explanation diazepam and
lorazepam are benzodiazepines;
for Question 20 for remaining definitions. diazepam is less
ideal than oxazepam because it
has active metabo
22. (E) In this young patient on no medications, a urine lites and undergoes
extensive metabolism in the
toxicology screen is most likely to determine the liver. Oxazepam and
lorazepam are not dependent
cause of her current state, prob ably due to drug on liver function
for their metabo lism; they are
intoxication. Blood glucose is important to test therefore ideal to
treat DTs in patients with
since hypoglycemia can present with mental status underlying liver
disease. Disul firam is not
changes and dia phoresis; however, it would be appropriate in the
acute treat ment of DTs, but may
unlikely in this otherwise healthy woman. have a role in
sustained abstinence. Phenobarbital
Catecholamine metabolites would be premature at may be required to
induce sedation in patients
this time. An ECG would be part of a workup to who develop seizures
despite benzodiazepine
rule out any arrhythmias, but any abnormalities treatment or those
who require high doses of
would not necessarily identify a specific etiol ogy benzodi azepines
(e.g., enough to require
or account for her dilated pupils. Thyroid function intubation).
Phenytoin does not have a role in
tests are important, as thyrotoxicosis can manifest acute DTs treatment.
to make up for
any caloric intake she may have
26. (C) Delirium tremens, if untreated, has a mor tality had. Anemia is
not uncom
rate of close to 30%. Because of early recognition mon in
patients with anorexia, and fatigue is a
and improved treatments, the mortality rate of frequent
complaint. The presentation is not
DTs is now approximately 5%. consistent
with avoidant/ restrictive food intake
disorder,
which describes an eating dis
27. (B) Klüver–Bucy syndrome presents with docility, turbance
regarding food’s sensory characteris tics
lack of fear response, anterograde amnesia, or the concern
about adverse consequences of
hyperphagia, and hypersexuality. Arnold–Chiari intake not
related to body image. In buli mia
syndrome describes a condi nervosa,
patients engage in food binges,
tion with hydrocephalus and cerebellar ana tomic consuming
large quantities of food and later
and functional abnormalities. Möbius syndrome is
a congenital absence of the facial nerves and
nuclei with resulting bilat eral facial paralysis. feeling intense
guilt and shame. Patients with
Pick disease is a form of major frontotemporal bulimia may also
engage in compensatory
neurocognitive disor der (dementia), often mechanisms, such
as laxatives, ipecac, self
indistinguishable from induced
vomiting, and over exercise to “make up”
for the caloric
intake. Exercise-induced
amenorrhea is
possible, but less likely given this
major neurocognitive disorder (dementia) due to patient’s
distorted necessity to compensate for any
Alzheimer disease, in which the frontal and food intake.
Many patients with anorexia also
temporal lobes are preferentially atrophied. have obsessive-
compulsive tendencies (per
Punch-drunk syndrome describes an acquired fectionism), but
there is no indication that the
movement disorder associated with traumatic patient has OCD.
damage to the substantia nigra, for example, from
boxing. 31. (A)
Benzodiazepines are effective in acutely
stopping a panic
attack such as described in the
28. (A) Klüver–Bucy syndrome is most closely vignette.
Alprazolam has the shortest half life as
associated with severe damage to, or discon well as the most
rapid onset of action, and is
nectivity of, the amygdalae bilaterally. The therefore the
agent most likely to abort the panic
hippocampus, although important to short term attack. However,
because of these same qualities,
memory, is not directly involved with regulating it also has
significant abuse potential and should
aggressive drives, sexual behav iors, or fear only be used for
short term relief.
responses. The lateral ventricle is a Chlordiazepoxide
is a longer-acting
CSF-containing space that has no direct benzodiazepine
but does not have as rapid an
neuropsychiatric functional role. The insula, onset of action.
Divalproate is not indicated for
found deep within the central sulcus, is medial to this patient in
the absence of mood insta bility.
the temporal lobe and not involved in Fluoxetine, an
SSRI, and phenelzine, an MAOI,
Klüver–Bucy syndrome. Superior temporal gyri both are
indicated for the long term treatment of
are more generally involved with pro cessing panic disorder,
but onset of symptom relief may
complex auditory information such as the take weeks. As
such, many clinicians will start an
understanding of language. SSRI but will
also add an adjunctive
benzodiazepine
for the first few weeks of
29. (E) This patient is suffering from an eating disorder, treatment, until
the SSRI starts to take effect.
possibly anorexia nervosa. Many patients with
eating disorders have strong d rives for perfection. 32. (B) At this
time his diagnosis is most consis tent
Psychod ynamically, patients often feel a lack of with an adjustment
disorder, specifically with anxiety.
control and seek to control the one thing they Adjustment
disorder notes the onset of mood and
perceive they can, that is, food. Further, patients behavioral
changes fol lowing an acute stressor. The
also lack a sense of validation from caregivers. symptoms remit
within 6 months after the stressor.
They d on’t characteristically have legal prob lem Adjustment disorder can
be specified as occurring with
s, and often are of a high socioeconom ic status. depressed mood, with
anxiety, with mixed anxiety and
depressed mood,
with distur bance of conduct, with
30. (A) This patient’s most likely diagnosis is anorexia mixed disturbance of
emotions and conduct, and as well
nervosa. She exhibits intense food restriction as as remain ing
unspecified. In acute stress disorder, the
well as compensatory behaviors of over exercising patient is exposed to a
traumatic event and experiences
reexperiencing (e.g., flashbacks), numbing, and
increased arousal symptoms for up to 1 month after the lack of social
interaction. Agoraphobia is tied to
event. A breakup is not typically “traumatic” in the the fear of
having panic-type symptoms in public.
sense that life Such symptoms are
not mentioned in this case.
Answers: 25–35 63
Autism spectrum
disorder is a devel
opmental disorder
characterized by deficits in
is not usually threatened. While this patient has an social
communication and with repetitive,
inability to relax an associated muscle tension, restricted
behaviors and interests. Persons with
symptoms commonly associated with generalized avoidant
personality disorder are shy and fearful
anxiety disorder, his worries are limited to the of social
rejection. However, their lack of
recent relationship, and his symptoms have not socialization is
distressing to them. Schizotypal
been long-standing. The patient does not meet individuals can
have schizoid fea
criteria for major depres tures but they
also have bizarre thinking and an
sive disorder at this time, but if his symptoms odd or eccentric
manner.
persist or worsen, this diagnosis should be
considered. This patient’s symptoms are more 36. (D) Methadone
maintenance is the most appropriate
than what would be expected for a normal pharmacological
therapy for the treatment of
reaction—his sleep has been disrupted and it is opiate (heroin) use
disorder, even in pregnancy.
starting to affect his schoolwork for the past 5 Methadone
maintenance, especially when
weeks. combined with
psychoso
cial services and
obstetric monitoring, sig
33. (D) At this time the patient’s condition has nificantly
improves neonatal outcomes and
worsened, and he now has a pervasive depressed obstetric
outcomes for women addicted to heroin.
mood and disturbance in sleep, appetite, The lowest
effective dose should be used during
concentration as well as associated hopelessness pregnancy. There
are no known teratogenic effects
and worthlessness. In addition to worsening from methadone.
Women may require higher
self-care, his symptoms are now significantly doses of
methadone in the third trimester due to
affecting his schoolwork and quality of life. He increased metabo
lism of the drug at that time.
therefore now meets criteria for major depressive Buprenorphine is
a partial opiate agonist that has a
disorder, single episode, and warrants appropriate role in
maintenance treatment, but its safety
treatment, whether with medication, therapy, or during pregnancy
and breastfeeding has not been
both. definitively
established, and its use in preg nancy
may lead to
withdrawal symptoms in
34. (E) While this patient’s symptoms are consis tent opioid-dependent
fetuses. Clonidine can be used
with MDD, her ongoing alcohol use pre cludes the to treat
hypertension and other symp toms during
diagnosis at this time; therefore, acute opiate
withdrawal. While haloperidol (a
substance-induced mood disorder is the most first-generation
antipsychotic) may be used safely
likely diagnosis. If she remains sober for a period in pregnancy, it
has no use for the treatment of
of time and her depressive symptoms continue, severe opiate use
disor der, unless agitation from
she would then be diagnosed with MDD. acute withdrawal
is problematic. Naloxone is an
Amotivation syndrome is a controver sial opiate antago
nist, used for opiate overdose, not
syndrome associated with cannabis use, and is not appropri ate in
this case.
appropriate in this case given the lack of cannabis
use. In acute stress disorder, the patient is exposed 37. (C) This patient
exhibits episodic mood symp toms
to a traumatic event and experiences as well as chronic
psychotic symptoms outside of those
reexperiencing (e.g., flashbacks), numbing, and mood episodes,
making her most likely diagnosis
increased arousal symptoms for up to 1 month schizoaffective dis
order. There is no evidence of prior
after the event. Adjustment disorder is manic episodes,
making bipolar disorder unlikely.
characterized by the onset of mood and behavioral
changes following an acute stressor that are out of
proportion to a typical response. If the
hallucinations and delusions only occurred
in the context
of her depressive episodes, the most
35. (D) Persons with schizoid personality dis order are likely
diagnosis would be major depression with
detached, reclusive, demonstrate a restricted range of psychotic
features. While schizophrenia would be
affect, and do not mind the on the differ
64 2: Adult Psychopathology
ential, her affective (depressive) symptoms are common in
demented patients but not specific to
prominent. Schizophreniform disorder is NPH.
Oculomotor difficulties are a part of the
characterized by symptoms of schizophre nia, Wernicke–
Korsakoff syndrome.
which last from 1 to 6 months, so it is not
appropriate given the chronicity of her symptoms. 41. (B) In NPH, CT scan
commonly reveals dilated
ventricles
thought to be the result of increased
38. (B) This patient’s history is consistent w ith a mood pressure
waves impinging within the ventric ular
pattern defined by periods of hypoma nia system.
Cerebellar atrophy is seen most often in
(symptoms of mania not severe enough to cause congenital
disorders and alcoholism.
occupational dysfunction or psychi atric
Hypointensities found in subcortical areas are
hospitalization) and currently a major depressive often
indicative of lacunar strokes. Frontopa rietal
episode. Hypomania with major depression atrophy is
seen in major neurocognitive disorder
defines bipolar II disorder. In bipolar I disorder, due to
Alzheimer disease. Fronto temporal atrophy
the mania is more severe, causing notable is found in
Pick disease, an uncommon type of
occupational dysfunction, psychotic symptoms, or
frontotemporal neurocog nitive disorder
hospitalization. While this patient is in the midst (dementia).
of a major depressive episode, his history of hypo
mania indicates a bipolar diagnosis; this is an 42. (C) This
patient is most likely suffering from major
important distinction because improper treatment
neurocognitive disorder (dementia), characterized
with antidepressants can precipi tate a manic by
progressive impairment of cognitive function
episode. Borderline personality disorder is in the
absence of delirium. Memory impairment
characterized by a persistent pat tern of unstable and loss of
function in at least one other cognitive
relationships, mood states, and self-image. domain are
hall
Narcissistic personality dis order is characterized marks. While
some loss of cognitive function is
by a pervasive pattern of grandiosity, need for expected with
age (mild neurocognitive disorder
admiration, and lack of empathy. does not
interfere with capacity for

independence), it is too severe and this patient is


39. (C) Dry mouth, dizziness (associated with no longer
able to function independently.
hypotension), and urinary hesitancy are due to Delirium is
characterized by acute mental sta
anticholinergic and antiadrenergic effects of tus change
with deficits in attention (i.e., fluc
tricyclic antidepressants (TCAs) such as tuating
levels of consciousness). While major
imipramine. Divalproex sodium is a mood depressive
disorder can present with neuro
stabilizer that may cause gastrointestinal upset and cognitive
deficits (“pseudodementia”), there is no
is commonly associated with seda indication of
depression. This patient does not
tion and tremor at higher doses. Fluoxetine, a demonstrate
the gait apraxia and urinary
serotonin-specific reuptake inhibitor (SSRI), is incontinence
characteristic of normal pressure
most often associated with gastrointestinal upset,
hydrocephalus.
sexual dysfunction, and activation/
Answers: 36–44 65
agitation. Lithium is another mood stabilizer used
in bipolar disorder, most often caus ing polyuria, 43. (A) The most
common type of major neurocog nitive
polydipsia, tremor, and men tal confusion at higher disorder
(dementia) is due to Alzheimer disease,
doses. Phenelzine, a which
demonstrates diffuse atrophy on

neuroimaging. Major neurocognitive dis order due


to Lewy
body disease is clinically similar to
monoamine oxidase inhibitor (MAOI), can be
Alzheimer’s, but also includes hallu cinations,
associated with hypotension but is less likely to
parkinsonian features, and extrapy ramidal signs.
have anticholinergic effects. Major
neurocognitive disorder due to traumatic
brain
injury is characterized by emotional lability,
40. (C) This patient likely has normal pressure
dysarthria, and impulsiv ity. Major neurocognitive
hydrocephalus (NPH), one of the few poten tially disorder
due to vas cular disease (formerly
reversible causes of neurocognitive dis order multi-
infarct dementia) is the second most
(dementia). The classic triad of NPH is confusion, common
type of demen tia, and involves small
gait apraxia, and incontinence. Ele vated opening widespread
lacunar infarcts. It is seen in those
pressure is not found in NPH (thus the “normal with
hypertension or other cardiovascular disease;
pressure” part of the diag nosis). Frontal release this
patient does not have either conditions.
signs and perseveration are nonspecific findings
Unspecified major neurocognitive disorder is
reserved for patients in whom the precise etiology schizophrenia
except that the duration of his
cannot be deter mined with sufficient certainly to illness has
been less than 6 months. Brief
make the appropriate attribution. All the major psychotic
disorder may present with psycho
neuro cognitive disorders must be distinguishes sis but the
duration of the disturbance must be 1
from major depression; “pseudodementia” refers month or
less. This patient’s delusions are too
to a condition where the patient demonstrates bizarre to
typically be consistent with delusional
symptoms consistent with memory difficulties, disorder, and
the age of onset for delusional
but in the absence of a dementia; it is often seen in disorder is
usually middle age. There is no
depression. Patients with pseudodemen tia usually evidence of a
depressive disorder at this time.
have more depressive symptoms, have more
insight into their symptoms than demented 47. (A) This
patient has symptoms consistent with
patients, and will often have a prior history of PTSD or acute
stress disorder (i.e., traumatic
depression. event,
intrusive memories of trauma, avoid ance of
reminders,
hypervigilance), but the fact that the
44. (D) Major frontotemporal neurocognitive dis order trauma was
only 3 weeks prior means that she
(formerly Pick disease) is characterized by would be
diagnosed with acute distress disorder.
preferential atrophy of the frontotemporal region, For PTSD,
there must also be evidence of
as opposed to the parietotemporal dis tribution negative
cognitions or moods. If her symptoms
seen in Alzheimer disease. Patients may exhibit did persist
after 4 weeks, then she would be
either a behavioral variant or lan guage variant, but diagnosed
with PTSD. Adjust
many have features of both. Patients with the ment disorder
most often represents a change in
behavioral variant must show three or more mood,
anxiety, or conduct that happens after a
behavioral symptoms (behavioral disinhibition, nontraumatic
event. In GAD, there is no
apathy or inertia, loss of sympathy or empathy, associated
trauma, and the symptoms of
perseverative/ stereotyped/ ritualistic behaviors, or
hypero rality and dietary changes) or a prominent
decline in social cognition and/ or execute anxiety
last for at least 6 months. The patient has
abilities. Patients with the language variant no
evidence of a primary depressive illness such
display a decline in language ability (difficulties as MDD.
in speech production, naming, word finding,
comprehension). 48. (C) The
lifetime incidence of suicide in patients w
66 2: Adult Psychopathology ith
schizophrenia is approximately 10%,
compared
with less than 1% in the general
45. (E) The DSM-5 distinguishes the severity of alcohol
population.
(and other substance) use disorders on the basis of
how many criteria are met. Cri teria A include 49. (A)
Clozapine, the first atypical antipsy chotic, has
evidence of a problematic pat tern of alcohol use been shown
to decrease suicid ality in patients
that results in impairment and/ or distress. Patients with
schizophrenia and is particularly effective in
meeting 2 to 3 symp toms meet criteria for mild treatment-
refractory patients, such as the one in
alcohol use dis order; those that have 4 to 5 this
example. Clo zapine requires regular blood
symptoms meet criteria for moderate alcohol use monitoring
to screen for the potentially serious
disorder; those that meet 6 or more of the possible side
effect of agranulocytosis. None of the
listed 11 symptoms can be diagnosed with severe remaining
anti psychotics, such as fluphenazine,
alcohol use disorder (formerly alcohol depen
haloperidol, or ziprasidone, have been shown to
dence). The DSM-5 does not distinguish abuse be effec
tive in treatment-refractory schizophrenia
from dependence, and requires the number of or to
decrease suicidality. While lithium has also
problematic symptoms to determine severity. This been shown
to decrease suicidality, its use is not
patient drinks alcohol in large amounts, has had indicated
in this case given the lack of any
unsuccessful efforts to cut down, has cravings, has affective
symptoms.
had persistent interpersonal problems due to
drinking, has had recurrent physical problems due 50. (A) Feelings
and attitudes originating from the
to drinking, and has had withdrawal symptoms; he clinician,
evoked by the patient, are called
meets at least six criteria and therefore can be
countertransference. Transference denotes feel
diagnosed with alcohol use disorder, severe. ings and
attitudes about the physician coming
from the
patient. Empathy is the ability of the
46. (D) This patient would meet the criteria for clinician
to psychologically “put oneself in the
patient’s shoes” and thereby understand the treatment is
psycho therapy. Medication can be a
patient’s thinking, feelings, or behavior. Iden useful adjunct
in managing symptoms sufficiently
tification is a defense mechanism characterized by so that the
patient can later engage more
the unconscious incorporation of some one else’s effectively in
psychotherapy. Benzodiazepines
traits into one’s own manner—for example, may treat
anxiety in these patients, but they have
adolescents having hairstyles similar to admired particular abuse
liability in these impulsive
rock stars. Projection is a primi tive defense patients.
mechanism whereby one assigns emotions to
another person in an attempt to psychologically 53. (B)
Psychotherapy is the mainstay of border line
defend against (cover up) the presence of those personality disorder
treatment. Of the available types of
emotions within oneself; for example, the doctor therapy, DBT has shown to
be particularly effective with
in the case may tell his fel low resident that he borderline
indi viduals, especially when used in
believes that his female patient is attracted to him,
conjunction
when in actuality he is attracted to her.
Answers: 45–56 67

51. (A) This patient displays symptoms of fear of loss with groups.
DBT is an offshoot from CBT and
and abandonment, intense interpersonal focuses on
mindfulness and distress tolerance.
relationships, recurrent suicidal behavior or CBT is often
a short course of therapy focused on

dysfunctional thoughts with their result


ing
behavioral and emotional manifestations.
threats, affective instability, and difficulty Group
therapy is a component of DBT, but should
controlling intense anger, consistent with be used in
conjunction with, not in place of,
borderline personality disorder. Patients with individual
therapy. Psychoanalysis and
cyclothymic disorder often present with similar
psychodynamic therapies utilize psychoana
symptoms of mood changes due to hypomanic lytic
principles and focus on formative child hood
episodes and minor depressive episodes, but they experiences
and the recurrent patterns of
are not usually as frequent, short-lived, or intense, behaviors in
interpersonal relationships, unre
and they are not asso ciated with the other solved
feelings, and building of ego defenses.
symptoms of borderline personality disorder While these
therapies may be of benefit, the
above. Patients with his regression
and intense emotions evoked may be
trionic personality disorder, like those with too
difficult for the patient to handle.
borderline personality disorder, may display
excessive emotionality and attention seeking, but 54. (A) Amnesia is
a common side effect of ECT,
their core symptoms center around super presenting
in the form of short-term memory
ficial seductiveness and theatricality. While deficits.
Aspiration, fractures, and arrhyth mias are
patients with borderline personality disorder may rare side
effects. Psychosis can be improved,
have depressive symptoms that develop into rather than
worsened, with ECT; in fact, ECT is a
MDD, this patient’s depressive symptoms are treatment of
choice in patients with major
fleeting and in response to stressors such as depressive
disorder with psychotic features.
perceived abandonment. Although this patient
appears paranoid that “all the staff hates me,” 55. (C) For major
depressive disorder, 6 to 12 sessions
there is no other evidence for a primary psy chotic are generally
optimal for a good risk–benefit ratio
illness such as schizoaffective disorder. Of note, of positive
treatment effects versus memory
however, patients with borderline personality impairment
(which is more likely with greater
disorder may experience tempo than 20
treatments). For catatonic conditions, two
rary psychotic symptoms, such as paranoia, or or four ECT
treat
severe dissociation under times of perceived stress ments may be
effective. When ECT is used for
or substance use. psychosis or
mania, a larger number of treatments
may be
necessary.
52. (D) Psychotherapy and steady social support
represent the best long-term method for the 56. (B) This
patient has distressing recurrent and
management of borderline personality dis order. intrusive
thoughts (obsessions). She attempts to
Antidepressants, antipsychotics, and mood ignore or
suppress her symptoms by not
stabilizers do have some efficacy in treating target explaining to
her husband how she feels and by
symptoms in some patients with borderline getting rid
of all the knives in her kitchen.
personality disorder, but the long-term mainstay Obsessions
are the mental acts, whereas com
pulsions are physical actions performed to
decrease anxiety associated with the obses sions.
Obsessive-compulsive personality dis order is a present with
a more stable MSE than is illus trated
pervasive character style marked by a pattern of in this
case; in addition, there is no evi dence that
preoccupation with orderliness, perfectionism, and this woman
is depressed. Although this patient
mental and interpersonal control. These would be
likely to have a neuro cognitive disorder
individuals are often described as “control freaks” (dementia)
as she came from a nursing home, her
by laypersons and are fluctuating
mental status is indicative of a
68 2: Adult Psychopathology delirium.
Dementias are diagnosed in the context
of a
relatively sta ble set of deficits on MSE, and
noted to be particularly inflexible. Delusional the
underlying causes are rarely associated with
disorder or schizophrenia are inappropri ate acute medi
cal insults. There is no evidence of a
diagnoses because this patient is not psychotic; substance or
other cause of a major
her worries are conceived of as a product of her
neurocognitive dis order (dementia). Importantly,
mind and not as implanted messages, as may however, the
presence of a major neurocognitive
occur in schizophrenia. There is no mention of a disorder
(dementia) does predispose a patient to a
history of odd beliefs or strangeness prior to the delirium.
birth of her child, which would be consistent with
schizotypal personality disorder. 59. (B)
Polypharmacy can worsen delirium. Anti
psychotics
such as haloperidol can be used to
57. (C) Fluvoxamine and other serotonin-specific calm an
agitated patient with delirium.
reuptake inhibitors (SSRIs) (e.g., paroxetine,
Anticholinergic agents like diphenhydramine
fluoxetine, sertraline, citalopram) are known for should be
avoided as they can worsen or cause
their antiobsessional effects as well as for their delirium,
and sedating medications should be used
antidepressant and antianxiety effects. Patients at lowest
possible doses. Benzodiaz epines such as
may require higher doses than those needed for lorazepam
should be avoided in most cases of
depression. Lithium is a mood sta delirium
(with the exception of sedative-hypnotic/
bilizer used for bipolar disorder. Lorazepam, a alcohol
withdrawal) as they can disinhibit patients
benzodiazepine, may decrease anxiety but has no and worsen
the behavioral problems. In cases of
direct antiobsessional effects. Haloper idol is a delirium
with very severe agitation, phenobarbital
typical high-potency antipsychotic (neuroleptic) may be used,
but this would not be the appropri
that may be useful in severe forms of OCD in ate
management in this case. Valproic acid, an
which there is a psychotic component. However, it
anticonvulsant and mood stabilizer used in bipolar
does not ameliorate purely obsessional thinking. disorder, is
not indicated in this patient.
Because of the importance of serotonin in the
treatment of obsessive-compulsive disorder, of the 60. (A) The
definitive treatment for delirium is to treat
tricy the
underlying cause, in this case, treat ment of the
clic antidepressants (TCAs), clomipramine would urinary tract
infection with anti biotics. Mental status
be preferred if this class were selected. However, needs to be
monitored carefully as some antibiotics can
TCAs are not first-line treatment for OCD. also worsen
delirium. Scheduled chest x-rays or CT
scans have
no role in treatment of this patient’s con
58. (A) This patient’s presentation and examina tion are dition and
carry unnecessary radiation expo sure. An
most consistent with delirium, as evi denced by a indwelling
Foley catheter would increase the risk of
fluctuating mental status with attentional deficits. Any developing a
urinary tract infection, and unless there is
number of medi cal conditions can cause delirium, and indication, should
be avoided. Any associated electrolyte
the elderly are particularly prone. This woman is abnormalities
that may have resulted from the patient’s
elderly, dehydrated, and with an active infec tion; any
poor oral intake need to be corrected,
one of these can increase the risk of delirium. Urinary
tract infections are a fre quent cause of altered mental
status in the elderly. A chest x-ray can also elucidate the but
intravenous fluids will not, by themselves,
presence of infection, and a head CT can rule out acute treat the
infection or the delirium.
intracranial pathology, both of which can cause delirium.
While major depressive disorder can cause cognitive 61. (C) Delirium
carries a high-mortality rate and is a
deficits on MSEs that are reversible with antidepressant poor prognostic
sign. The 1-month mortal ity rate of
treatment, these “pseudodementias” usually patients who have
had one episode of delirium is up to
14%, and the 6-
month mortality rate is approximately
22%. in their
teens or twenties. It is a dis order of REM
sleep
mechanisms, character ized by cataplexy
62. (C) About one-third of all patients with depression (sudden
loss of muscle tone following intense
respond to the surprisingly pow erful placebo
emotion), hypnopompic or hypnagogic
effect. However, this response is not as well
hallucinations, or sleep paralysis (an inability to
sustained as in patients placed on antidepressant perform
voluntary movements either at sleep
medications. onset or
awakening that can be terrifying).

Circadian rhythm sleep–wake disorder is a


63. (D) Standard antidepressant therapies, includ ing disorder
caused most often by sleep-scheduling
SSRIs and TCAs, have a positive response in 55% changes
such as night-shift work.
to 65% of patients.
Hypersomnolence disorder describes self-reported

excessive sleepiness despite main sleep period of


64. (C) This patient likely has major depressive
sufficient time. Insomnia disorder notes a
disorder. The rate of completed suicide in
complaint of dissatisfaction with sleep quality and
hospitalized patients with major depression is
quantity. Nightmare disorder is diagnosed in the
approximately 10% to 15%. Most patients with absence
of other predisposing mental illness (e.g.,
major depression will have some degree of PTSD),
in which the patient has repeated
suicidal ideation, and up to 40% of patients with
nightmares causing significant distress.
major depression will attempt suicide. Screening
for suicide should be performed in any patient 68. (D)
Methylphenidate, a stimulant, has been shown
with depression, and studies have shown that to be
useful in inhibiting the onset of narcoleptic
asking about suicide does not plant the thought in episodes,
likely owing to the capacity for the drug
patients; most patients who have attempted to enhance
CNS arousal mechanisms that inhibit
suicide have seen a doctor within 4 weeks of the REM-
related mecha
attempt. nisms.
Antidepressant treatment with tricyclic

antidepressants has been found to be useful in


65. (C) The number one predictor of a completed
combination with methylphenidate in some
suicide is a previous attempt. Older single (or patients
with narcolepsy. However, the efficacy of
divorced) males are more likely to attempt. bupropion,
fluoxetine (a serotonin-specific
Religion, good support, and children can be reuptake
inhibitor), and phenelzine (a mono
protective factors. Concurrent substance use and amine
oxidase inhibitor) has not been well stud ied
anxiety disorders can increase the risk of suicide. in
narcolepsy. Benzodiazepine regimens are not

particularly effective for narcolepsy.


66. (C) There is no evidence of occupational or social
dysfunction or marked distress caused by this patient’s 69. (D) This
patient’s symptoms fit DSM-5 crite ria for
activities. Virtually all DSM-5 diagnoses are qualified by PMDD. The
remittance of symptoms in
these criteria. If the patient did describe such concerns, 70 2: Adult
Psychopathology
the diag nosis would most likely be sexual masochism
disorder, in which patients are aroused by psy the week
after menses is typical of PMDD. In
chologically or physically punishing acts by another adults,
persistent depressive disorder is char
person (or fantasies of punishment). In acterized
by subthreshold depressive symp toms,
Answers: 57–69 69 for more
days than not, over a 2-year period. Her
symptoms
are not severe enough nor of sufficient
sexual sadism disorder, the patient is aroused by duration (2
or more weeks) to diagnose MDD, and
giving such punishment (or fantasies of giv ing it) her
symptoms are more disruptive and cause more
to others. Fetishistic disorder involves sexual distress
than purely PMS.
arousal connected to nonliving objects. In
frotteuristic disorder, the patient is sexually 70. (C) This
patient is displaying symptoms con sistent
aroused by touching or rubbing up against a with major
depressive disorder, single episode. The
nonconsenting person. All are examples of average
untreated depressive episode lasts anywhere
paraphilias—persistent sexual interest other than from 6 to
13 months; with treatment, the duration
sexual interest in genital stimulation or decreases to
approximately 3 months. Treatment can
preparatory fondling with normal, mature, also
decrease the risk of recurrence in the future.
consenting human partners.
71. (C) Heroin overdose
is most likely to have caused
67. (D) Narcolepsy is a disorder often affecting persons the clinical
situation described, includ ing the
decreased level of consciousness, respiratory therapy is a
long-term dynamic therapy. Although
depression, and pinpoint (con stricted) pupils. it may be use ful
in some cases of panic disorder, it
Alcohol intoxication and PCP may cause coma has not been as
valid ated as CBT. Interpersonal
but both are associated with nystagmus rather than ther apy
addresses relationships as a contributor of
pupillary size changes. Cocaine use causes depression and is
used to treat individu als w ith
pupillary dilatation, not constriction, and is major depression.
The purpose of supportive
usually associated with agitation in large amounts. psychotherapy is
to strengthen the patient’s d
Inhalants may rarely cause coma but are not efense mechanisms
in ord er to return them to a
classically asso ciated with pupillary constriction. previous level of
function ing. It has not been
adequately
studied for panic disorder.
72. (E) Naloxone is used to reverse the acute effects of
opiate overdose by blocking CNS opioid 75. (A) This case
depicts a patient who is feign ing or
receptors. Acetylcysteine is administered in producing symptoms
of an illness to gain gratification
acetaminophen overdose and deferoxamine is by assuming
the sick role. Factitious disorder is
used in iron overdose. Methadone is used for the sometimes referred to
as Munchausen syndrome; when
long-term maintenance of opiate addiction and caretakers
intentionally cause illness in their charges,
would only worsen the symptoms of her such as children, the
disorder is called fac titious disorder
oin overdose. Methylene blue is used to treat imposed on another. In
functional neurological symptom
methemoglobinemia. disorder
(conversion disorder), patients present with
neurologic symptoms
that are not consistent with known
73. (D) This patient is suffering from symptoms of
anatomy or physiology, and the
posttraumatic stress disorder (PTSD), manifesting
in some patients following the exposure to
threatened or actual death, severe injury, or concern is usually
with a specific symptom, not
violence, and consisting of intru having a particular
disease. In illness anxi ety
sion symptoms, avoidance of related stimuli, disorder, patients
have few somatic com plaints but
negative alterations in mood and cognitions, and are primarily
concerned with the idea of being
increased autonomic arousal. The best studied and sick and having or
acquiring a serious illness; the
most efficacious medications are intensity is not so
severe as to warrant a diagnosis
of delusional
disorder. Somatic symptom disorder
is characterized by
multiple, current, somatic
considered to be the SSRIs. Antipsychotics, symptoms that are
distressing or result in
including the atypicals, should not be used as significant
disruption. There is no reason to
monotherapy, although may be used in believe this act
could help her achieve financial or
conjunction, especially if there are associated other material gain
as in malingering.
psychotic symptoms. Benzodiazepines should be
avoided in this patient population not only 76. (C) Patients with factitious
disorder have an
because of the significant comorbidity with unconscious desire to
assume the sick role,
substance addiction but also as they have not been although their symptom
production is fully
found to be particularly effective. Mood conscious. In
malingering, both symptom
stabilizers, such as lithium and valproic acid, are production and the
desire for secondary gain
occasionally used in addition to SSRIs in order to (work avoidance) are
conscious. In functional
target the mood lability or aggression sometimes neurological symptom
disorder, the symptom
seen in PTSD, although they are not as effective production is believed
to be due to uncon scious
as SSRIs. conflicts.

74. (A) This case is an example of panic d isorder and 77. (B) This patient meets
criteria for a current manic
agoraphobia inadequately treated with appropriate episode with
symptoms of psychomo tor agitation,
med ication, namely sertraline. Many mental pressured speech,
grandiose delusions, flight of
illnesses, including anxiety disorders, are best ideas, and history
of exces sive spending as
treated with a combina evidenced by his
credit card debt. His history of
tion of pharmacology and psychotherapy. The good premorbid
function ing and a remitted major
best-stud ied psychotherapy for panic disord er is depression are also
consistent with a diagnosis of
CBT. Eye movement d esensitiza tion and bipolar disor der.
Adjustment disorders can present
reprocessing (EMDR) is a specific therapy that follow ing an
identifiable stressor and may
has been developed for PTSD. Insight-oriented manifest with mild
mood, anxiety, or behavioral
distur bances, but not overt mania. Brief psychotic doses, such as with
IV steroids, steroid psychosis
disorder is characterized by psychotic symp toms can occur in
patients treated with chronic lower
lasting 1 day to 1 month, but which are not better level oral steroids
as well. Her psychotic
accounted for by another mood dis order with symptoms rule out an
adjust ment disorder, and her
psychotic features (such as bipolar disorder) or age and lack of
psychi atric history would not be
psychotic disorder. Cyclothymic disorder is consistent with a
first episode of bipolar disorder
incorrect as this patient has had both manic and or schizophrenia.
depressive episodes, whereas in cyclothymia the
highs do not exceed a hypo manic episode, and the
lows do not extend to a major depressive episode.
The patient’s cur rent manic episode rules out
MDD; in fact, although he may have future major
depres sive episodes, he should never be diagnosed
with MDD.
Answers: 70–80 71

78. (D) Lithium and valproic acid, along with atypical


or second-generation antipsychot ics (e.g.,
olanzapine) are considered first-line treatments for
acute mania, as well as preven tion of future mood
episodes. Carbamazepine is also an effective
mood stabilizer, but due to side effects is not
usually first line. While halo peridol can be used to
treat the acute agitation and psychotic symptoms
in bipolar disorder, it is not as effective for the
mood component or in preventing episodes of
mania, and its side effects (e.g., extra-pyramidal
symptoms, tardive dyskinesia) limit its use as
monother apy. Lamotrigine and lithium are both
used in the treatment of acute depressive episodes
of bipolar disorder. Antidepressants such as ser
traline should be avoided in this case as they can
worsen mania and likely promote rapid cycling of
mood episodes.

79. (B) Classically, infarcts of the left frontal hemi


spheres (part of left middle cerebral artery dis tribution)
present with depression, whereas those of the right
frontal hemisphere present with euphoria, inappropriate
indifference, or mania (although recent research tends to
refute this). Obsessive-compulsive behaviors pres ent
occasionally after diffuse bilateral frontal injury. Anxiety
and panic symptoms have not been described as having
any particular asso ciation with left middle cerebral
artery strokes, although such comorbid cases may exist.

80. (E) This woman has developed psychosis marked


by hallucinations, delusions, and paranoia. She
was most likely given high-dose steroids to treat
her acute multiple sclerosis flare, and the steroids
induced a psychotic state. High-dose steroids can
cause mood dis
turbance in many patients, and florid psycho sis in
a smaller subset. The symptoms usually entirely
remit once steroids are stopped or tapered
appropriately. While most symptoms occur at high

You might also like